Sunteți pe pagina 1din 59

and Other Important Modalities

Mark Rodrigues and Zeshan Qureshi

Join Our Medical Book Writing Project (details inside)

Bonus Orthopaedic X-Rays

Core Radiology Curriculum Covered: 100 Annotated X-Rays (including how to present them), 300 Multiple Choice Questions (with detailed explanations)

Bonus Abdominal X-Rays

Orthopaedic X-Rays, plus CTs, MRIs

Radiology: Chest, Abdominal and

Bonus Chest X-Rays

The Unofficial Guide to

Fluoroscopy

Nuclear Medicine Scans

USS Scans

MRI Scans

CT Scans

Orthopaedic X-Rays

Abdominal X-Rays

Chest X-Rays

Introduction

THE UNOFFICIAL GuIDE tO RADIOLOgY


FIRST EDITION
MArK RodriGUes BSc (Hons) MBChB (Hons)
Radiology Registrar, Royal Inrmary of Edinburgh; Honorary Clinical Tutor, University of Edinburgh, UK

ZeshAn QUreshi BM BSc (Hons)


Academic Clinical Fellow (International Child Health) Great Ormond Street and Institute for Global Health, London; Honorary Clinical Tutor, University of Edinburgh, UK

INTRODUCTION
The Unofcial Guide to Radiology is the fth book in the Unofcial Guide to Medicine series. Almost every patient has some form of medical imaging performed during his or her investigations and management. The commonest form of imaging, and the modality which all doctors should be able to interpret, remains the X-ray. This important aspect of radiology is therefore the main focus of the book. Other imaging modalities are specialised investigations interpreted by radiologists. However, medical students, doctors, nurses, physicians associates, and surgeons need to understand what these tests involve, when they are indicated and contraindicated, and how to best request them. Therefore, these aspects are also covered in the book.

Despite its universal importance, X-ray interpretation is often an overlooked subject in the medical school curriculum, which many medical students and junior doctors nd difcult and daunting. I was no different when I started work as a junior doctor. However, since starting radiology training, I have realised X-ray interpretation should not be that way. The keys to interpreting X-rays are having a systematic method for assessing the X-ray and getting lots of practice at looking at and presenting X-rays. Occasionally, there may be a complex X-ray you nd difcult, or a subtle nding you overlook, but thats what keeps people like me in a job, so do not worry about it. The 4 Ds are a useful framework for X-ray interpretation which underpins the approach used in this book. First, you need to Detect and Describe the abnormalities on the X-ray. You then need to form a Differential diagnosis based on clinical and X-ray ndings before Deciding what further imaging and management is required. There are lots of radiology textbooks available, but I do not think there is one which is ideally suited for teaching medical students and junior doctors. Many have small, often poor quality images. Radiology is a visual subject and therefore such images are difcult to use to demonstrate key clinical ndings. This is confounded by the fact that the ndings are usually only described in a gure below the image, and it is often difcult to know exactly what part of the image corresponds to which nding! Another fundamental problem with many radiology textbooks is that they deal with X-rays in isolation. In reality, X-rays are part of the clinical assessment and management of patients, and thus they should be taught in a clinical context. The content, layout and approach used in this book are designed to make it as useful and clinically relevant as possible: Over 200 large, high quality radiological images are used throughout the book and important ndings are annotated on the images to highlight the key points and ndings to the reader. The chest, abdominal and orthopaedic X-ray chapters contain step-by-step approaches to interpreting and presenting X-rays. Each of these chapters also covers 20 common and important X-ray cases/diagnoses. They are labelled as Case X to not give away the diagnosis, but at the end of the book there is a list of all the diagnoses. The X-rays are presented in the context of a clinical scenario. The reader is asked to present their ndings before turning over the page to reveal a model X-ray report accompanied by a fully annotated version of the X-ray. This encourages the reader to look at the X-ray thoroughly, as if working on a ward, and come to their own conclusions about the X-ray ndings and any further management required before seeing the answers. To further enhance the clinical relevance, each case has 5 clinical and radiology-related multiplechoice questions with detailed answers. These are aimed to test core knowledge needed for exams and working life, and illustrate how the X-ray ndings will inuence patient management.

The bonus X-ray chapter provides over 50 further X-ray cases to help consolidate the readers knowledge and provide an opportunity to practice the skills they have learnt. Five chapters are devoted to other important imaging investigations: computed tomography (CT), magnetic resonance imaging (MRI), ultrasound (USS), nuclear medicine, and uoroscopy. These cover the details of what the examinations entail, their common indications, contraindications and key imaging ndings. The content is in line with the Royal College of Radiologists Undergraduate Radiology Curriculum 2012, making it up to date and relevant to todays students and junior doctors.

With this textbook, we hope you will become more condent and competent in these radiology competencies, both in exams and in clinical practice, and we also hope that this is just the beginning. We want you to get involved, this textbook has been a collaboration with junior doctors and students just like you. You have the power to contribute something valuable to medicine; we welcome your suggestions and would love for you to get in touch. A good starting point is our Facebook page, which is growing into a forum for medical education: Search for The Unofcial Guide to Medicine or enter the hyperlink below into your web browser.

Please get in touch and be part of the medical education project.


Mark Rodrigues, mark.a.rodrigues@gmail.com Zeshan Qureshi, zeshanqureshi@doctors.org.uk, @DrZeshanQureshi

Facebook: http://www.facebook.com/TheUnofcialGuideToMedicine

FOREWORD
Radiology is encountered every day by medical students. From the wards to OSCEs, from theatre to outpatient clinics, radiology is everywhere at the undergraduate level and beyond. The importance of understanding the principles of imaging, radiation doses and the clinical interpretation of results is paramount. Radiology is a key diagnostic and monitoring tool in modern medicine, so the ability to assess an X-ray in a systematic order is a vital skill. The vast range of techniques, and the complexity of the human body, means this is not a subject that can be learnt overnight, and this book aims to provide you with a grounding in this mammoth specialty. The authors have ensured they have included the basic scientic principles underlying radiology. The book covers many imaging modalities and presents them in a systematic James Brookes, order to give you a clear approach to interpreting what you see. Detailed pictures along Medical Student, University of Southampton the way point out normal anatomical features as well as deformities and anomalies. Perhaps one of the biggest strengths of this book is the cases section, which allows you to practice not only interpreting high quality images but also to link them to a case history. The questions that follow not only test your radiology knowledge, but also your understanding of signs, symptoms, underlying pathophysiology and management of the condition. As well as providing detailed answers in each section, the book also shows you the best way to present each case, whether in an OSCE situation or on a ward round. The Unofcial Guide to OSCEs has quickly become established as one of the most useful undergraduate books. The ease of use, detailed pictures and emphasis on key points of this title should cement it as the number one undergraduate book for radiology. I hope you nd it invaluable throughout your studies and it brings you success in all of your exams!

James Brookes

Which radiographs from each system are most likely to be presented in exams? This excellent book presents the classics, and at one level this makes it a high-yield textbook that will be extremely valuable to medical students and junior doctors. But it is much more than that. Not only does it teach pattern recognition, it also clearly and simply explains the underlying concepts which make such images easier to interpret and answers all those tricky questions that return to haunt clinicians on a regular basis. For example, when should I use CT and when would magnetic resonance imaging be more appropriate?
Bob Clarke, Associate Dean, Professional Development, London. Director, Ask Doctor Clarke Ltd.

This book also teaches a systematic approach to reporting, with the bonus cases particularly useful in enabling readers to check that they have learnt from the core cases that have gone before. This interactivity is essential to its success and the skills acquired will transfer to life beyond the exams. What is especially striking is the denition and clarity of the illustrations, with onimage labelling enabling one to be absolutely certain of which is the endotracheal tube, the nasogastric tube and the central line, for example.

Mark Rodrigues and Zeshan Qureshi are to be congratulated on producing this excellent volume. As with the other books in this series, the multi-author collaborative approach works exceptionally well and the democratisation of the reviewing process ensures that this will meet the needs of medical students and junior doctors, both in their exams and in their day to day work. In summary, this is another classic in the Unofcial Guide series.

Bob Clarke

ABBreViationS
A&E AAA ACE ACJ ADEM ADH AIN AMT ANCA AP ATLS AV AVN AXR BTS CABG CBD CCAM cm COPD CPAP CRP CSF CT CTPA DDH DEXA DHS DIPJ DMSA DTPA DVT DWI ECG accident and emergency abdominal aortic aneurysm angiotensin converting enzyme acromio-clavicular joint acute disseminated encephalomyelitis anti-diuretic hormone anterior interosseous nerve abbreviated mental test anti-neutrophil cytoplasmic antibodies anterior to posterior Advanced Trauma Life Support arteriovenous avascular necrosis abdominal X-ray British Thoracic Society coronary artery bypass graft common bile duct congenital cystic adenoid malformation centimetre chronic obstructive pulmonary disease continuous positive airway pressure ventilation C-reactive protein cerebrospinal uid computer tomography computer tomography pulmonary angiogram developmental dysplasia of the hip dual energy X-ray absorptiometry dynamic hip screw distal interphalangeal joint dimercaptosuccinic acid diethylenetriaminepentaacetic acid deep venous thrombosis diffusion weighted image electrocardiogram ENT ERCP ESR ESWL ET EVAR FAST FDL FDP FDS FLAIR FOOSH G g/L G&S GCS GI GTN HAS HIDA HIP HLA HPOA HRCT HRT HU ear, nose and throat endoscopic retrograde cholangiopancreatogram erythrocyte sedimentation rate extracorporeal shock wave lithotripsy endo-tracheal endovascular aneurysm repair focused assessment with sonography in trauma exor pollicis longus exor digitorum profundus exor digitorum supercialis uid attenuation inversion recovery falls onto an outstretched hand gauge grams per litre group and save Glasgow coma scale gastrointestinal glyceryl trinitrate human serum albumin hepatobiliary iminodiacetic acid heparin-induced thrombocytopaenia human leukocyte antigen hypertrophic pulmonary osteoarthropathy high resolution CT hormone replacement therapy Hounseld unit

IRMER ionising radiation [medical exposure] regulations ITP ITU IU IUCD IV IVC idiopathic thrombocytopenic purpura intensive treatment unit international unit intrauterine contraceptive device intra-venous inferior vena cava

ABBreViationS
IVU JVP kg KUB LDH LIF LLL LUL m MAG3 MCPJ MDP MDT MIRP mm mmHg intravenous urogram jugular venous pressure kilogram kidneys, urethra, bladder lactate dehydrogenase left iliac fossa left lower lobe left upper lobe metre methyl-acetyl-gly-gly-gly metacarpophalangeal joint methylene disphosphonate multi-disciplinary team minimally invasive retroperitoneal pancreatic necrosectomy millimetre millimetres of mercury SLE SMA SMV SP STIR SUFE TB THR TNF TNM U&Es USS VQ VTE -HCG PET PFO PICC PIN PIPJ PR RLL RML RUL RUQ positron emission tomography patent foramen ovale peripherally inserted central catheter posterior interosseous nerve proximal interphalangeal joint per rectum right lower lobe right middle lobe right upper lobe right upper quadrant

SCIWORA spinal cord injury without radiological abnormality SHO SI SiADH senior house ofcer sacroiliac syndrome of inappropriate anti-diuretic hormone systemic lupus erythematosus superior mesenteric artery superior mesenteric vein spinous process short tau inversion recovery slipped upper femoral epiphysis tuberculosis total hip replacement tumour necrosis factor tumour, nodes, metastases urea and electrolytes ultrasound scan ventilation/perfusion venous thromboembolism beta human chorionic gonadotrophin

mmol/L millimoles per litre MRA MRCP MRI MRSA mSv MTPJ NAI NG NSAID OA PA PaCO2 PaO2 PE PEA PEG magnetic resonance angiography magnetic resonance cholangiopancreatogram magnetic resonance imaging methicillin resistant Staphylococcus aureus milliSieverts metatarsophalangeal joint non-accidental injury naso-gastric non-steroidal anti-inammatory drug osteoarthritis posterior to anterior partial pressure of carbon dioxide partial pressure of oxygen pulmonary embolus pulseless electrical activity percuntaneous endoscopic gastrostomy

ContriButorS
Editors
M.Rodrigues

Mark Rodrigues Zeshan Qureshi

Radiology Registrar, Edinburgh Royal Inrmary, Edinburgh, UK Academic Clinical Fellow, Great Ormond Street and Institute of Global Health, London, UK

Authors
J.Rodrigues B.Hedayati K.Varghese A.Cheng C.Gee

Jonathan Rodrigues Chest X-rays Abdominal X-rays Mark Rodrigues Introduction Chest X-rays Abdominal X-rays CT MRI Ultrasound Nuclear Medicine Scans Fluroscopy Bijan Hedayati Chest X-rays Chris Gee Orthopaedic X-rays Amanda Cheng Bonus X-rays Kabir Varghese Bonus X-rays

Radiology Registrar (Bristol Royal Inrmary, Bristol) Radiology Registrar (Edinburgh Royal Inrmary, Edinburgh)

Radiology Consultant (Lewisham Hospital, London) Orthopaedic Registrar, Trauma and Orthopaedics (Western Sussex Hospitals, Sussex) Radiology Registrar (Western General Hospital and Edinburgh Royal Inrmary, Edinburgh) Radiology Registrar (Chelsea and Westminster, London)

Reviewers
Brendan Kelly Chloe Thomson Marianna Christodoulou Madelaine Gimzewska Katherine Lattey Jessica Spiteri Paris University College Dublin University of Leicester University of Manchester University of Edinburgh Brighton and Sussex Medical School University of Malta

Z.Qureshi

ConTENTS
Introduction........................................................... 11
What are X-rays?........................................................................ 11 How are X-rays used to produce images?......................... 12 The main densities on X-ray. .................................................. 12 Magnication. ............................................................................. 12 The hazards of using X-rays................................................... 13 Relevant legislation................................................................... 13 Pregnancy and X-rays............................................................... 14 How to request radiology examinations........................... 14 When and how to discuss a patient with radiology...... 15

MRI Scans......................................................... 543


MRI Head....................................................................................544 ...................................................................................549 MRI Spine. MRCP............................................................................................553 MRI Small Bowel......................................................................554 MRI Knee & Other Joints.......................................................555

Ultrasound Scan.............................................. 557


Neck USS.....................................................................................558 Chest USS...................................................................................558 Abdominal USS.........................................................................560 Pelvic USS...................................................................................562 FAST Scanning...........................................................................563 Vascular USS..............................................................................563 Musculoskeletal USS..............................................................564 Ultrasound Guided Procedures...........................................564

Chest X-Rays* .....................................................17


Introduction................................................................................. 17 20 Clinical Cases. ........................................................................ 29

Abdominal X-Rays*......................................... 181


Introduction...............................................................................181 20 Clinical Cases. ......................................................................189

Nuclear Medicine Scans................................. 565


VQ scan. .......................................................................................566 Myocardial perfusion scan...................................................567 Genitourinary scan. .................................................................567 Bone imaging............................................................................568 PET/CT..........................................................................................569

Orthopaedic X-Rays*....................................... 335


Introduction...............................................................................335 Spine X-Ray Cases ...................................................................367 Shoulder X-Ray Cases ............................................................391 Elbow X-Ray Cases...................................................................399 Wrist X-Ray Cases ...................................................................407 Hip X-Ray Cases .......................................................................429 Knee X-Ray Cases ....................................................................473 Tibia/Fibular X-Ray Cases . ...................................................497 Ankle X-Ray Cases . .................................................................513

Fluoroscopy...................................................... 571
Contrast Swallow.....................................................................572 Barium Follow Through.........................................................576 Contrast Enema........................................................................576 Tubogram...................................................................................576

CT Scans............................................................ 521
CT Head.......................................................................................525 CT Cervical Spine......................................................................530 CT in Orthopaedics..................................................................530 CT Chest......................................................................................531 CT Abdomen and Pelvis.........................................................535

Bonus Cases*.................................................... 579


Bonus Chest X-Rays. ................................................................579 Advanced Chest X-Rays.........................................................603 Bonus Abdominal X-Rays......................................................613 Advanced Abdominal X-Rays...............................................625 Bonus Orthopaedic X-Rays...................................................637 Advanced Orthopaedic X-Rays............................................671

* For Chest X-Rays, Abdominal X-Rays, Orthopaedic X-Rays and the Bonus Cases, all cases have been labelled Case X to mimic real life clinical situations/assessment. For reference, the X-rays are also all listed by diagnosis and clinical signs on p695.

Cop

t of yrigh

Zesh

ure an Q

shi

CHEST X-RAYs
Cop t of yrigh Zesh

ure an Q

shi

Cop

t of yrigh

Zesh

ure an Q

shi

Case 1................. 29 Case 2................. 37


to yrigh ha Case 3................. f Zes45 Cop 4................. 53 Case

Case 6................. 69 Case hi 7................. 77 Case 8................. 85 Case 10............... 97


ures hi Cop 91 Case 9................. t of yrigh

Case 11.............105 ............121 esha 13. ZCase Case 12.............113 hi


res n Qu

Case 16.............145
u an Q h Case 18..f...........161 s e o Z right y p o C Case 19.............169

res n Qu

Case 17.............153

resh

Case 14.............129 Case 15.............137


ures hi

Case 5................. 61

Case 20............. 175


ures hi

nQ n Q framework for approaching chest nQ This introductions the chapter is aimed at providing a systematic ha e to esha esha Z Z Z f f f o o o X-rays. details and examples of the specic are covered more ht ght riFurther right X-ray ndings discussed below rig opychapter Copy Cthe Copy extensively in the example cases later in and in the bonus X-ray chapter. ures hi shi Qure n a h 1. Projection (AP/ PA) es t of Z

In this book we look only at frontal chest X-rays (PA and AP X-rays), as these account for almost all chest X-rays performed. The lateral chest X-ray is not commonly performed and has been largely replaced by CT.

Z h ht of g g i i r r y y Cop Cop can affect its The projection of a chest X-ray Systematic approach to appearance and interpretation. Therefore it is chest X-rays important to determine which projection has been used. shi hi shi 1. Projection ures Qure Qure Q n n n a a a h two possible projections for a frontal sh sh s The chest 2. Patient details of Ze of Ze of Ze t t t h h h g g g i i i X-ray are the anteroposterior r r (AP) and the yr Copy Copy 3. Technical adequacy Cop righ Copy
4. Obvious abnormalities Review 6. an Q areas sh e 7. Summary 5. Systematich review of the X-ray i

t of Z

KEY POINT

nQ esha

nQ esha

ures

hi

rig Copy

ht of

ures

ure provides the best assessment of ure The PA QX-ray an an Q h h s s e e Zbe able f Z the thorax but requires the patient t ofto ight o igh r r y y p p to stand (or sit on a stool). This is the standard Co Co projection, so if there is no annotation stating otherwise, you can assume the X-ray is PA.

posteroanterior (PA). Somwhere on the X-ray you should see something that indicates whether it is AP or PA.

shi

shi

Cop

t of yrigh

Zesh

ure an Q

shi

ight opyr

of Ze

Qu shan

resh

i Figure 1: These shitwo chest X-rays from the same patient. reare resh u u Q Q A ZImage han han A Is a well inspired PA chest X-ray. f es f Zes o o t t h h ig inspired AP X-ray. Image B Is a less well (but still adequately) rig opyr Copy Notice the dramatic effect ofC the projection and degree of inspiration on the apparent heart size. Also note the amount of the scapulae which is projected over the lungs in each projection. hi hi r Copy ight o f B an Zesh Qure s r Copy resh i righ Copy i righ Copy t of Z e t of Z e ight o f an Zesh Qure s

righ Copy

t of Z

Qu shan

resh

i righ Copy t of Z e

Qu shan

Qu shan

resh

righ Copy

t of Z

Qu shan

resh

i righ Copy t of Z e

Qu shan

resh

Qu shan

resh

Chest X-Rays - Introduction 17

Chest X-Rays

i i i resh resh resh u u u Q Q Q an an anuseful for are AP X-rays provide a less comprehensive Zesh Zesh The age and gender of thetpatient Zesh f f f o o o t t h h h helping to formulatey your the of g rig assessment than PA X-rays due to rieffects rig differential diagnosis. Copy Copy Cop sh which one is the standard Check sh sh thorax If you cannot remember Qure Qure that the X-ray includes all of the Qure n n n a a a h h h es esof the ribcage, (both lung apices, the lateral sides Zes view, Z remember, AP is crAP, so PA is standard. t of Z ht of ht of h g g g i i i r r r and both costophrenic angles). Important y y p Cop If you are asked to justify why an Co Copy X-ray is PA, pathology can be missed if the entire thorax is remember that in PA X-rays, the patients arms not imaged. are positioned in such a way that the scapulae unlikely It is that you will be given an X-ray r in the are pulled almost r fully hiout of the lung elds. In hi hi u es ures u es Q Q Q n n n exam that does not show the entire lungs, but AP X-rays, a positioning is not possible, and the hthis ha ha f Zesare projected further over the lungs. f Zes f Zes in practice. o o o t t t some parts are occasionally missed scapulae h h h rig rig rig Copy
magnication and the position of the scapulae (gure 1). They are usually only performed for haemodynamically compromised patients.
Chest X-Rays

3. Technical Quality
i i

Copy

2. Patient Details
Z Z ht of ht of g g i i r r y y the X-ray (unless op Cop The patients details will be on C
anonymised for the exam). Say the name, age/date of birth, and when the X-ray was taken.ureshi

It is important to assess RIP Rotation, Inspiration, Penetration.

Copy

i hi It is important to resh you are looking at the uensure ures Q Q Rotation n n a a correcte X-ray sh from the correct patient. esh

rig Copy

ht of

nQ esha

KEY POINT

There is a saying in radiology that the most important X-ray Inspiration is the s previous one. It is always i i h hi eshAP ur ures PA and X-rays are taken in held deep inspiration. Qure to compare the current Q Q helpful n n n a a a esh X-ray with previous X-rays and esh esh Count the ribs to assess inspiratory t of Z t of Z t of Z effort. h h h g g g i i i r r r opy has been opy imaging to see if C there Copy Cdown You should count to the lowest rib crossing any change in the ndings. through the diaphragm. Six anterior ribs or 10

hi shi ures Q Qure n n a a Patient rotation can erroneously give the sh esh of Ze of Zor t t impression of mediastinal shift lung pathology h h g g i i r r Copy Copy (gure 2).

righ(medial ends) should The heads of the clavicles Copy be equidistant from the spinous processes of the vertebral bodies. If they are not, the patient is rotated.

t of Z

nQ esha

ures

hi

Cop

t of yrigh

Zesh

an Q

hi ures Cop resh i C resh i

posterior ribs indicate adequate inspiratory effort.

esh fZ o A t h yrig

ure an Q

shi

Cop resh i C resh i

t of yrigh

Zesh

ure an Q

shi

ight opyr

of Ze

Qu shan

ight opyr

of Ze

Qu shan

ight opyr

of Ze

Qu shan

resh

righ Copy

t of Z

Qu shan

righ Copy

t of Z

Qu shan

righ Copy

t of Z

Qu shan

resh

Copy

Figure 2: These are two X-rays from the same patient. Image A: The patient is well centred. i i i h sh Image B: The in apparent left lower zone consolidation. uresh res is markedly rotated, which has resulted upatient ure Q Q Q n n n ha ha ha X-ray ThisZ appearance is, however, probably caused by the abnormally positioned cardiac shadow, and a repeat f es f Zes f Zes o o o t t t with the patient well-centred should be obtained. h h h g g g ri ri ri

Copy

Copy

18 Chest X-Rays - Introduction

and hold a deep breath (due to pain, breathing Is it focal or diffuse, rounded or spiculated, well problems, or confusion). Underinspired X-rays or poorly demarcated? i markings at h hi hi can cause crowding of the lung s e ur ures ures Q Q Q n n n the bases, incorrectly Density ha ha sha giving the impression of f Zeor f Zes f Zes o o o t t t consolidation other pathology. Additionally, h h h ig ig pyrig opyr 1). opyr Describe the density of anC abnormality in relation to Cothe C heart may appear falsely enlarged (gure the normal surrounding tissue, e.g. if the abnormality More ribs, particularly with attened is in the lung, compare it to the normal lung; if in the bone, compare it with the other bones. diaphragms, indicate hyperination due to hi shi shi e e r r u u ures airway obstruction, such as chronic obstructive Q Q Q n n n a ha Ife the abnormality is denser (i.e. whiter) sha Zeshthe airway disease (COPD). f Zes fZ f than o o o t t t h h h normal tissue, you can say that there ig is increased rig rig opyr Copy Copy C opacication or density; if less-dense (i.e. blacker), Penetration say there is increased lucency or reduced density. The X-ray is adequately penetrated if you can hi just see the vertebral bodies ures behind the heart. Q

Z Z Z has a You should assess whether the abnormality Underpenetrated means that you cannot see ht of ht of ht of g g g i i i r r r y y y p p uniform or heterogenous C appearance. op the heart and overpenetrated means Cobehind Co that you will be able to see the vertebral bodies Other features very clearly.

n esha

Texture

nQ esha

ures

hi

nQ esha

ures

hi

If there isranything else in the abnormality, such reshi hi hi Over and under penetration ures can obscure or u es Q Q Qu n n as air bronchograms or uid levels, thensmention a a obliterate signicant ndings, particularly in han sh sh e e e Z Z Z f f f to to these as well. the ight o yrlungs. yrigh yrigh This is less of a problem with the advent of Are there other abnormalities, such as volume digital viewers which allow the X-ray windows change, bony abnormalities, or surgical clips? to be manipulated. However, this function can hi hi hi only manipulate the Q image uresso far, so adequate ures ures Q Q n n n a a a 5. Systematic Review of the X-rayZ (Figure h important. h penetration is still es es esh 3) t of Z t of Z t of h h h g g g i i i r r r Initially assess from a distance Copy Copy Copy to see differences in lung shadowing/obvious masses. Previously, when using hard-copy X-rays, you would be Rotated, under-inspired or taught to look i i at the X-ray initially from four i under/overpenetrated X-rays resh resh now most X-rays are viewed onQuresh u u feet; however, Q Q an han han can hinder accurate assessment. sh computer so make sure you zoom out as much f Zes f Zes f Ze o o o t t t h h h These technical factors must rig rig yrig as possible for your initial inspection. Copy Copy Cop be taken into account when After that, reassess from close-up to look for assessing the X-ray. subtle abnormalities.

Cop

Cop

Cop

KEY POINT

urematter what system you use for n Qure It does Qnot n a a h 4. Obvious Abnormalities esh Ze assessing the X-ray, as long as you do not miss f f Zes fZ o o o t t t h h h ig ig rig any areas. opyr opyr Cyou Copy If can see obvious abnormalities, sayC so and A useful system is ABCDD (Airway, Breathing, describe them: Circulation, Diaphragm/ Delicates). Which lung is involved? i h hi hi ures ures on manmade abnormalities, n Qures Also comment Q Q n n a ha sha Which part o of tube. Zeslung? Zesh f the of Ze e.g. lines, pacemakers, a nasogastric of (NG) Qu shan CoIf possible, say which lobe is/lobes are Co Co A Airway involved. Remember it is not always possible to determine this on an X-ray in which case Is the trachea central? i to describe use upper, middle, or lower zone h hi s shi e r re u u ures Q Q Q If not, is it deviated due to patient rotation or n n n the abnormalitys location. CT can locate ha ha sha f Zemore f Zes f Zes o o o pathology? t t t abnormalities accurately. h h h g g g ri ri ri Copy Copy Copy
Chest X-Rays - Introduction 19

resh

shi

shi

pyrig

ht

pyrig

ht

pyrig

ht

Chest X-Rays X Rays

i i resh resh u u Q Q n an Fewer ribs indicate Size esha Zesh an underinspired X-ray. This t of Z f o t h h may be due to the timing of the X-ray, or, more yrig pyrig Shape Cofrequently, Cop because the patient is unable to take

Cop

t of yrigh

Zesh

ure an Q

shi

i i i resh resh resh u u u Q Q Q n an andue to may If theZ cause eshais pathological, is the trachea being eshbe Zesh Widening of the mediastinum Z f f f o o o t t t h h h technical factors (e.g. AP as iglobar or rig pulled to one side (volume loss, such r rig projection), vascular Copy lung collapse) or pushed away (increased Copy Copy of the thoracic aorta or structures (e.g. unfolding volume
aortic dissection), masses (mediastinal tumours or lymph node enlargement) or haemorrhage i (e.g. aorta). The clinical ndings in shi eshi resh reruptured rsuch u u u Q Q Q n n n B Breathing can ha ha cases are important, as the cause ha be difcult Zes f Zes f Zes f be o o o t t t h h h to determine on X-ray. CT can used if required ig rig yrthe to pyrig o Copy Start in the apices and work down Cop C for further assessment. costophrenic angles, comparing both lungs to such as a large pleural effusion or mediastinal mass)? The right paratracheal stripe can be useful to assess, i if visible. It is composed of the soft Ensure that you inspect shi the entire lung, including sh shi e e r r u u ure tissue between the medial wall of the right lung Q Q Q n n n the apices, hila, and costophrenic angles. ha ha ha and the right wall of the trachea. It is visible in f Zes f Zes f Zes o o o t t t h h h ig rig rig lower 50-60% of X-rays measure <5mm in opyrthan the opyshould Copy The left hilum should never be C Cand right. If this is the case, you must look for volume diameter. If it is thickened, it is commonly due to loss either pulling the right hilum up or pulling lymph node enlargement. the left hilum down. i h hi The window is another area aortopulmonary shi ures ure ures Q Q Q n n n a a a Both hila should be the same density and have esh to assess for lymph node enlargement. h h The f Zes or convex margins. fZ f Zes between o o o t t t h h h no lumps aortopulmonary window is located g g g ri ri yri Copy Copy Cop the aortic arch and the left pulmonary artery. Look around the edge of the lungs, assessing Normally there should be no soft tissue visible for pneumothoraces. These can be particularly in this region, thus giving the impression of subtle at the lung apex. i i h hi au window. s resh If this is not the case, you must ures Qure Q Q n n n a a a consider lymph node enlargement. esh esh esh look for differences.

Chest X-Rays

Copy

Both hemidiaphragms should be visible and is not the case, you must rst check that the upwardly convex. Flattening of a hemidiaphragm patient is not rotated. Then you must assess suggests for volume change in the i lungs (either volume h hi raised intrathoracic pressure either hi ures towards the abnormal ures ures Q Q Q from lung hyperexpansion, as seen in air loss pulling structures n n n ha sha esha Ze for f Zes f Zpneumothoraces. o o o trapping with COPD, or tension side increased volume pushing them away), t t t h h h rig rig pyrig Copy accounting for the position of the Comediastinum Copy The right hemidiaphragm is normally slightly and heart. Marginal mediastinal shift can be higher than the left due to the mass effect of observed if the margins of the thoracic vertebral the adjacent liver. If this is not the case, you must bodies can be clearly seen i beyond the cardiac h hi hi s e consider r u ures whether one of the hemidiaphragms ures is Q Q Q and mediastinal contours on a well-centred n n n ha ha is being abnormally pulled up or sha down. pushed f Zes f Zes f Ze o o o X-ray. t t t h h h g g g i i i

n n n esha esha esha Z Z Z f f f o o o The t mediastinum and heart should be igh D Diaphragm opyright r right Copy positioned over the thoracic vertebra. Copy If this C

Assess the heart size. Cardiomegaly is dened by presence of gas within it (pneumomediastinum). the maximal transverse cardiac diameter being This appears as linear lucencies projected over greater than 50% of the maximal transverse the mediastinum. These often extend into hi hi internal thoracic diameter shi (cardiothoracic ratio). s e e r r u u ures the neck and may be associated with surgical Qaccurately assessed on a wellQ Q n n n a a a This can only be esh esh esh emphysema (gure 4). t of Z PA X-ray due the effects of magnication t of Z t of Z h h h inspired g g g i i i r r r y opy Copy on AP and underinspired X-rays (see Copgure Cremember It is important to that the lung 1). continues behind the heart (a large portion of However, it is still important to assess cardiac size the left lower lobe is behind the heart). The on an AP X-ray if its normal on the AP, then it i i cardiac hi will be normal on the PA; conversely, if it is grossly resh reshshadow should be of uniform density. resIf u u u Q Q Q n PA, it is likely to be enlarged on shan this is not the case, you must consider n enlarged hathe eson esha whether f ZX-ray. f Ze f Zconsolidation, o o o t t t retrocardiac pathology, such as h h h the PA rig rig pyrig is present. This can be omass, Copy Copy C lobar collapse, or a The cardiac and mediastinal borders should be difcult to assess due to the overlying cardiac clearly visible. If this is not the case, you must shadow. Inverting the image often makes any consider whether there is pathology in the i abnormality more obvious (gure 5). shi shi adjacent lung. Quresh Qure Qure

rig Copy

C of Z and mediastinum htCardiac

rig Copy

ht of

Z ht of g i r y p You should assess the mediastinum for the Co

Copy

Copy

20 Chest X-Rays - Introduction

i i i resh resh resh u u u Q Q Q anlungs extend behind an an Remember that the D Delicates sh e Zesh Z Zesh f f f o o o t t t h h h therdiaphragms, so you need to look for lung ig rig yrig op Assess the bones. LookC at the ribs for fractures Copy Copy
or bone destruction. Assess the rib spaces, which should be roughly equal. Narrowing can be seen with volume loss i in the underlying lung. Review eshi hi sh res rethe u u Look for free air under the diaphragm. This can Q Q Qur the rest of imaged skeleton for fractures n n a a anor h h h s s s e e e Z be difcult, f Z destructive bone lesions. of Z ht of as the gastric bubble and bowel ight o rigcan r6). right y y y p p p loops have a similar appearance (gure o o o C C C Look at the soft tissues for evidence of surgical The costophrenic angles should be sharp. If not, emphysema (gas [black areas] in the soft tissues) there is likely to be pleural uid present. and previous surgery (surgical clips, mastectomy). pathology through the hemidiaphragms. Again, inverting the image can make such pathology more obvious.
Chest X-Rays X Rays

righ Copy

t of Z

Qu shan

resh

righ Copy hi righ Copy hi rig Copy hi

t of Z

Qu shan

resh

righ Copy hi righ Copy hi rig Copy hi

t of Z

Qu shan

resh

righ Copy

t of Z

nQ esha

ures

t of Z

nQ esha

ures

t of Z

nQ esha

ures

hi

rig Copy

ht of

nQ esha

ures

ht of

nQ esha

ures

ht of

nQ esha

ures

hi

rig Copy

Z of Z Figure ht of3: A normal PA chest X-ray demonstrating ight the r y p normal anatomy. Co ure an Q shi Cop resh i C resh i righ Copy t of Z ight opyr t of yrigh

nQ esha

ures

nQ esha

p Co A

t of yrigh

Zesh

Zesh

an

f Z lucencies Figure 4: PA chest X-ray showing linear ight o r y p o mediastinum. Their projected over the upper C location and appearances are consistent with a pneumomediastinum. There may also be evidence of gas within the soft tissues (surgical emphysema) i hi spericardium or (pneumopericardium). Quresh Qure Cop resh i C resh i righ Copy t of Z e ight opyr of Ze t of yrigh Zesh an

ures

nQ esha

ures

hi

B
Qu shan

ight opyr

of Ze

Qu shan

of Ze

Qu shan

resh

righ Copy

t of Z

Qu shan

Qu shan

Qu shan

resh

Copy

Figure 5: Image A: This X-ray looks of the review areas reveals a reshi hi shi normal on initial viewing; however, closer resinspection urewhich uof u Q Q n n very subtle retrocardiac mass, will be located in the medial aspect the lower lobe. This abnormalitys is an Q ha ha hmuch s s e e e Z Z Z f f easier to see if the t of is inverted (Image B). ght o ghX-ray ght o

ri

Copy

ri

Copy

ri

Chest X-Rays - Introduction 21

Copy
Chest X-Rays

Z ht of rigA

esh

ure an Q

shi Copy

B right of Z

esh

ure an Q

shi

Cop

t of yrigh

Zesh

ure an Q

shi

righ Copy

t of Z

Qu shan

resh

i righ Copy t of Z e

Qu shan

resh

i C ight opyr of Ze

Qu shan

resh

Figure 6: These three X-rays show how difcult it can be to diagnose free subdiaphragmatic gas. Image A: Is a normal chest X-ray with gas in the stomach. We know the gas is within the stomach, as it is under the left hemidiaphragm, and the soft tissue rim overlying the gas is a few millimetres thick, as it consists of the stomach wall and adjacent diaphragm. hi i s eshi resh u ursoft Q Qurea large pneumoperionteum. In this case, Q Image B: Contrast that appearance to the centre X-ray, which shows the n n n a a a h h sh f Zes tissue rim between the lung and abdomen f Zeis f Zes very thin, as it solely represents the diaphragm. o o o t t t h h h g g g ri ri pyri the right hemidiaphragm below Copy Image C: Is a mimic of free subdiaphragmatic Copy gas. In this case, inspection of the area Co reveals bowel markings. These appearances are due to interposition of a loop of bowel between the liver and right hemidiaphragm, and is known as Chilaiditis sign.

righ Copy

t of Z

Qu shan

resh

i righ Copy t of Z e

Qu shan

resh

i righ Copy t of Z e

Qu shan

resh

An endotracheal (ET) tube should have its tip Problems to look out for include misplacement proximal to the carina. Problems can arise if it into the lungs, and the tip being within the distal is inserted too far and the tip enters one of the oesophagus. i hi shi in collapse of the none bronchi. This will result r resh u ures lines are most commonly inserted uinto Q Q Q Central n n n a a a ventilated eshlobes. esh esh the internal jugular veins. Their t of Z t of Z t of Ztips should be in h h h g g g i i i r r r py lie well mid or lower superior Copy The tip of a nasogastric (NG) tube Coshould Copy vena cava. Complications below the left hemidiaphragm in the stomach. include misplacement and a pneumothorax.

rig Copy

f Z Figure 7) Lines ht o(see

nQ esha

ures

hi rig Copy ht of Z

nQ esha

ures

hi rig Copy ht of Z

nQ esha

ures

hi

Cop

t of yrigh

sh NG tube Ze

ure an Q

shi Cop t of yrigh

Zesh

ure an Q

shi Cop t of yrigh

Zesh

ure an Q

shi

ight opyr

of Ze

Qu shan

resh

i C ight opyr of Ze

Qu shan

resh

i ight opyr of Ze

Qu shan

resh

Copy

f Ze ight o

Qu shan

resh

i Copy r f Ze ight o

shan

Copy

f Ze ight o

Qu shan

resh

i Copy r f Ze ight o

shan

Figure 7: C An AP chest X-ray demonstrating satisfactorily positioned endotracheal tube, right and left internal jugular central lines, and nasogastric tube. Note that the right i shiinternal jugular line descends straightu e r resh u Q Q down n a h es the right side of the mediastinum (as it t of Z h g i travels through the right brachiocephalic r y Cop the left sided central line vein), whereas passes diagonally across the mediastinum (as it travels through the left brahiocephalic i h hi s vein). The tips of both of the lines should ures Qure be projected over the mid or lower Q superior n a Zesh vena cava. ht of

Copy

rig

22 Chest X-Rays - Introduction

Cop

t of yrigh

Zesh

ure an Q

shi Cop t of yrigh

Zesh

ure an Q

shi Cop t of yrigh

Zesh

ure an Q

shi

righ Copy

t of Z

Qu shan

resh

i righ Copy t of Z e

Qu shan

resh

i C ight opyr of Ze

Qu shan

resh

righ Copy

t of Z

Qu shan

resh

i righ Copy t of Z e

Qu shan

resh

i righ Copy t of Z e

Qu shan

resh

righ Copy

t of Z

nQ esha

ures

hi righ Copy t of Z

nQ esha

ures

hi

6. Review Areas

rig Copy

ht of

nQ esha

ures

hi rig Copy ht of Z

nQ esha

ures

hi

Figure 8: The common chest X-ray review areas. ureshi Remember toQ han s e Z of right add to this list y p o any sites where C you commonly overlook pathology. i

KEY POINT

rig Copy

ht of

nQ esha

ures

Double-check the following areas, since pathology i Remember you are looking hi shi on initial viewing resh at a chest X-ray, not a lung X-ray. is easily overlooked atn these sites ure u ures Q Q Q n n a a a sh esh esh (see gure 8): Ze ofof the Ensure you assess t of Z t of Z tall h h h g g g i i i r r r py X-ray, including CoApices Copy Copy the soft tissues, bones such as the clavicles, Hila scapulae and visible humeri, Behind the heart i i i resh resh and the upper abdomen. resh u u u Q Q Q an an Costophrenic angles shan Ze Zesh Zesh

pyrig the diaphragm CoUnder

ht of

rig Copy

ht of

rig Copy

ht of

7. Summary

a Summarise Z your and give a differential hndings e e Pneumonia f es o t t of Z t of Z h h h list. Think about the history and clinical g g g i i i r r r y py Coexamination Cop Copy usually unilateral. as well as the X-ray ndings when Dense or patchy consolidation, making your differential diagnosis. May contain air bronchograms (air containing Say whether you would like to review previous bronchiolessrunning through consolidated lung). shi hi eshi help. urwould ure imaging if you think this Q Q Qure n n n a a a h h h s s es In the lower zones, pneumonia mayf be of Ze investigations, including ight of Z o Zedifcult Suggest ht further htboth g g i i r r r to distinguish from effusions, so should be y y y p Coimaging, Cop Cop which may be useful. on your differential list (remember there is often a parapneumonic effusion). Suggest a management plan for the patient. t of Z e Qu shan resh i righ Copy t of Z e Qu shan resh i righ Copy t of Z e Qu shan resh i

res n Qu

hi

SpEcific fiNdiNGs oN cHEst X-raY i


Qu shan resh

Qu shan

resh

righ Copy

Chest X-Rays - Introduction 23

Chest X-Rays X Rays

i i i resh resh resh u u u Q Q Q n these an sign is useful for locating in which an usually sharp border between thee The silhouette shaand Zesh Zesh Z lung f f f o o o t t t h h h structures is thus lost y if there ig is consolidation in rig lobe of the lung the pathology is located yrig (gure op r Copy 9). Normally, there is a sharp border Copbetween C the lobe abutting these soft tissues.
the aerated lung and the soft tissues of the It is necessary to know which lobes contact the heart and diaphragm. This is due to the large heart and diaphragmatic borders in order to be hi differences in the number shi of X-rays attenuated shi e e r r u u ures Q able to use the silhouette sign: Q Q n n n by the soft tissues (a relatively high proportion of ha ha sha f Ze f Zes f Zes o o o t t t h h h o X-rays) and the lung (relatively few X-rays). Diaphragms: left y and rig right lower lobes yrig yrig If there is consolidation, the normally aerated lung is replaced by uid or pus. This attenuates X-rays to a similar extent to the h heart and diaphragms. The i

Chest X-Rays

Cop

Cop

o o

Right heart border: right middle lobe

Cop

Copy

f Ze ight o

shan

s Qure

A
nQ esha ures hi

Copy

f Ze ight o

shan

Left heart border: lingula (part of the left hi eshi lobe) urupper ures

B
nQ esha ures hi

Copy

f Ze ight o

shan

righ Copy

t of Z

righ Copy hi rig Copy hi rig Copy shi Cop resh i C

t of Z

righ Copy hi rig Copy hi rig Copy shi Cop resh i C

t of Z

nQ esha

ures

hi

rig Copy

ht of

nQ esha

ures

ht of

nQ esha

ures

ht of

nQ esha

ures

hi

fZ ight o r y p C Co

nQ esha

ures

ht of

nQ esha

ures

D
ure an Q

ht of

nQ esha

ures

hi

Cop

t of yrigh

Zesh

ure an Q

t of yrigh

Zesh

t of yrigh

Zesh

ure an Q

shi

ight opyr

of Ze

Qu shan

ight opyr

of Ze

Qu shan

ight opyr

of Ze

Qu shan

resh

hi hi hi ures ures ures Q Q Q n n n Figure 9: X-rays demonstrating consolidation and the silhouette sign. sha esha esha fZ f Zhemidiaphragm f Zeindicating o o o t t t Image A : Shows loss of the medial aspect of the right but a clear right heart border, right h h h rig rig rig Copy Copy lower lobe consolidation. Copy
Image B: Shows an indistinct right heart border with preservation of the right hemidiaphragm, in keeping with right middle lobe consolidation. i border but loss of the left hemidiaphragm, Image C: Shows a clear left heart h hi consistent with left lower lobe consolidation. hi ures ures ures Q Q Q n n n a bottom right X-ray shows loss of the left a border but a clear left hemidiaphragm, Image D: heart a hThe h hindicating f Zes f Zes f Zes o o o consolidation within the lingula. t t t h h h g g g i i i

Copy

Copy

Copy

24 Chest X-Rays - Introduction

i i i resh resh resh u u u Q Q Q You can Z use the X-rays hanchecklist below to assess the chest han shown in this chapter. han f es f Zes f Zes o o o t t t h h h rig rig rig Copy Copy Copy
Technical Aspects
Chest X-Rays

Check patient details (name, date of birth, hospital number)

Cop

to yrigh

sha of the X-ray Checkfthe Zedate

res n Qu

hi

Identify the projection of the X-rayC

op

t of yrigh

Zesh

res n Qu

hi Cop t of yrigh

Zesh

res n Qu

4 4 4 4

hi

Assess technical quality of X-ray (rotation, inspiration, penetration)

Ze Ze ht of ht of g g i i r r y y Cop Describe any obvious abnormality Cop


Site (lung and zone/lobe) Size (if relevant) Quresh an

Obvious Abnormalities shan

shi Qure

shan

shi Qure righ Copy ures hi righ Copy hi rig Copy shi C

t of Z

Qu shan

resh

4 4
res u 4 hi

i righ Copy t of Z

Co

h f Zes o t h g pyriShape (if relevant)


Density

nQ esha

t of Z

nQ esha

4 4
hi

Systematic Reviewrof hi X-ray esthe

rig Copy

Z Position ht of of trachea

nQ esha

Assessment of lungs Size and appearance of hila

rig Copy shi

ht of

nQ esha

ures

ht of

nQ esha

ures

4 4 4shi 4 4 4

ight opyr

sh Assess Zecardiomegaly f for o

ure an Q

C and cardiophrenic angles Assess cardiac and mediastinal borders


Position and appearance of hemidiaphragms

ight opyr

of Ze

sh

ure an Q

ight opyr

of Ze

sh

ure an Q

Cop

t of yrigh

Q Evidence of h pneumoperitoneum (free air under the diaphragm) an Q han Zes


Assess the imaged skeleton

hi ures

Cop

t of yrigh

Zes

hi ures Cop resh i

t of yrigh

Zesh

ure an Q 4

shi

4 4
i resh u 4 Q n sha

Assess the imaged soft tissues (e.g. surgical emphysema, mastectomy)

e e f Ze to t of Zcostophrenic angles, under the diaphragm) t of Z h h h g g g Look at review areas (apices, hila, behind the heart, i i i r r r Copy Copy Copy
Summary
Present ndings

sh Comment on iatrogenic Qure abnormalities shan

Qu shan

righ Copy

e Ze Review relevant previous imaging if appropriate t of Z ht of


Provide a differential diagnosis where appropriate

Qu shan

resh

i rig Copy

Qu shan

resh

i righ Copy t of Z e

Qu shan

i 4 resh

4 4 4

Suggest appropriate further imaging/investigations if relevant

Co

ht o pyrig

f Zes

ure an Q

shi

Co

ht o pyrig

f Zes

ure an Q

shi Co ht o pyrig

f Zes

ure an Q

shi

28 Chest X-Rays - Introduction

PRESENTshi ure an Q YOUR h s e fZ ight o FINDINGS... r y p Co


t of Z e Qu shan resh i

i i resh resh u u Q Q An 18 year old presents pain and an hanwith sudden onset right sided chestZ esh f Zes As part of his work up he undergoes f chest o o shortness of breath. a X-ray. t t h h rig rig Copy Copy resh i C resh i righ Copy hi righ Copy hi rig Copy hi rig Copy shi Cop resh i C resh i righ Copy i righ Copy t of Z e t of Z e ight opyr of Ze t of yrigh ht of Z ht of Z t of Z t of Z e ight opyr of Ze resh i
Chest X-Rays

CASE 1

righ Copy

righ Copy i righ Copy hi righ Copy hi rig Copy hi rig Copy shi Cop

t of Z

Qu shan

Qu shan

righ Copy

t of Z

Qu shan

resh

t of Z

Qu shan

Qu shan

resh

righ Copy

t of Z

nQ esha

ures

t of Z

nQ esha

ures

nQ esha

ures

hi

rig Copy

ht of

nQ esha

ures

ht of

nQ esha

ures

nQ esha

ures

hi

rig Copy

ht of

nQ esha

ures

ht of

nQ esha

ures

nQ esha

ures

hi

Cop

t of yrigh

Zesh

ure an Q

t of yrigh

Zesh

ure an Q

Zesh

ure an Q

shi

ight opyr

of Ze

Qu shan

resh

i C ight opyr of Ze

Qu shan

Qu shan

resh

righ Copy

t of Z

Qu shan

resh

i righ Copy t of Z e

Qu shan

Qu shan

resh

righ Copy

t of Z

Qu shan

resh

i righ Copy t of Z e

Qu shan

resh

Qu shan

resh

Chest X Rays - Case 1 29

Cop
Chest X-Rays

t of yrigh

op AnnotAted C X-rAY Qu shan resh i righ Copy i righ Copy hi righ Copy hi rig Copy hi rig Copy shi Cop resh i ight opyr t of yrigh ht of ht of

Zesh

ure an Q

shi t of yrigh

Zesh

ure an Q

shi Cop t of yrigh

Zesh

ure an Q

shi

righ Copy

t of Z

t of Z

Qu shan

resh

i C ight opyr of Ze

Qu shan

resh

righ Copy

t of Z

Qu shan

resh

t of Z

Qu shan

resh

i righ Copy t of Z e

Qu shan

resh

righ Copy

t of Z

nQ esha

ures

t of Z

nQ esha

ures

hi righ Copy t of Z

nQ esha

ures

hi

rig Copy

ht of

nQ esha

ures

nQ esha

ures

hi rig Copy ht of Z

nQ esha

ures

hi

rig Copy

ht of

nQ esha

ures

nQ esha

ures

hi rig Copy ht of Z

nQ esha

ures

hi

Cop

t of yrigh

Zesh

ure an Q

Zesh

ure an Q

shi Cop t of yrigh

Zesh

ure an Q

shi

C C pneumothorax. IN SUMMARY This chest X-ray shows a large right There is no evidence of associated tension. hi shi this X-ray, suggesting ureshi There is no underlying cause discernible ures ureon Q Q n n nQ esha esha esha Z Z Z f f f o o o ht spontaneous pneumothorax. that this is a o primary ight ight pyrig opyr opyr C C resh i righ Copy t of Z e resh i righ Copy t of Z e resh i

ight opyr

of Ze

Qu shan

of Ze

Qu shan

resh

i ight opyr of Ze

Qu shan

resh

righ Copy

t of Z

Qu shan

Qu shan

Qu shan

30 Chest X Rays - Case 1

Cop

t of yrigh

Zesh

ure an Q

shi

h f Zes o QUESTIONS t h rig Copy

ure an Q

shi Cop t of yrigh

Zesh

ure an Q

shi

PRESENT hi ures Q n YOUR ha f Zes o t h rig Copy FINDINGS...


This is a PA chest X-ray of an adult.

res There are no identifying n Qumarkings a h s like to ensure that this is I would Ze ht of g i r y p correct patient, and to check Cothe when the X-ray was taken.
The patient is slightly rotated; this is otherwise a technically shi Qure n adequate X-ray with adequate a esh fZ o t penetration and good inspiratory h g pyri No important areas are cut Coeffort. off at the edges of the lm. There is an obvious abnormality shi eline in the right hemithorax: a Qur n a sh with absence can clearlyfbe seen o Ze t h g i r markings beyond it, in of lung Copy keeping with a lung edge. The aerated right lung is otherwise normal in appearance. shi

hi

hi A) Male gender ures Q n ha B) Smoking f Zes o t h rig Copy C) COPD D) Trauma E) Marfans syndrome eshi

1. Which of the following are risk factors for a primary spontaneous pneumothorax?
ight opyr of Ze Qu shan

resh

p Cop A) Central trachea. Dull percussion and Co reduced air entry on the right
side of the chest B) Central trachea. Dull percussion with bronchial breathing and shiof the chest shi crackles on the Q right side ure Qure n n a a h sh trachea. Hyperresonant percussion and reduced C) Central f Ze f Zes air entry on o o t t h h g g ri ri Copy the right side of the chest Copy D) Central trachea. Hyperresonant percussion and reduced air entry on the left side of the chest i Hyperresonant percussion and reduced E) Trachea deviated torthe left. h shi u es Q Qure n n a a air entry on the right side of the chest. Hypotensive, tachycardic esh esh

r Qur 2. Which of the clinical ndings would be n Qu hafollowing han s s e e Z Z f of a large simple right sided ht of ight o yrsupportive yrigpneumothorax?

eshi

fZ fZ ight o ight o r r y y p p differential diagnoses Co 3. Which of the following are appropriate Co

for a patient who presents with sudden breathlessness?


A) Pulmonary embolus i B) Pneumothorax Quresh n a esh C) hPneumonia t of Z g i r Copy D) Heart failure E) Anaphylaxis

ure an Q h The tracheaZ and mediastinum s e of are not and the right ightdeviated, r y p Co

hemidiaphragm is not attened.

rig Copy

ht of

nQ esha

ures

hi

Reviewing the rest of the lm, the left lung is normal. i

sh Qureheart n a The heart is not enlarged, h es of Z tare borders clear, and there is no h g i r py Coabnormality visible behind the
heart. There is minor blunting of the hi costophrenic angles which may ures Q n ha represent small f Zesvolumes of o t h pyrig effusion. Copleural The hemidiaphragms are clear. There is no free air under the shi diaphragm. Qure

A) Erect PA chest X-ray B) PA and lateral chest X-rays C) Expiratory chest X-ray hi ures Q n D) Supine chest ha X-ray f Zes o t h ig opyrE) CT

i the most appropriate initial 4. Which of the following is i resh resh u u Q Q imaging investigation in a patient suspected Z of having han han a f Zes f es o o t t h h ig ig opyrsimple pneumothorax? opyr C resh i

ight opyr

of Ze

Qu shan

f soft tissue There h are ono rig t y p o abnormalities or fractures; in C particular, no rib fractures are visible. righ Copy t of Z e Qu shan resh

an Zesh

Copy

e A) hConservative management and discharge the of Ze t of Z htpatient g g i i r r y y Cop B) Conservative management with outpatient Cop follow up C) Admit for conservative management, high ow oxygen, and monitoring i hi shpneumothorax D) Aspirate as much of the as possible re u ures Q Q n n esha esha E) Chest drain insertion t of Z t of Z righ Copy righ

5. Which of the following is the most appropriate management option for a previously healthy patient i i resh resh with a small, asymptomatic primary pneumothorax? n Qu n Qu
sha sha

Chest X Rays - Case 1 31

Chest X-Rays X Rays

h h f Zes f Zes o o AnsWers to Questions t t h h rig rig Copy Copy


Chest X-Rays

ure an Q

shi

ure an Q

shi Cop t of yrigh

Zesh

ure an Q

shi

1. Which of the following are risk factors for a primary spontaneous pneumothorax?
t of Z e The correct answers are A) Male e Ze gender and B) Smoking. t of Z ht of h g g i i r r y Cop Copy ures ures Qu shan resh i Qu shan resh i Qu shan resh i

righ Copy

A pneumothorax is the presence of air or gas within the pleural space. The gas separates the visceral and parietal pleura, and can lung. as: i Pneumothoraces can be considered h i hilead to the compression of the adjacent h

f Ze f Ze ht o ht o g g i i Secondary spontaneous (occur in the setting of lung disease) r r y y Cop Cop


Traumatic (either blunt or penetrating)

Q Q Primary shan spontaneous (no cause is identied) shan

r Copy

ight o

han f Zes

Qure

precise cause of these pneumothoraces is uncertain. There is some evidence that they are due to the rupture of small sub-pleural blebs (small, air-lled eshi pleura). Patients are r u cysts just under the visceral Q an han typically tall, slim young men. Other risk factors f Zesh Zes f o o t t h righ rig for primary spontaneous pneumothoraces include Copy smoking and family history. Copy

hi hi hi ures ures ures Q Q Q n n n a a a established cause A) Male gender h Correct. As mentioned above, primary h D) Trauma Incorrect. Trauma is a well h Zesnot f Zes pneumothoraces occur in patients f Zes fdoes o o o t t t of a pneumothorax. However, it cause a spontaneous h h h g g g i ri ri opyr The Copy without underlying lung disease or C Copy spontaneous pneumothorax. Blunt trauma can result trauma.

Iatrogenic (such as after lung biopsy or central line or pacemaker insertion)

h E) ures is a risk factor for a QCOPD C) COPD Incorrect. n a an h h s s e e fZ pneumothorax. However, it results in secondary not t of Z igh ight o r r y y p p primary spontaneous pneumothoraces. Patients Co Co who have extensive emphysema and large bullae are most at risk, as these are thin-walled, air-containing structures which are prone to rupture. Most i pneumothoraces (>70%) reshare secondary. In addition to u Q n many other lung pathologies which shan ha COPD,Z there are e f es the risk of a secondary pneumothorax, o t t of Z h h can increase g g i i r r opy Copy including airway disorders (such asC asthma/interstitial lung disease), infections (such as tuberculosis (TB)/ necrotising pneumonia/pneumocystis jiroveci), systemic connective tissue i disorders (such as Marfans shsyndrome/rheumatoid e r u syndrome/Ehlers-Danlos Q shan shan arthritis), of Ze and lung cancer. of Ze ight opyr C ight opyr

B) Smoking Correct. Smoking is a recognised risk factor for primary spontaneous pneumothoraces.

hi shi Marfans ures syndrome Incorrect. Marfans syndrome Q Qure n a shwhich is is a systemic connective tissue disorder of Ze tpneumothoraces. h known to increase the risk of It is g i r Copy considered a risk factor for secondary, not primary
spontaneous pneumothoraces.

in rib fractures which tear the lung surface, whereas penetrating trauma can injure the lung surface directly. In hi addition to air, there may be blood inrthe shi pleural ures space, resulting in a haemopneumothorax. Q Qu e A n a esh horizontal air uid level is a useful clue to the presence of Z t h g i of both air and uid in the pleural space (remember r y p Co will usually have a curving that a pleural effusion meniscus rather than a completely horizontal upper margin).

Qure

shi Cop t of yrigh

Zesh

ure an Q

shi

Qure

shi C ight opyr of Ze

Qu shan

resh

Cop

t of yrigh

Ze

Qu shan

ha f Zes o of Ze disorders t h htsystemic g g Most pneumothoraces are secondary, and a variety of lung and i i r r y y Cop Spontaneous primary pneumothoraces Cop typically occur in young, can be implicated.
slim, tall male smokers.

resh

i KEY POINT resh n Qu

Qu shan

resh

righ Copy

t of Z

Qu shan

resh

i righ Copy t of Z e

Qu shan

resh

i righ Copy t of Z e

Qu shan

resh

32 Chest X Rays - Case 1

Cop

t of yrigh

Zesh

ure an Q

shi Cop t of yrigh

Zesh

ure an Q

shi Cop t of yrigh

Zesh

ure an Q

shi

2. Which of the following clinical ndings would be supportive of a large simple right sided pneumothorax?
righ Copy t of Z e Ze t of Z ht of h g g i i r r y The correct answer is C) trachea. Hyperresonant percussion and op CCentral Copy e Qu shan resh i Qu shan resh i Qu shan

resh

reduced air entry on the right side of the chest.


resh i

u Qu Accurate clinicalh assessment of the patient is a key part of clinical n Qu practice, and is commonly assessed an Q ha han in s s s e e e Z Z Z of It is important to know the different of combinations of clinical ndings associated of with a examinations. right right right y y y p p p o o o C pneumothorax, a pleural effusion, lobar C collapse, and pneumonia. Briey, the chest C examination should follow the pattern of inspection, palpation, percussion, and auscultation. Look for symmetrical shape and chest expansion. Assess the position of the mediastinum (trachea and apex beat) and assess for chest expansion. Percuss and hiauscultate both lungs. Assessing routine shiobservations, such as oxygen shi ures Qure Qure saturations and h blood n n an Qpressure, is also important. a a h h CoCentral trachea. Dull percussion and reduced Co Co percussion and reduced A) air D) Central trachea. Hyperresonant entry on the right side of the chest Incorrect. This air entry on the left side of the chest Incorrect. These combination of ndings is suggestive of a right sided ndings would be in keeping with a simple left sided pleural effusion. With a pleural h effusion you would pneumothorax. hi hi s i re u ures ures expect to nd reduced chest expansion, a very dull/ Q Qdeviated Q n n n a a a E) Trachea to the left. Hyperresonant h h h stony dull o percussion f Zes note, absent or reduced breathht of Zes f Zes oright t t percussion and reduced air entry on the side of h h g g g i i i r r sounds, sounds pyr reduced vocal resonance, and no added the chest. Hypotensive, tachycardic Coon Copy Copy Incorrect. This the side of the effusion. With large effusions there combination of clinical ndings is worrying and should may be a shift of the mediastinum to the contralateral raise your suspicions of a tension pneumothorax. side. In addition to the usual ndings associated with a i hi sh shi e e r r B) Central trachea. Dull percussion and bronchial breathing simple pneumothorax, there is mediastinal shift to Qures u u Q Q n n n a a a h contralateral side and evidence of signicantly sh right side of the chest Incorrect. f Zes and crackles one the the esh t of Z of clinical ndings is in keeping with to t of Z cyanosis, h h h g g g i i i This combination impaired ventilation and circulation (hypoxia, r r r py opy opy Coa right sided pneumonia. Typically, there is C reduced hypotension, tachycardia,C reduced consciousness level). chest expansion, dull percussion, bronchial breathing Tension pneumothoraces are a medical emergency and with added crackles, and increased vocal resonance in need urgent treatment (there is a case covering tension pneumonia. pneumothorax i i more detail later in the chapter). sh shin shi han han C) Central trachea. Hyperresonant percussion and f Zes f Zes o o t t h h reduced rig pyrig air entry on the right side of the chest py osimple CoCorrect. C These ndings are consistent with a right pneumothorax. You would expect to nd reduced chest expansion, hyperresonant percussion, and absent breath sounds, with hino added sounds on hi ures There should be no ures Q Q the side of the pneumothorax. n n a esha Zesh mediastinal t of shift. t of Z C igh opyr C Qure Qure Copy right sha of Ze n Qu re ht o pyrig f Zes ht o pyrig f Zes ht o pyrig f Zes

resh

resh

KEY POINTs

igh opyr

ight opyr

of Ze

Qu shan

resh

hi hi shi if any ndings on clinical 1. Patients have ureswith a small pneumothorax may urefew ures Q Q Q n n n ha ha sha a more sensitive test for identifying a pneumothorax, f Zes examination. Chest X-ray is h f Ze f Zes o o o t t t h h ig ig ig particularly a small pneumothorax. opyr opyr opyr C C
2. Not all patients have classic ndings on history and examination. It is important to use your clinical ndings to request appropriate investigations, such as blood i tests and ai chest X-ray, to help narrow your differential diagnosis. resh resh

righ Copy

t of Z

Qu shan

righ Copy

t of Z

Qu shan

righ Copy

t of Z

Qu shan

resh

Chest X Rays - Case 1 33

Chest X-Rays

h the following are appropriate h 3. Which of diagnoses for a patient who esh f Zes f Zdifferential f Zes o o o t t t h h h rig rig rig Copy presents with sudden breathlessness? Copy Copy
Chest X-Rays

ure an Q

shi

ure an Q

shi

ure an Q

shi

cardiac diseases, and systemic problems. It is important to be able to formulate an appropriate differential diagnosis from the history and examination to guide suitable investigations and initial management. Establishing the time frame of the onset of breathlessness is very helpful in formulating your differential hi hi eshi refers to breathlessness that develops diagnosis. Sudden onset ur uresover seconds and includes pulmonary ures Q Q Q n n n a ha ha embolus, inhaled pneumonia, Zesh f Zespneumothorax, anaphylaxis, and f Zesforeign bodies. Dyspnoea associated f with o o o t t t h h h rig rig rig and exacerbations of asthma or COPD tend more slowly, over pydevelop opy Copy heart failure, metabolic acidosis,C Coto minutes to hours. Other conditions, such as interstitial lung disease and anaemia, have a much more chronic onset.

e e e t of Z t of Z t of Z h h h g g g i i i r r r y y There is a wide range of pathologies include respiratory conditions, py Cop Copwhich can cause breathlessness. These Co

Q shan

The correct answers are A) Pulmonary embolus, B) Pneumothorax, and hi hi E) Anaphylaxis. ures ures
Q shan

Qu shan

resh

i onset of is usually more subacute, eshi A) Pulmonary embolus r Correct. hi Pulmonary embolus can hbreathlessness r u es ures over hours rather than seconds. Other Qbreathlessness. Q n n n Qu occurring features cause very sudden Other symptoms a a a h h h s s s e Ze f Ze sputum, a raised include a productive cough with green include t of Zpleuritic chest pain and dizziness. Clinical h ht of ht o g g g i i i r r r y y y p abnormal temperature, and appropriate clinical ndings. examination of the chest often reveals Cop Cono Cop signs. There may be a swollen limb, suggesting an D) Heart failure Incorrect. Heart failure is a common underlying deep venous thrombosis. The patient may cause of breathlessness, but usually its onset is more have had recent surgery or have other risk factors insidious. i h s shi Patients may also complain of orthopnoea shi present, such as being post-partum, or having an re ure u Qure Q Q and paroxysmal nocturnal dyspnoea. There may be a n n n a a a underlying malignancy or a genetic prothrombotic Zesh sh Zesh examination previous history of cardiac disease. of Ze of of Clinical t t t h h h tendency. g g g i i i r r pyr jugular venous pulse and may demonstrate an elevated Copy Copy Co B) Pneumothorax Correct. A pneumothorax often results evidence of pleural effusions. in a sudden onset of breathlessness and pleuritic chest E) Anaphylaxis Correct. Anaphylaxis is a systemic and life pain. There may be a history of an underlying lung threatening i examples of conditions hi condition (see question 1h for shi allergic reaction. It usually occurs shortly res recontact u u ures after with the allergen (previous sensitisation Q Q Q n n n a a a which predispose to secondary pneumothoraces) or esh esh esh to the allergen is required). Symptoms t of Z The constellation of clinical ndings t of Z t of Z develop rapidly trauma. associated h h h g g g i i i r r r over minutes and include opy 2. Copy with a pneumothorax is discussed inC Copy facial swelling, wheezing, question breathlessness, urticaria, and, potentially, shock. C) Pneumonia Incorrect. Pneumonia and other infections Urgent management using the ABCDE approach and are common causes of breathlessness; however, the administration of intramuscular adrenaline is required. t of yrigh Zesh ure an Q shi t of yrigh h Zes KEY POINT ure an Q shi t of yrigh Zesh ure an Q shi

Cop

Copy

right

Use the history and examination ndings to help formulate a differential diagnosis to guide appropriate investigations and management. But remember that some patients, such as those presenting with anaphylaxis, may need urgent treatment i hi hi resh and examination. In these patients, before full history ures you will be able to complete an u ures Q Q Q n n sha use the ABCDE approach to assessment sha and management. sha of Ze of Ze of Ze

Cop

Cop

Copy

right

Copy

right

Cop

t of yrigh

4. Which of the following is the most appropriate initial imaging investigation in a patient hi hi hi ures a simple pneumothorax? ures ures suspected of having Q Q Q n n n a a a
Zesh C The correct answer is A) Erect PA chest X-ray. op t of yrigh Zesh Cop t of yrigh Zesh

Copy

i line imaging modality for suspected i The chest X-ray is r theh rst pneumothoraces. Imaging reshi hsimple u es ures Q Q Qu n n has a key role in conrming the diagnosis of a pneumothorax, assessing its size, and excluding ha ha hanother s s s e e e Z Z Z t of t of t of differential diagnoses. There are various different types of chest X-ray, which are discussed below. righ righ righ Copy Copy

34 Chest X Rays - Case 1

i i i resh resh resh u u u Q Q Q nCorrect. The standard method n a a D) Supine can A) Erect PA chest X-ray han chest X-ray Incorrect. Pneumothoraces Zesh Zesh chest f pneumothoraces f Zes f supine o o o t t t be very subtle and difcult to identify on for imaging is an erect PA chest h h h rig rig pyrig X-rays, as the air collects anteriorly, performed during inspiration. This provides CoX-ray Copy an Copy and there may not
be a clearly discernible lung edge as seen in an erect accurate and reliable assessment for a pneumothorax. X-ray. Supine chest X-rays are thus not performed A well inspired PA X-ray allows a better assessment of routinely. Instead they are reserved for trauma patients the lungs and cardiac contours compared to an AP or i i shi on a chest X-ray is shsafely e who cannot moved. Features suggestive ofQ a uresh r expiratory X-ray. The key nding u urebe Q Q n n n halung edge (visceral pleura) away ha pneumothorax on a supine chest X-ray include aa sharply displacement of the es Zesh fZ f Zes fdeep o o o t t t h h h outlined dome of the hemidiaphragm, a lateral from the chest wall, with no lung markings visible ig rig pyrig opyr Coperipheral Copy C costophrenic sulcus, and a hyperlucent upper quadrant to this. of the abdomen (due to the lucent gas within the B) PA and lateral chest X-rays Incorrect. The lateral chest pleural space overlying the upper abdomen). X-ray can provide helpful information if a pneumothorax i Whilst CT is the most sensitive and E) CT Incorrect. hi shi sh is not visible on the PA X-ray. However, it is not part of ure urefor ures Q Q Q n n n specic test a pneumothorax, its high radiation dose routine practice. ha ha ha f Zes f Zes f Zes and the (approximately equivalent to 400 chest X-rays!) o o o t t t h h h rig rig C) o Expiratory X-rays pyrig chest X-ray Incorrect. Expiratory satisfactory accuracy of standard C may Copy Copy PA chest X-rays mean make a subtle pneumothorax more readily visible. CT is not the rst line imaging modality for suspected However, there are not performed in the rst instance. pneumothoraces; instead, it is reserved for patients in Additionally, they can limit the assessment of the lungs whom there is continued suspicion of a pneumothorax and cardiac shadow. hi hi hi but no evidence s e r ures on a chest X-ray, or for assessing n Qures Qu Q n n a a a trauma patients. CT also provides the most e reliable h h h f Zes f Zes fZ s o o o t t t assessment of pneumothorax size. h h h g g g i i i
Chest X-Rays

Copy

Copy

KEY POINTS
hi

Copy

res that your environment can affect res res 1. Remember n Qu n Quhow easy or difcult it is to interpret n Qu a a a h h h s s s e Ze lit room, with glare and a low resolution f Ze X-rays. For example, a brightly fZ fward ght o ght o ight o i i r r r y y y p p p monitor, may make it odifcult or impossible to identify a small Co C Co pneumothorax. Always try to optimise your chance of picking up pathology by using diagnostic quality workstations in a dark room if you are making diagnostic decisions.

hi

hi

shi shi shi 2. Pneumothoraces can be difcult to see on chest Qure QureX-rays. Inverting the image can an Qure n n a a Also, lines esh make the lung edge more obvious. esh your eyes tend to see horizontal esh t of Z t of Z t of Z h h h g g g i i i r r r better than vertical ones; it is thus sometimes useful to rotateo the chest X-ray by Copy Copy C py 90 degrees so that the lateral chest wall (and therefore the lung edge) is horizontal.
3. Always check the apices on an erect chest X-ray and around the lung bases on a i supine chest shi X-ray for evidence of a pneumothorax. re resh

Cop

t of yrigh

Zesh

u an Q

Cop resh i

t of yrigh

Zesh

u an Q

Cop resh i

t of yrigh

Zesh

ure an Q

shi

5. Which of the following is the most appropriate management option for a previously healthy patient with a small, asymptomatic primary pneumothorax?
e of Ze of Ze up. t The t of Z t follow h h h correct answer is B) Conservative management with outpatient g g g i i i r r r Copy Copy Copy Qu shan Qu shan Qu shan

resh

A) Conservative management and discharge the patient This is to ensure that the patient has not deteriorated Incorrect. Whilst asymptomatic (not breathless) and the pneumothorax has not increased in size. patients with a small (<2cm) spontaneous primary Patients should be followed up with the respiratory hi hi resolution of their pneumothorax reshi pneumothorax can be safely managed conservatively team untilrcomplete s e r u u es Q Q n n n Qu with analgesia, it is important to arrange for the patient has occurred. Patients should also be instructed to ha ha ha s s s e e e Z Z Z f f to be reviewed return to hospital if they develop isymptoms. ht of in the outpatient clinic in 2-4 weeks. ght o ig ight o

Cop

The management of pneumothoraces depends on whether the patient is symptomatic, the size of the pneumothorax, and the r presence of underlying lung disease. As is the hi hicase with all potentially unwell hi u es ures ures Q Q Q n n n patients, it is useful and the ha initially to approach the assessment ha management of the patient using ha f Zes f Zes f Zes o o o t t t ABCDE approach. h h h rig yrig yrig

Cop

Copy

Copy

Copy

Copy

Chest X Rays - Case 1 35

i i i resh resh resh u u u Q Q Q n n Aspirate as much of the pneumothorax npossible as B) Conservative with outpatient follow up - haD) ha shamanagement f Ze f Zes f Zes symptomatic o o o t t t Incorrect. Patients with a large and/or Correct. See above. The patient should be h h h rig rig yrig opyexperience Copy instructed to return immediately ifC Cop primary or secondary pneumothorax need active they
symptoms, to identify those patients with enlarging pneumothoraces. All patients with a pneumothorax should avoid air travel until complete resolution. shi e r u Diving shouldn be permanently avoided in most Q a have h han s e patients who had a pneumothorax. f Zes of Z
Chest X-Rays

oxygen, and monitoring Incorrect. If the patient is not breathless and has a small primary spontaneous pneumothorax, he or she can usually be managed hi safely as an outpatient; ures admission to hospital Q n n ha oxygen is not required. In contrast, ZeshaE) and f high ow es Z f o o t ht symptomatic patients with a small (<2cm) secondary righ pyrig Copy pneumothorax should be admitted, Co treated with high ow oxygen (if necessary), and monitored for 24 hours. Supplementary oxygen not only improves hypoxaemia, but also increases the speed at which a shi Qure pneumothorax an resolves. an

ight ight o r r y y p p o o C) Admit for conservative management, C C high ow

Qur

management. Needle aspiration, using a 14-16G needle, is as effective as chest drain insertion, but reduces length of hospital stay and morbidity. These i shi should be performed in the safe triangle sh e r procedures u Q Qure in n a h s the mid-axillary region. If, following needle aspiration, of Ze timproved h g i the patients symptoms have and the r opy pneumothorax is C <2cm in size, then no further intervention is required at that stage. If, however, this is not the case, the patient may require a chest drain i (repeat needle aspiration should not be performed). esh eshi

ha Chest drain insertion Incorrect. Small f Zes bore (<14F) o t h chest drains are indicated in patients who do not ig opyr improve followingC needle aspiration and those with bilateral or tension pneumothoraces. shi righ Copy t of Z nQ esha ures

n Qu

Co

ht o pyrig

f Zes

Co

ht o pyrig

f Zes

Qure

hi

KEY POINTS

1. The size of the pneumothorax is one of the parameters used in deciding shi is dened by a rim of >2cm ureshi eshi treatment. A large pneumothorax Qurappropriate Qure n n a a an Q h sh schest sh e e e between the lung margin and wall, measured at the level of the hilum. Z Z Z f f f o o o right right right If this rim is <2cm Copy Copythe patient has a small pneumothorax. Copy 2. Patients admitted with a pneumothorax should be reviewed by a respiratory physician within 24 hours.

rig Copy

ht of

nQ esha

s es res 3. is a procedure in which the space is obliterated u Pleurodesis Qurpleural Qure n n a a h esh (either chemically or t of Zes t of Z h h g g i i r r py be used surgically) and can Co Copy in patients who suffer recurrent pneumothoraces. shi

hi

hi

hi

Cop

t of yrigh

Zesh

ure an Q

ight opyr

of Ze

Qu shan

resh

righ Copy

t of Z

Qu shan

resh

righ Copy

t of Z

Qu shan

resh

i RtAnt i IM hO P res resh LE A u u R nin G Q Q P O ints n han A pneumothorax esha : is a colle f Zes fnZ o o ct io t t of air h in h the rig pleural space. opyrig Copy C It can be primary, se condary to underlying lung disease, related to trauma, or related to med ica i l procedures. h hi s e ur Typical ures Q Q n n clinical features of a a ha e sim pl esh f Zes fZ o o t t pn eu mo h h th orax inclu g duced ch rig yr:ire est opde Copy expansion, hypeC rresonant percussion, and reduced air entry ipsilaterally, without evidence of m ed iastinal shift or i hi sh re rd iovascular compromise uca ures Q Q n n . ha The er ha ect PA chest X-ray is th f Zes f Zes o o t t h h e fir st line rig pyrig investigation forC o Copy su sp ected pneumothorax; however, CT is the gold standard. Treatment depend s on whether the patie nt is symp hi hi matic, the size of the resto u ures pn eu mo th Q Q or ax , n n and the presence of un ha sha derlyin go f Zes fZ lun ge disease. o t t h h g g ri ri Copy Copy

36 Chest X Rays - Case 1

Case 1...................189 Case 2...................195


ha Case 3...................203 f Zes o right y p o Case 4...................211 C

Cop

t of yrigh

Zesh

ure an Q

ABDOMINaL X-RaYS
Case 6...................225 Case hi 7...................233
Cop t of yrigh Zesh

shi

ure an Q

shi

Case 11.................263
res n Qu

Cop

t of yrigh

Zesh

ure an Q

shi

Case 16.................299 Case 17.................305 hi


Abdominal X-Rays

res n Qu

Case 8...................241 f ZeCase ................277 sha 13.


o right y p o Case 9...................247 C

Case 12.................269 hi

o right y p o Case 14.................285 C Case 19.................321

han Case 18. ................313 f Zes

Qure

Case 5................... 217


Qu shan

Case 10.................255
resh i

Case 15.................293
Qu shan resh i

Case 20.................327
Qu shan resh i

This introduction systematic framework for approaching abdominal e e Zeto the chapter is aimed at providing Z ofaZ ht of details and examples of the specic ht X-rays ht of more g g g i i i r r r X-rays. Further ndings discussed below are covered y y y Cop Cop Cop extensively in the example cases later in the chapter and in the bonus X-ray chapter.

Co

ht o pyrig

f Zes

hi KEY POINT ures Q n a h

2. Patient Details eshi


t of Z nQ esha ur

yrigh Systematic approach to Cop abdominal X-rays


1. Projection

It is important to check you are looking Z at the ht of g i r y correct X-ray from the C correct patient. op

nQ esha

ures

hi

The patient details should be listed on the lm.

Co

ht o pyrig

f Zes

h 2. Patient details
3. Technical adequacy Co

ure an Q

shi

ht o pyrig

f Zes

State the name, age (or date of birth), and the shi eshi r u Q Qure date on which the lm was taken. n n a a h h

4. Obvious abnormalities

3. Technical Adequacy

Co

ht o pyrig

f Zes

Copy

o right

shi review of 5. Systematic Qure n a h the X-ray f Zes


6. Summary

an an Zesh Zesh f f o o t t Check the X-ray includes the hemidiaphragms igh righ opyr and hernial orices. Copy down to the symphysisC pubis

The entire abdomen should be included in the i shi X-ray. Quresh Qure

p 1. Projection Co

t of yrigh

Zesh

ure an Q

shi C ight opyr

of Ze

If the entire abdomen has not been included, you need to decide whether a repeat/additional i i resh resh u u Q Q X-ray is required. han han

The standard abdominal X-ray (AXR) is an AP X-ray with the patient in the supine position. hi res You can assume this is u the case unless told Q n sha otherwise. of Ze

4. Obvious Abnormalities
If there is an h obvious abnormality, such as hi s i e r u ures Q Q small bowel dilatation, comment on this before n n a ha eshlm. Zes Zthe conducting your systematic review of ht of ht of

ight opyr

of Ze

p CoSometimes, Cop is Cop a lateral decubitus (the patient rolled onto their left side and the X-ray taken in an AP direction) or a lateral shoot through (the 5. Systematic Review of the Film i patient is supine but a lateral X-ray is taken) h hi eshi (see gure 1) ures Such X-rays should Assess urBowel ures the Q Q Q n n n abdominal X-ray is performed. ha ha ha f Zes They are most frequently ht of Zes f Zes o o be clearly labelled. t t h h ig rig When looking at the bowel o try to identify: pyrig opyr a Coundertaken C py in neonates to conrm or C exclude pneumoperitoneum. Large and small bowel. f Zes h ure an Q shi righ Copy to h ures Q n Bowel ha wall thickness. f Zes
Diameter of the i bowel.

yrigh

yrig

yrig

Co

ht o pyrig

righ Copy

t of Z

Qu shan

resh

Abdominal X-Rays - Introduction 181

PRESENTshi ure an Q YOUR h s e fZ ight o FINDINGS... r y p Co


t of Z e Qu shan resh i

i i resh resh u u Q Q n A 38 year old mans pain, han to A&E with acute left sided loin haurinary e presents fZ f Zes X-ray is o o frequency and haematuria. As part of his work up, an abdominal t t h h ig rig opyr performed. C Copy resh i C resh i righ Copy hi righ Copy hi rig Copy hi rig Copy shi Cop resh i C resh i righ Copy i righ Copy t of Z e t of Z e ight opyr of Ze t of yrigh ht of Z ht of Z t of Z t of Z e ight opyr of Ze resh i
Abdominal X-Rays

CASE 11

righ Copy

righ Copy i righ Copy hi righ Copy hi rig Copy hi rig Copy shi Cop

t of Z

Qu shan

Qu shan

righ Copy

t of Z

Qu shan

resh

t of Z

Qu shan

Qu shan

resh

righ Copy

t of Z

nQ esha

ures

t of Z

nQ esha

ures

nQ esha

ures

hi

rig Copy

ht of

nQ esha

ures

ht of

nQ esha

ures

nQ esha

ures

hi

rig Copy

ht of

nQ esha

ures

ht of

nQ esha

ures

nQ esha

ures

hi

Cop

t of yrigh

Zesh

ure an Q

t of yrigh

Zesh

ure an Q

Zesh

ure an Q

shi

ight opyr

of Ze

Qu shan

resh

i C ight opyr of Ze

Qu shan

Qu shan

resh

righ Copy

t of Z

Qu shan

resh

i righ Copy t of Z e

Qu shan

Qu shan

resh

righ Copy

t of Z

Qu shan

resh

i righ Copy t of Z e

Qu shan

resh

Qu shan

resh

Abdominal X-Rays - Case 11 263

Cop

t of yrigh

op ANNOtateD C X-raY resh i

Zesh

ure an Q

shi t of yrigh

Zesh

ure an Q

shi Cop t of yrigh

Zesh

ure an Q

shi

hi shi ures Q T11 han Qure n ha Rounded f Ze f Zes f Zes Left renal o o o t t t h h h rig rig rig calcifications outline Copy Copy Copy in keeping with renal calculi T12 hi hi hi ures ures ures Q Q Q n n n a ha ha Zesh f Zes f L1 f Zes o o o t t t h h h rig rig rig Copy Copy Copy Qu shan sh Qure i righ Copy ures hi Copy shi rig Copy ht of righ

Abdominal X-Rays

L2
t of Z nQ esha

righ Copy

t of Z

n esha

ures

hi

L3
ures hi

rig Copy

ht of

nQ esha

Q shan e L4 Z f to

Z ht of g i r y Cop Expected course of the left ureter (over the transverse hi ures processes of the eshan Q fZ lumbar spine) ight o r y p Co ures hi

nQ esha

ures

hi

ight opyr

of Ze

sh

ure an Q

L5
Z

nQ esha

ures

hi rig Copy ht of Z

nQ esha

Cop

t of yrigh

Zesh

ure an Q

shi

ight opyr

of Ze

Qu shan

resh

h f Zes o t h rig Copy Fixation screws projected over left sacro-iliac hi joint ures Q n ha f Zes o t h rig Copy resh i

ure an Q

shi Copy r

Partial sacralisation i of the L5 resh u Q vertebra han f Zes o t h ig

ight opyr

of Ze

Qu shan

resh

righ Copy

t of Z

Qu shan

resh

i righ Copy t of Z e

IN SUMMARY This abdominal X-ray shows multiple small left sided renal calculi.
resh i righ Copy t of Z e Qu shan resh i righ Copy

Qu shan

righ Copy

t of Z

Qu shan

resh

righ Copy

t of Z

Qu shan

t of Z

Qu shan

resh

264 Abdominal X-Rays - Case 11

Cop

t of yrigh

Zesh

ure an Q

shi Cop

t of yrigh

Zesh QUESTIONS

ure an Q

shi

This is a supine AP X-ray ofe the hi ur s Q n abdomen. sha

ht anonymised and the rigbeen oIt has C py timing of the examination is not available. I would like to check the name and date of birth, as hi es well as the time and date the Qurof n a h examination. f Zes ght o rihernial y p o The orices and right C ure an Q
hemidiaphragm have not been included on the image; therefore, this is a technically inadequate hiX-ray.

of Ze

Cop

Cop

A) Pain B) Nausea and vomiting C) Dysuria shi Qure D) Haematuria n a esh f ZAll o t of the above E) h g yri

ur following are symptoms n Qur 2. Which the nQ haof ha s e Z f f Zes o o t t associated with renal calculi? h h yrig yrig Cop

eshi

eshi

The most pertinent esh abnormality is of Z t h the rounded areas of calcication g i pyr Coprojected over the left upper quadrant at the level of T12/L1. Given their position (overlying the left renal outline) and appearance, shi Qure n a they are most in keeping with small sh e t of Z h renal calculi. g i r py

A) Uric acid ureshi Q shan oxalate e B) Calcium Z f o right C) Calcium phosphate Copy D) Struvite E) Cysteine

3. Which type of renal stone is least likely to be visible on an X-ray?


rig Copy ht of Z nQ esha

righ Copy

t of Z

nQ esha

ures

hi

ures

hi

There is no evidence of urinary calculi in the expected distribution of the left ureter, nor are there any visible i resh u right sided urinary tract calculi. Q n

Co

Co

f The ght o gas pattern is normal. ibowel r y p CoNo plain lm evidence of bowel obstruction, perforation or mucosal oedema. sh Fixation screws are projected Qure over n a h es joint. There is the left sacro-iliac t of Z h g i r an abnormal area of bone on the Copy left side of the L5 vertebral body which is in keeping with partial sacralisation of the L5 vertebra hi (a congenital anomaly, which ures is Q n esha signicance, usually ofo no clinical fZ t h rig the transverse process of where Copy the L5 vertebral body fuses onto the sacrum). Otherwise, there is no signicant abnormality of the shi imaged skeleton. Qure righ Copy t of Z e shan i

Zesh

Co

A) Abdominal X-ray/KUB (Kidney, Ureter, Bladder) B) Intravenous urogram C) Ultrasound hi i res resh u u Q Q D) Non-contrast CT KUB han han f Zes f Zes o o t t h h abdomen and pelvis rig pyrig E) Contrast enhanced CT of the py

Qu diagnostic modality of choice Qu 4. What is anthe an for h h s s e e Z fZ ht oidentifying renal calculi? pyright of pyrig Co

resh

resh

Copy

righ A) Urgent ureteroscopy and stone righ Copy retrieval

5. What is the most appropriate initial management of a well patient with renal colic and a small hi shi calculus at the left (<5 mm) left sided ure ures Q Q n n a ha eshKUB? junction identied on f Zes f ZCT t ovesicoureteric to

Co

B) Urgent extracorporeal shock wave lithotripsy (ESWL) C) Urgent percutaneous nephrostomy hi hi D) Conservative approach (supportive treatment with ures ures Q Q n n ha X-ray and anti-emetics and follow up with esha es f Zanalgesia fZ o o t t h h ig until the stone rig KUBs [if the stone is visible on r X-ray] Copy Copy has passed) E) Percutaneous nephrolithotomy

righ Copy

t of Z

Qu shan

resh

i righ Copy t of Z e

Qu shan

resh

Abdominal X-Rays - Case 11 265

Abdominal X-Rays

PRESENT Qureshi n esha Z f o YOUR right Copy FINDINGS...

hi hi A) Hypercalcaemia ures ures Q Q n n ha B) esha tubular acidosis f ZRenal f Zes o o t t h h rig C) Over hydration rig Copy Copy D) Inammatory bowel disease E) Recurrent urinary tract infections

1. Which of the following is/are risk factors for developing renal calculi?

Cop

t of yrigh

Zesh

ure an Q

shi

h h f Zes f Zes o o ANSWerS tO QueStIONS t t h h ig ig opyr opyr C

ure an Q

shi

ure an Q

shi Cop t of yrigh

Zesh

ure an Q

shi

1. Which of the following is/are risk factors for developing renal calculi?

r Copy

ight o

The correct hi answers are A) Hypercalcaemia, hiB) Renal tubular acidosis, hi ures ures ures Q Q Q n n n a Inammatory bowel disease h ha E) Recurrent urinary tract infections. ha D) and f Zes f Zes f Zes
r Copy ight o r Copy ight o

Abdominal X-Rays

Renal calculi are common, occurring in 5-10% of the population. They typically occur in middle age (30-60 years) and affect men more commonly than women. There are several risk factors for developing renal calculi.

A) Hypercalcaemia Correct. Calcium is a component of Normally, calcium within the lumen of the GI tract binds i hi of calcium h sform shi and the resultant calcium oxalate is poorly e e r r most renal calculi, in the oxalate or to oxalate u u ures Q Q Q n n n ha absorbed from the GI tract. Fat malabsorption ha occurs in sha calcium phosphate. There is increased risk of developing f Ze f Zes f Zes o o o t t t h h h patients who have hadp small igbowel resections or Crohns rig calcium containing renal calculi with conditions rig that cause o yr Copy hypercalcaemia, such as primary hyperparathyroidism, Copy disease (due to the C disruption of the enterohepatic malignancy, sarcoidosis, Addisons disease and circulation of bile acids). Unabsorbed fats preferentially medications (thiazides, vitamin D analogues, lithium). bind with luminal calcium. This results in oxalate binding with sodium B) Renal tubular acidosise s Correct. There is an increased risk hi hi in the GI tract. Sodium oxalate is absorbed hi es r r u u ures in the colon and results in high serum oxalate levels, Q Q Q of developing calcium renal stones in type 1 renal tubular n n n a a a h sh esh formation. increasing the risk of calcium oxalate f Ze f Zes f Zstone acidosis. This is due to alkaline urine, hypercalciuria and o o o t t t h h h g g g ri yri yri this E) Recurrent urinary tract infections Correct. Struvite stones opcondition. Copy low citrate which are associated withC Cop (magnesium ammonium phosphate) form in alkaline C) Over hydration Incorrect. Renal calculi are more common urine which contains ammonia. These conditions can in hot climates due to dehydration. Dehydration results occur in the presence of urease producing bacteria, such as in concentrated urine, which can become supersaturated i i i Proteus, and Enterobacter (urease metabolises h shKlebsiella with substances that es in stone formation. resh ure u Qurresult Q Q n n n urea into ammonia and carbon dioxide). Struvite stones a a a h sh bowel disease Correct. Calcium oxalate sh esmultiple D) Inammatory typically occur in patients who have had urinary of Ze of Ze of Z t t t h h h g g g i i i disease r stones are more common in patients with yr yr with vesicoureteric reux, pCrohns p tract infections such as those o Copy or those who have had a previous small Co C bowel resection. neurogenic bladder and obstructive uropathies.

shi shi Qure Qure n n a a esh esh The correct answer is E) All of the above. t of Z t of Z h h g g i i r r y y p p Co Co

2. Which of the following are symptoms associated with renal calculi?


rig Copy ht of Z nQ esha

ures

hi

Many patients with renal calculi are asymptomatic; however, renal calculi can cause a variety of symptoms.
A) Pain Incorrect. Renal colic is the classic symptom C) Dysuria Incorrect. Dysuria can occur with urinary tract i shi calculi. It is caused by a associated with urinary tract calculi. Itiis also commonly seen in cystitis (bladder e r resh resh u u u Q Q Q an calculus e becoming to infection). han lodged in a ureter, with resultant hyperhan inammation, which is often secondary h fZ s f Zes f Zes o o o t t t peristalsis of the ureter in an attempt to overcome the h h h D) Haematuria Incorrect.y Haematuria is another classic rig rig yrig calculi typically Copy blockage. The 3 anatomical sites at which Cop Cop with renal calculi. It is often clinical nding in patients become obstructed are a) the pelvi-ureteric junction, b) microscopic and is related to inammation and trauma where the ureter crosses over the iliac vessels, and c) at caused by the calculus. However, the absence of the vesico-ureteric junction. Additionally, any pathological haematuria on urinalysis does not exclude renal calculi. hi shi can cause stone impaction. shi e e narrowing, such as a stricture, r r u u ures Q Q Q E) All of the above Correct. As the previous answers n n n a ha The pain usually to sha starts acutely, spreads from the loin f Ze Zesh suggest, renal calculi can cause a variety f and f Zes of symptoms. o o o t t t groin comes in waves. In contrast to peritonitis, patients h h h ig rig rig They can also cause fever pyrin Copy with renal colic usually writhe around the Cobed Copy and rigors if there is superagony. added infection, acute kidney injury secondary to B) Nausea and vomiting Incorrect. Nausea and vomiting ureteral obstruction, as well as being asymptomatic. is common. It is mediated via an autonomic response to the pain. The ganglion which i receives pain signals from i i sh e r resh resh u the kidneys also supplies the stomach. nQ n Qu n Qu

Cop

t of yrigh

Zesh

r Copy

ight o

C cause loin pain similar in Dissecting or C leaking abdominal aortic aneurysms can nature to renal colic. In addition, if the aneurysm is adjacent to a ureter, there may be haematuria. Therefore, it is important to consider this serious differential i i years presenting with renal colic eshi particularly in patients agedu over 65 resh resh r udiagnosis Q Q n n n Qu a a a h h h for the rst time. s s s Ze Ze Ze r Copy ight o f r Copy ight o f

op

t of yrigh

KEY POINT

Zesh

op

t of yrigh

Zesh

266 Abdominal X-Rays - Case 11

3. Which type of shrenal stone is least likely to be esh on an X-ray? f Ze f Zvisible


Cop to yrigh

ure an Q

shi

The correct answer is A) Uric acid.

Cop

to yrigh

ure an Q

shi

Cop

t of yrigh

Zesh

ure an Q

shi

A) Uric acid Correct. Uric acid stones are the least radioopaque stones out of the listed options. They are typically hi radiolucent on X-ray butQ usually ures visible on CT. They form n in patients with increased sha levels of uric acid, such as those f Ze f who o o t h with gout or are being treated for myeloproliferative rig right y y p p o o C disorders. They account for approximately 10% C of renal calculi. Xanthine stones are also typically radiolucent but are a rare cause of renal calculi. B) Calcium oxalate Incorrect. Calcium shi containing renal Quretype of stone, accounting calculi are the mosta common n h for approximately f Zes 75%. They are the most radio-opaque f o t h g ight o pyri calculi. orenal opyr

C) Calcium phosphate Incorrect. See B. D) Struvite Incorrect. Struvite calculi form in the presence hi shi bacteria (see Question 1) and account e r u ures of urease producing Q Q n n a ha esh for approximately 15% of renal stones. They generally Zes f Zare o t h g radio-opaque, and are the second type of renal pyridensest Coare calculi. Most staghorn calculi composed of struvite. E) Cysteine Incorrect. Cysteine stones are rare. They occur in patients with cystinuria, an uncommon autosomalrrecessive hi condition. Cysteine stones are shi u es Q Qure n n usually radio-opaque on plain X-ray but less dense than a a h h Zes f Zes calcium containing or struvite stones. ght o

KEY POINT

Copy

ri

Copy

Not all renal calculi are radio-opaque on X-ray. Therefore, a normal abdominal/ i i hi resh resh u u ures Q Q Q KUB X-ray does not exclude renal calculi. CT is much more sensitive for identifying n n n a a a sh sh sh of Ze renal calculi. of Ze of Ze t t t h h h g g g i i i r r r

Copy

Copy

4. What is the diagnostic modality of choice for identifying renal calculi?


shi shi Qure Qure n n a a h h KUB. The answer is D) Non-contrast CT escorrect es t of Z t of Z h h g g i i r r Copy Copy rig Copy ht of Z nQ esha ures

hi

ha A) Abdominal X-ray/KUB f Zes Incorrect. Whilst 75-90% of renal f o t h calculi identify igare radio-opaque, they can be difcult to ight o r r y y p p Coon abdominal X-ray if they are small. Additionally, Co it can be difcult to differentiate urinary tract calculi from other causes of renal, abdominal or pelvic calcication such as nephrocalcinosis and calcied phleboliths. An shi ure abdominal/KUB X-rayn should be performed if a calculus Q a sh is identied CT; X-rays can be used to monitor its Ze fon o of t h response rig to treatment if the calculus is radio-opaque right y y p p o o C and visible on the X-ray. Additionally, if a patient C is known to have radio-opaque calculi, plain X-rays can be used if they re-present with acute pain to assess any change in position of the known i hcalculi. ures In the past, IVUs Q B) Intravenous urogram Incorrect. n ha f Zes test of choice for urinary tract ht of were the diagnostic o t h rig pyrig However, they have now been superseded Cocalculi. Copy by CT. IVUs involve intravenous administration of contrast followed by plain X-rays of the kidneys, ureters and bladder (KUB) once the contrast is within hi the urinary collecting system. Urinary calculi, including ures Q n radiolucent calculi, will show as lling defects ha f Zes tract. Additionally, urinary tract ht of within the t ourinary gh g Copy ri Copy ri

Imaging is used to conrm the diagnosis of renal/urinary tract calculi, exclude differential diagnoses, such as appendicitis and diverticulitis, and identify any complications, such as hydronephrosis. All the options listed can help diagnose urinary hi tract calculi. hi hi ures ures ures

nQ

Q nQ ha shan e Non-contrast esCT D) CT KUB Correct. Non contrast KUB Z Z f o ht is the imaging modality of o choice. pyrigIt is a low dose nonC contrast CT which has a sensitivity of 95-100% and a

C) Ultrasound Incorrect. Ultrasound can occasionally identify renal calculi. However, this modality is not as sensitive as CT. Additionally, it is usually impossible to image the entire hi ureter due to overlying bowel hi ures ultrasound has a low sensitivity for ures Q Q gas. Therefore, n n ha sha egood Zes f Zis detecting urinary tract calculi. Ultrasound at o t h rig which may be y p identifying urinary tract obstruction o C caused by calculi. Furthermore, it can identify other causes of loin pain and haematuria such as renal tumours, as well as gynaecological pathology, such as hi hi ovarian cysts, ureswhich can present in a similar manner. ures

Q an Q han shurinary obstruction (hydronephrosis) secondary to the e Zes Z f o tract calculus can be diagnosed. right Copy

higher specicity than IVU. It is readily available and quick to perform. As it is a non-contrast examination, it can be safely in patients with renal i hi shperformed ure(which ures Q Q impairment may be secondary to urinary tract n n ha ha Zes Zescan calculi). As well as identifying calculi, of KUB t CT

Copy

righ

Abdominal X-Rays - Case 11 267

Abdominal X-Rays

i shi Different types of renalucalculi approximately 75-90% of renal calculi ureshi resh have varying X-ray density. Overall, ure Q Q Q n n hastones are radio-opaque on CT. The are radio-opaque (i.e. visible) on plain X-rays, and almost exception shan esha esall e Z Z Z f f f o o o ht is calculi treated ght stones are ithese right that form in HIV patients beingo py pyrig with the protease inhibitor indinavir; opyr Co C C classically radiolucent even on CT.

helping to identify or exclude other differential diagnoses, such as appendicitis and diverticulitis (although the accuracy for this is limited by the low i dose nature of the examination and the lack of resh u Q n a h intravenous contrast). As discussed in Question 3, Zeshan s e Z f o t of atsmall proportion of urinary tract calculi ighnot righ rare y y p p o o C C be secondary visible on CT; however, there may still signs of urinary tract calculi, such as perinephric and peri-ureteral stranding.

i i i resh resh resh u u u Q Q Q n Contrast enhanced CT of the abdomen and pelvis assess for complications (e.g. urinary tract obstruction han haE) han s f Ze f Zes f Zes o o o t t t Incorrect. Standard contrast enhanced CT can identify causing hydronephrosis). Additionally, it permits an h h h rig rig rig opy opy urinary tract calculi, it uses a higher dose of radiation and pelvic organs, Copy assessment of the other abdominal C Cbut
than CT KUB and requires intravenous contrast. Therefore, non contrast scans are preferred. It does, however, provide a better assessment of the abdominal i shi organs. Additionally, a delayed phase u and pelvic scan, resh ure Q Q n ha where the patient is imaged when the contrast is being Zes f urogram) o t h excreted into the urinary tract (CT , would help rig Copy calculi as these would appear identify non-radio-opaque as lling defects in the urinary tracts.

Abdominal X-Rays

e Ze 5. h What of a well patient with of Ze t of Z is the most appropriate initial ht management ht ofrenal colic and a g g g i i i r r r y y y small (<5 mm) left sided calculus Cop Cop at the left vesicoureteric junction Cop identied on CT KUB?

Qu shan

resh

Qu shan

resh

Qu shan

resh

Copy

rig

The correct answer is D) Conservative approach (supportive treatment with shi shi X-ray KUBs [if the stone is ureshi analgesia up with ure Qure and anti-emetics and follow Q Q n n a a h h han s s s e e e Z Z Z visible on X-ray] until the h stone t of has passed). ht of ht of
Copy rig Copy rig

Copy

sh stone retrieval Incorrect. A) Urgent ureteroscopy and Qure n a Ureteroscopy for Zeshan Zesh is an invasive procedure reserved f o t t h of patients with larger stones, persistent pain or igh ig r r y y p p Co Co therapy. those who fail to respond to conservative A ureteroscope is inserted via the urethra, through the bladder and into the appropriate ureter. Most ureteric calculi are accessible. However, results are best for hi distal ureters. sthe e r those calculi located in u Q an shan e Z Z-esh B) Urgent extracorporeal shock wave lithotripsy (ESWL) f f o o t t igh righ ESWL is another treatment modality opyr reserved Copy Incorrect. C for patients who fail conservative management or have large stones. An external energy source causes shock waves which are targeted towards the stone. The aim is to fragment stones to shi them to pass through the reallow u Q ureters. The fragments can cause renal colic and ESWL han an sh e es E) Z f may fail in large and hard stones. to t of Z h h g g i i r r py o Incorrect. Copy C) Urgent percutaneous nephrostomyC Urgent percutaneous nephrostomy is needed in patients with an obstructed kidney and super-added infection. It is usually performed under sedation hi with antibiotic coverage ures to reduce the risk of Q n n septicaemia. Once the collecting system has esha esha Z Z f f o o t suitably drained and the infection treated, t been righ yrigh Copy the access gained via the nephrostomy Cop can be used for nephrolithotomy (procedure to remove the stone) if required. An alternative approach to draining an infected, obstructed collecting i sha system is from below retrograde fashion) ure(in Q n n sha withfan endoscopically placed ureteric stent e esha Z Z f o o t extending from the bladder, up the ureter ight righ rand Copy into the renal pelvis. Copy D) Conservative approach (supportive treatment with analgesia and anti-emetics and follow up with X-ray KUBs (ifs the hi stone is visible on urehas passed) - Correct. Q X-ray) until the stone n n a esh(<5 esha Z Z Most small mm) stones in the ureter pass f f o o t t righ Copy righ

The management of urinary tract calculi depends on the size and site of the stone and whether there is evidence of associated infection.

i hi spontaneously, particularly if they are located distally. resh ures Q Qu n a Patients can therefore be treated conservatively sh of Ze obstructed t h if there is no evidence of an infected, g i yr Cop collecting system (see above). The mainstay of

Qure

treatment is analgesia (NSAIDs such as diclofenac are used as 1st line), anti-emetics and IV uids if dehydrated. alpha i Medications such as tamsulosin (an e shor shi e r r blocker) nifedipine (a calcium channel blocker) u u Q Q n a shthe stone. If can be used to facilitate the passage Zeof ofsuch t of h g visible on X-rays, the progress stones can be i r py Co monitored by serial X-ray KUBs. If the stone fails to pass after 4-6 weeks, the pain becomes intolerable, or if the patient develops an infected, obstructed collecting is i hi system, an alternative treatment option resh ures (see above). u required Q Q an Zesh f Incorrect. Percutaneous nephrolithotomy o t h rig is used to fragment Percutaneous nephrolithotomy Copy large renal calculi. It is usually reserved for large or complex stones or patients in which ESWL and ureteroscopy have failed.

shi

268 Abdominal X-Rays - Case 11

e common and can caus hi esh ea ures Qur Q variety of symptoms, ho n ha wever, mo es Zsto nes are fst o t asymptomatic. h rig Copy 75-90% of stones ar e visible on X-rays, whils t almost all are visible on CT. Non-contrast low do se CT KUB is the imaging hi shi re ures mo dality of choice, with XQu Q n ray KUBs rese h ve da for f Zers follow-up of radio-opa o t h qu g e i sto ne r s. y Treatment dependsCop on the size and site of th e stone and the presence of infection or urinary tract obstruction.

IMPORTAnT LEARninG POinTs: Renal ca i lculi ar

ight opyr

of Ze

Qu shan

resh

Cop Spine Case 1.................367

t of yrigh

Zesh

ure an Q

ORTHoPAEDIC X-RAYS
Cop Wrist Case 1.................407 t of yrigh Zesh

shi

ure an Q

shi

Cop Knee Case 2..................481 Hip Case 3.....................443

t of yrigh

Zesh

ure an Q

shi

Knee Case 3..................489 i Hip Case 4.....................451 Wrist Case 2.................415 Spine Case 2.................375 shi shi sh e e r r u u Q Wrist Case 3.................423 Q5.....................457 Qure n n n a a a Tibia/Fibular Case 1....497 Hip Case Spine Case 3.................383 h h h es Zes Zes t of Z ht of ht of h g g g i i i r r r y y Tibia/Fibular Case 2....505 Hip Case 6.....................465 Hip Case 1.....................429 Shoulder Case 1...........391 Cop Cop Copy Ankle Case 1.................513 Knee Case 1.................. 473 Hip Case 2.....................435 Elbow Case 1................399
Orthopaedic X-Rays

hi hi hi ures ures ures Q Q Q n n n a chapter is aimed at providing a systematic ha framework for approaching orthopaedic ha X-rays. This introduction to the sh f Ze f Zes f Zes o o o t t t h h h g Further and examples of the specic X-rays igndings discussed below are coveredpmore ridetails rig extensively in the opyr Copy cases later in the chapter and theC Co y example bonus X-ray chapter. hi shi shi ures Qure Qure Q n n n a a a h h h f Zes Systematic approach to orthopaedic f Zes X-rays f Zes o o o t t t h h h g g g ri ri ri Copy Copy Copy 4. Obvious abnormalities 1. Projection
2. Patient details 5. Systematic review of the X-ray 3. Technical hi adequacy

KEY POINT

1. Projection

Co

ht o pyrig

f Zes

ure an Q

Co

ht o pyrig

f Zes

ure an Q

6. Summary hi

Assessment of any bone or joint in general requires eshiis one too few! These at least two views one view Qur Q Q n a sh shan normally consist of AP and lateral X-rays (gure 1). f Zeshan e e Z Z f f fractured it is o If the shaft of a long bone isig ht o ight o right pyrsome pyr above and below imperative to X-ray the joint sites, such as the scaphoid, where CoFor Copy Co because of the potential for additional injuries fractures are difcult to detect, it is routine to (fracture or dislocation) at these sites. obtain more than two views.

For some patients in whom the clinical suspicion of a fracture is high but is not evident on the usual two views, additional views may be requested, i hi such as resh rotation views of the hip. uinternal ures

rig Copy

ht of

nQ esha

ures

hi

p Co A

t of yrigh

Zesh

ure an Q

shi

i Comment shiwhether the patient is skeletally mature reon resh u u Q Q an epiphyses/growth plates). This is (fused useful h han f Zes because the types of injury andh f Zes vary o o t t pathology h rig rig Copy Copy and mature patients. between skeletally immature res n Qu hi

Cop

t of yrigh

Zesh

res n Qu

hi Cop t of yrigh

KEY POINT

Zesh

s Qure n a h Remember: children develop and Zes t ofrates h g i mature at different and r Copy
therefore the age of a patient does not tell you whether they are skeletally mature or not.

hi

righ Copy

t of Z

Qu shan

resh

i righ Copy

e t of Z Figure 1.

Qu shan

resh

righ Copy

t of Z

Qu shan

resh

i Copy r

Image A: AP view of this nger shows apparently normal alignment of the interphalangeal joints Image B: The lateral view shows posterior dislocations at hi shi both of these joints! ure ures Q Q n n sha X-rays dramatically show why one view shatoo few! These eone f Ze f Zis o o t t h h g g i i

righ Copy

t of Z

Qu shan

resh

Copy

Orthopaedic X-Rays - Introduction 335

Copy

Z ht of rigA

esh

ure an Q

shi Cop t of yrigh

sh Ze B

ure an Q

shi Cop t of yrigh

Zesh

ure an Q

shi

Medial epicondyle ossification centre

righ Copy

t of Z

Normal Qur shan appearance of the anterior pad

eshi

righ Copy

t of Z

Qu shan

eshi C

Lateral epicondyle ossification centre

ight opyr

of Ze

Qu shan

resh

righ Copy

t of Z

righ Copy

t of Z

Trochlear ossification centre Capitellar ossification centre

righ Copy

t of Z

righ Copy

t of Z

nQ esha

Olecranon ossification centre

i resh righ Copy

t of Z

nQ esha

i resh

Radial head ossification centre

righ Copy hi

t of Z

nQ esha

ures

hi

fZ fZ fZ Figure 20 . Lateral (Image A) and AP (Image B)ielbow ossication ght o X-rays demonstrating the various p ght o centres ight o i r r r y y y p p Trochlear, Olecranon, External/Lateral Co (Capitellar, Radial head, Medial/Internal epicondyle, Co Co epicondyle = CRITOE).

nQ esha

ures

hi

nQ esha

ures

nQ esha

ures

hi

The Gartland classication system is useful for categorising the severity of supracondylar fractures, and the X-ray features are summarised in Table 1. It relates to extension type supracondylar injury, where the hi hi hi s s e e r r u u ures distal fragment is displaced 5% of supracondylar fractures, and Q posteriorly. Flexion type injuries comprise Q Q n n n a a a sh sh are classied separately. Zesh of of Ze of Ze

Copy

right

Gartland 1

Copy

right

Gartland 2

right Copy Gartland 3

Cop

t of yrigh

Zesh

ure an Q

shi Cop t of yrigh

Zesh

ure an Q

shi Cop t of yrigh

Zesh

ure an Q

shi

ight opyr

of Ze

Qu shan

resh

i C ight opyr of Ze

Qu shan

resh

i C ight opyr of Ze

Qu shan

resh

Qu fracture. An undisplaced supracondylar han s e Z f A line drawn along the anterior edge ight o (anterior humeral line) rhumerus y p of the o C transects the capitellum. Qu shan resh i

resh

Qu This is a displaced supracondylar han s e Z fracture but the of posterior cortex righthumerus is intact. y p of the distal o C However, the anterior humeral line now does not transect the capitellum. Copy r esha n

Copy

f Ze ight o

f Z for supracondylar fractures. Table 1: The Gartland Classication ight o


Qu This is a completely displaced han s e Z supracondylar fracture, ht of and the rigof y p posterior cortex the distal o C humerus is not intact. These are the injuries most often associated with neurovascular compromise hi and compartment syndrome. s shi e r Qu Qure Copy r f Ze ight o shan

resh

resh

Orthopaedic X-Rays - Introduction 353

Orthopaedic X-Rays

Radial head ossification centre

Qu shan

resh

Qu shan

resh

Qu shan

resh

PRESENTshi ure an Q YOUR h s e fZ ight o FINDINGS... r y p Co


t of Z e Qu shan resh i

i re resh u u Q Q A 75 year old mans has fallen over at home. He was unable to get up anand han esh f Ze to A&E by ambulance where it was f Zhis o o has been brought noted left leg was t t h h ig ig opyr opyr shortened and externally rotated. This is his X-ray. C C resh i C resh i righ Copy hi righ Copy hi rig Copy hi rig Copy shi Cop resh i C resh i righ Copy i righ Copy t of Z e t of Z e ight opyr of Ze t of yrigh ht of Z ht of Z t of Z t of Z e ight opyr of Ze resh i

HIP CASE s5 hi

righ Copy

righ Copy i righ Copy hi righ Copy hi rig Copy hi rig Copy shi Cop

t of Z

Qu shan

Qu shan

righ Copy

t of Z

t of Z

righ Copy

t of Z

nQ esha

ures

t of Z

nQ esha

ures

nQ esha

ures

hi

rig Copy

ht of

nQ esha

ures

ht of

nQ esha

ures

nQ esha

ures

hi

rig Copy

ht of

nQ esha

ures

ht of

nQ esha

ures

nQ esha

ures

hi

Cop

t of yrigh

Zesh

ure an Q

t of yrigh

Zesh

ure an Q

Zesh

ure an Q

shi

ight opyr

of Ze

Qu shan

resh

i C ight opyr of Ze

Qu shan

Qu shan

resh

righ Copy

t of Z

Qu shan

resh

i righ Copy t of Z e

Qu shan

Qu shan

resh

righ Copy

t of Z

Qu shan

resh

i righ Copy t of Z e

Qu shan

resh

Qu shan

resh

Orthopaedic X-Rays - Hip Case 5 457

Orthopaedic X-Rays

Qu shan

resh

Qu shan

Qu shan

resh

Cop

t of yrigh

op AnnoTATED C X-RAY Qu shan resh i righ Copy i righ Copy hi righ Copy hi rig Copy hi rig Copy shi Cop resh i t of yrigh ht of ht of

Zesh

ure an Q

shi t of yrigh

Zesh

ure an Q

shi Cop t of yrigh

Zesh

ure an Q

shi

righ Copy

t of Z

t of Z

Qu shan

resh

i C ight opyr of Ze

Qu shan

resh

righ Copy

t of Z

Qu shan

resh

t of Z

Qu shan

resh

i righ Copy t of Z e

Qu shan

resh

Orthopaedic X-Rays

righ Copy

t of Z

nQ esha

ures

t of Z

nQ esha

ures

hi righ Copy t of Z

nQ esha

ures

hi

rig Copy

ht of

nQ esha

ures

nQ esha

ures

hi rig Copy ht of Z

nQ esha

ures

hi

rig Copy

ht of

nQ esha

ures

nQ esha

ures

hi rig Copy ht of Z

nQ esha

ures

hi

Cop

t of yrigh

Zesh

ure an Q

Zesh

ure an Q

shi Cop t of yrigh

Zesh

ure an Q

shi

e of Ze a displaced left intracapsular IN SUMMARY This X-ray tshows t of Z h h g g i i r r C Copy Copy fracture of the neck of femur. I would want to see a lateral X-ray of the hip to complete my radiological assessment of the fracture. hi hi hi ures changes of the hips and ures are evident and theren Qures Q Q Degenerative spine n n ha ha ha f Zes f Zes f Zes o o o t t t h h h ig vascular calcification. rig rig is C marked opyr Copy Copy ight opyr resh i righ Copy t of Z e resh i righ Copy t of Z e resh i

of Ze

Qu shan

Qu shan

resh

Qu shan

resh

righ Copy

t of Z

Qu shan

Qu shan

Qu shan

458 Orthopaedic X-Rays - Hip Case 5

Cop

t of yrigh

Zesh

ure an Q

shi Cop

t of yrigh

Zesh QUESTIONS

ure an Q

shi

PRESENT hi ures Q n YOUR ha f Zes o t h rig Copy FINDINGS...


This is an AP X-ray of the pelvis of hi a skeletally mature patient. ures

Zes It has been and the f anonymised o t h rig of the examination is not y p timing o C available. I would like to conrm the patients details and timing of the examination before I make hi ures any further assessment. an Q
The ght ois adequately riX-ray y p o C penetrated, with no important areas cut off.

Z Z ht of ht of g g i i r r y y 2. What grade is this fracture Cop Cop using the Garden


A) 1 shi B) 2 Qure n a h es 3 f ZC) o t h g ri D) 4 Copy E) A

Classication?

f Zes

righ Copy

t of Z

nQ esha

ures

hi

of There is a displaced fractureh i res u the left hip. It is an intracapsular Q n a Zesh of the femoral subcapital offracture t h g i pyr Coneck. There is shortening and external rotation of the femur.

Qur is No other fracture nvisible. ha of Ze s

eshi

ht There pyrig are no areas of lucency Coor cystic changes to suggest a pathological fracture.
in There is a loss of joint space i resh u Q both hips with osteophytes, n esha and f Zsclerosis o subchondral t h pyrig cysts consistent with Cosubchondral osteoarthritis. Degenerative changes are also hi the visible in the lumbar spine and ures Q n lumbosacral esha f Zjunction.

A) Hemiarthroplasty B) Total hip replacement C) Surgical xation shi eshi Qur Qure n n a a D) Traction and bed rest esh esh t of ZE) Early mobilisation t of Z h h g g i i r r Copy Copy

3. What is the best hi surgical management for this shi ures Q Qure n n a a 75 year old patient if they are independent sh esh of Zand of Ze t t h h g g medically t prior to the injury? i i opyr opyr
C

Cop

ha es f ZA) Hemiarthroplasty o t h yrig

4. If this fracture occurred in a t and well 20 year old, which would be the best surgical i i resh resh option? u u Q Q n n
B) Total hip replacement Cop C) Surgical xation D) Traction and bed rest hi ures E) Early mobilisation nQ

t of yrigh

Zesh

p oThere is calcication of the C femoral vessels bilaterally, in keeping with diabetes or renal failure. shi Qure n a h a There are Zes surgical clips f few ht oover g i r projected the left thigh y Cop
suggesting previous surgery.

to yrigh

ha e f Zes o t t of Z h h g g i i r r 5. Which of these is absolutely necessary prior Copy Copy

Qu shan

resh

to the patient having surgery?

A) Consent from the patient B) Nil by mouth hifor 6 hours hi ures ures Q Q n n ha cross-matched ha C) Blood f Zes f Zes o o t t h h g rig riincluding D) Results of a full set of bloods urea and Copy Copy electrolytes E) None of the above

righ Copy

t of Z

Qu shan

resh

i righ Copy t of Z e

Qu shan

resh

i righ Copy t of Z e

Qu shan

resh

Orthopaedic X-Rays - Hip Case 5 459

Orthopaedic X-Rays

han

i hi A) Stabilise the patient esh ur ures Q Q n n ha ha Identify any medical issues es f ZB) f Zes o o t t h h rig rig C) Take a full history including Copy Copya history of the mechanism of injury D) Take a full social history E) Request r a CT hiof the hip to allow accurate hi u es ures Q Q n n planning ha espreoperative esha

1. The initial management of this fracture involves which of the following?

Cop

t of yrigh

Zesh

ure an Q

shi

h h f Zes f Zes o o AnSWERS To QuESTIonS t t h h rig rig Copy Copy resh i

ure an Q

shi

ure an Q

shi Cop t of yrigh

Zesh

ure an Q

shi

1. The initial management of this fracture involves which of the following?


t of Z e The correct answers are A) Stabilise the patient, B) Identify any issues, Ze Ze t ofmedical ht of h g g i i r r y C) Take a full history including a history of the mechanism Cop Copy of injury and Qu shan Qu shan resh i Qu shan resh i

righ Copy

D) Take a full social history.


shi

re re urea full history including a history of the A) Stabilise the patient Take n Qu Correct. Stabilising all trauma shaC) n Q n Qu a a h h s s eis important however there are other of Ze e history of the patients mechanism of injury Correct.o A full t of Z points to consider. Patients should t fZ h ht h g g g i i i r r r important be mechanism of injury is very important. Did they trip y y y Cop Cop Cop assessed using the ATLS ABCDE algorithm if there or did they collapse? is any concern about major trauma. Stabilisation D) Take a full social history Correct. It is important includes uid resuscitation and early pain relief. to takeh ai full social history from all patients who hi Analgesia should be e using the WHO pain shi rprescribed res fractures to the femoral neck. This u u ures sustained includes Q Q n starting at the most appropriate shan nQ ladder ass a guide, a a h h s e block is a very effective method of e of assistance Znerve where the patient lives, the requirement f Ze step. t of A t of Z h ht opain h g g g i i i r r r y y y with their activities of daily living, the use of walking opa trained relief but should only be performed Cop Cby Cop aids both in the house and outside and an abbreviated member of staff. mental test (AMT). The degree of independence will impact on surgical decision-making. B) Identify any medical issues Correct. Patients with i h hi hi femoral neck fractures are typically elderly and ures ures a CT of the hip to allow accurate n Qures Q E) Q Request n n often have other medical co-morbidities which must a a a h sh esh preoperative planning Incorrect. This is not f Zes o of Ze of Z be considered. These include chronic co-morbid t t t h h h g g g i i i r r r y y y required. The main questions which needs to be p p p conditions such as renal failure, COPD, Co Co ischaemic heart Co answered for preoperative planning are whether the disease and dementia as well as acute problems which fracture is intra or extracapsular and the degree of may have contributed to the fall (e.g. infection) or displacement. AP and lateral X-rays can almost always resulted from the fall (e.g. intracranial haemorrhage). i h hi hi answer be used if s These must be identied ures these questions. CT of the hip cann ures Qure and treated early to Q Q n n a a a there is high clinical suspicion of a hip fracture but no h patient for surgery. Patients may also optimise the Zesh Zesh imaging f Zes of alternate to t of to t an evidence on the X-rays. MRI is h h h g g g be on warfarin. This will need to be reversed prior i i i r r r py Copy surgery. However, the indication for Co Copy strategy in such cases. warfarin must be considered before reversing it, as the risks of stopping it may be high. In these cases, or if you are in doubt, you should discuss thes case i with a haematologist i i re h resh resh u u Q Q for advice.han Qu han han C ight opyr of Ze s C eshi ight opyr of Ze s C ight opyr of Ze s

shi

shi

Orthopaedic X-Rays

i hi resh for: ur tests should also be performed u ures Q Q Q Blood to assess n n n ha ha ha f Zes f Zes f Zes o o o t t t h h h A) Pre-operative anaemia patients may need a pre-operative rig rig rig transfusion Copy Copy Copy
B) Pre-operative renal failure or electrolyte derangement which may affect anaesthesia and uid management

KEY POINT

righ Copy

t of Z

Qu shan

i i hi sh C) Blood group and save surgery can associated with signicant blood loss esh re urbe ures Q Q n n ha and should be prepared esha f Zfor f Zes o o t t h h rig rig D) Inammatory Copymarkers patients may have sepsis Copy
E) Clotting drugs or liver dysfunction may affect clotting and may need to be corrected pre-operatively

righ Copy

t of Z

Qu shan

resh

righ Copy

t of Z

Qu shan

resh

righ Copy

t of Z

Qu shan

resh

460 Orthopaedic X-Rays - Hip Case 5

Cop

t of yrigh

Zesh

ure an Q

shi Cop t of yrigh

Zesh

ure an Q

shi Cop t of yrigh

Zesh

ure an Q

shi

2. What grade is this fracture using the Garden Classication?


t of Z e correct answer is C) 3. The Qu shan resh i righ Copy t of Z e Qu shan resh i C ight opyr of Ze Qu shan resh i

righ Copy

h igh igh pyrig opyr opyr CoThe C C classication system goes from 1 to 4, where 1 and 2 are undisplaced and 3 and 4 are displaced.

The subtle differences between 1/2 (undisplaced) and 3/4 (displaced) are best appreciated by looking at the trabeculation (lines seen within bone) in the different fractures. In Garden 1 fractures, the hi hi shi point vertically in comparison to u trabeculation lines in the head normal. In Garden 2 fractures, res ures Qure Q the Q n n n a a a h h they are all in line. than sh In Garden 3 fractures, the trabeculation f Ze f Zes lines in the head are more horizontal f Zes o o o t t t h h h g g g normal. has r displaced but returned to its normal alignment and so the i ri pyri In Garden 4 fractures, the head Cotrabeculations Copy Copy are all in line.
A) 1 Incorrect. In Garden 1 fractures, there is an i at the fracture incomplete fracture withu impaction resh Q n a site with the femoral sh head tilted into a valgus (more of Ze Therefore, the capsule is likely to t t upright) position. h g i yrigh pyr intact and the risk of AVN (avascular p Coremain Co necrosis) is reduced. Surgical xation using screws prevents displacement of the fracture and is the best option for most patients. Keeping the patients own femoral i resh or need for u head reduces the risk of dislocation Q an patient has little pain, this hthe revision surgery. If Zes f o t t h fracture surgery yrigh pyrig could potentially be treated without op Coand C with full weight bearing but follow-up X-rays are needed to ensure the fracture does not displace. B) 2 Incorrect. In Garden 2 fractures, the fracture is i complete undisplaced and again surgical xationureshi resh uand Q Q n a is is hpossible as the risk of AVN is low. This fracture s shan e e Z Z f f o to often grouped with Garden 1 fractures yrigh as undisplaced p o C and treatment is essentially the same although as the fracture is complete there is a higher risk of displacement if the fracture is treated non-operatively.

shi is a Garden 3 fracture. In Garden ureshi C) 3 Correct. This Qurethere is displacement and there is a Q n a 3 fractures, sh shan e e Z Z f f o o patients, a high chance of AVN. In the majority htof pyrig hemiarthroplasty or THRC iso necessary.
D) 4 Incorrect. In Garden 4, the fracture is fully displaced. For the vast majority of patients, THR or i hemiarthroplasty resh is required.

Cop

t of yrigh

Zesh

ure an Q

shi Cop t of yrigh

h sh A to D. E)e s A Incorrect. The classication is from 1fto enot Z Z4 o t h rig Copy

u an Q

ure an Q

shi

ight opyr

e f Ze of Ze 1. Undisplaced fractures have to t of Z h h a lower risk of AVN in comparison to displaced g g i i r r opy Copyof displacement fractures. Use the C Garden Classication to consider the degree
and then this will help guide your surgical decision making.

Qu shan

resh

KEY POINTS shi Qure


shan

Qu shan

resh

2. 1, 2 give aid hiit a screw, 3, 4, Austin Moore is an himemoire for the treatment of hi resbe ures u ures Q Q Q intracapsular fractures. Cannulated screws can used in undisplaced fractures n n n ha ha ha f Zes (Garden 1 or 2), whereas an f Zes f Zes o o o arthroplasty is usually required for displaced t t t h h h rig rig yrig opy Copy Cop fracture (Garden 3C and 4). Austin Moore is an outdated type of arthroplasty.

righ Copy

t of Z

Qu shan

resh

i righ Copy t of Z e

Qu shan

resh

i righ Copy t of Z e

Qu shan

resh

Orthopaedic X-Rays - Hip Case 5 461

Orthopaedic X-Rays

Intracapsular fractures often need surgery to have the femoral head replaced, but in certain cases this is not necessary, and in some cases conservative management might hi be considered. The Garden hi shi sintracapsular e e r r u u ures Classication is used to categorise the degree of displacement of an femoral neck fracture, Q Q Q n n n a sha to the femoral head has been fdisrupted. eshdenes esha and as such, also how likely it is that the tblood supply of Ze t of Z to Z

Cop

t of yrigh

Zesh

ure an Q

shi Cop t of yrigh

Zesh

ure an Q

shi Cop t of yrigh

Zesh

ure an Q

shi

3. What is the best surgical management for this 75 year old patient if they are independent and medically t prior to the injury?
t of Z e The correct answer is B) Total Ze ht of hip replacement. g i r y Cop Qu shan resh i Qu shan resh i C ight opyr of Ze Qu shan resh i

righ Copy

A) Hemiarthroplasty Incorrect. A hemiarthroplasty C) Surgical xation Incorrect. In this age group, it is is a half hip replacement. The ball of the ball and not advisable to attempt xation for a displaced socket joint is replaced but the socket is left alone. intracapsular fracture as the risks of further surgery This option has some advantages such as a low risk are too high i if AVN develops. However, if the fracture hi hi resh ures the implant differs from u ures of dislocation.n However, is undisplaced or minimally displaced, then this Q Q Q n n a a a sh sh esh normal anatomy and patients will often not beo able to may be a good option. The risko of AVN in displaced f Ze f Ze fZ o t t t h h h g g g i i i r r r return to full function. It is therefore usually reserved fractures is above 30%, but may be much lower in y y y Cop Cop Cop for patients with limited premorbid mobility. In undisplaced fractures. addition, if a patient had arthritis in the joint prior to D) Traction and bed rest Incorrect. Intracapsular the fracture, then there is the chance of ongoing pain fractures rest. i do not respond well to traction and bed because the arthritis is hi present in the acetabulum. hi sstill s re resh some extracapsular fractures will Q re u u u Although heal with Q Q n n n a a a h h sh Zes Zes B) Total replacement Correct. In a THR, the ball time, as this fracture is displaced intracapsular, of hip of Ze of and t t t h h h g g g i i i r yr the likelihood is the o pbetter pyr will not heal (non-union) Copy and socket are replaced, allowing for Coa C fracture correction of the prior anatomy and a better return and the patient will be left immobile with all the to function. NICE guidelines state that THR should associated/related complications. This treatment be considered for independent patients with a option may be considered in very frail patients who i h hi displaced intracapsular fracture of the neck of the arerexpected to die from an underlying or acutereshi s ure u es u Q Q n n femur who need no more than one stick to mobilise. medical problem (in these patients, surgery is not a a an Q h h h s s s e e e Z fZ fZ This takes slightly longer and is itherefore appropriate). t ofoperation ght o ght o igh i r r r y y y p p p o compared associated with slightly more blood Co Closs Co with a hemiarthroplasty. There is also a higher chance E) Early mobilisation Incorrect. Very occasionally, patients will present with an undisplaced of dislocation. However, by replacing both the ball intracapsular fracture where the fracture is old and and socket, it is possible to get a more anatomical i i the patient is able to mobilise. In such eshi h s healed or rebetter return to function. resh implant which allows u u ur Q Q n a an circumstances, it is possible to avoid an Q an operation. h h h s s s In addition, there are further benets for patients e e e Z fZ of Z X-rays are t of a However, should furtherrpain ght develop, igh ight o i with fracture and pre-existing hip joint arthritis, r r y y y p p p o o has been no displacement Co C Cthere repeated to ensure as replacing both the femoral head and lining the that would warrant surgical intervention. acetabulum will help alleviate the symptoms of arthritis.

righ Copy

The aim of surgery in fractures of the neck of femur should be to allow early mobilisation with limited chance of further surgery. opt for hi In the elderly, most surgeons will hifor either a THR or a hemiarthroplasty hi ures ures ures Q Q Q n n n intracapsular sha esha fractures. esha

Orthopaedic X-Rays

t of Z

righ Copy

t of Z

righ Copy

t of Z

e e e t of Z t of Z t of Z h h h g g g i i i r r r py ao t and well 20 year old, which would Copy 4. If this fracture occurred in C Copy be the best surgical

Qu shan

resh

Qu shan

resh

Qu shan

resh

option?

hi hi ures ures Q Q n n a correct answer is C) Surgical xation. hThe ha f Zes f Zes o o t t h h rig rig Copy Copy

righ Copy

t of Z

Qu shan

resh

r Copy

ight o

There is a signicant difference between femoral neck fractures in the elderly (> 65 years) and the young. Young patients are usually t and the fracture is often associated with high-energy trauma. They are hi hi of the native femoral hip is crucial hi expected to live longer preservation ures and be more active. Therefore, ures uresin Q Q Q n n n a young esha with an intracapsular fracture.Zesha Zpatients Zesh

r Copy

ight o

r Copy

ight o

462 Orthopaedic X-Rays - Hip Case 5

A) Hemiarthroplasty Incorrect. This surgical option would provide a poor outcome for a 20 year old, resulting in limited mobility and future problems i from the metal secondary to marked wear r tear esh u and Q n femoral head against sha the patients own acetabulum.

Cop

t of yrigh

Zesh

ure an Q

shi Cop t of yrigh

Zesh

ure an Q

shi Cop t of yrigh

Zesh

ure an Q

shi

D) Traction and bed rest Incorrect. This is not appropriate for patients in this age group with this fracture.

Patients under 65 should be considered for surgical xation C) Surgical xation Correct. Ideally, the patient needs to go straight to theatre. The fracture can be accurately of their intracapsular neck of i h hi femur fracture within 6 hours hi s s reduced and xed with either 3 screws or a short e e r r ures Qu Qu Q n n n a a a dynamic hip screw sh eshand a second screw. There is still a of Zesh of injury. Doing so may reduce f Zof f Ze o o t t t signicant risk AVN of the femoral head, which would h h h g g g yri and preserve pyri a further operation. However, as this pyri the riskC of AVN op Corequire Co patient is the femoral head, improving young and t, the risks of further surgery are relatively the long-term functional low. It is therefore desirable to keep the patients femoral head and accept the possibility of further shi shi outcome. shi

re surgery, rather than replacing n Qu it in the rst instance. right o ha f Zes Copy

Copy

right

han f Zes

Qure

Copy

right

han f Zes

Qure

i necessary prior to the patient i 5. Which of these is absolutely having surgery? resh resh rig Copy ht of Z nQ esha u

The correct answer is E) None of the above.

rig Copy

ht of

nQ esha

rig Copy

ht of

nQ esha

ures

hi

i i None of these are absolutely operation. In an emergency situation, ureshi reshnecessary for a patient to have an resh u u Q Q Q an to wait for the patient to be adequately you may not be able ife they are sh shan fasted or to consent the patient shan e e Z Z Z f f f o o ht o Similarly, blood results may not htavailable unconscious. be in time. Although these p are inot ght absolutely ig pyrig opyr o yr Conecessary C C in such patients, they are desirable and should be considered in a standard patient with a femoral neck fracture. hi 6 hours Incorrect. It is generally hi A) Consent from the patient u r Incorrect. for eshi Informed consent B) Nil by mouth ures ures Q Q Q n n n a should be obtained from the patient or guardian. important to make sure the patient is fasted for ha ha h6 Zes f Zes consenting should either be the ht of Zes f the o o t t Ideally, the person hours prior to the operation. This reduces risk h h rig rig pyrig surgeon or someone who can perform of aspiration of gastric contents Cooperating Copythe Copyon the anaesthetic operation. If not, then the person consenting should induction. Although this applies for all surgical patients, have a full understanding of all the risks and benets this is especially relevant in trauma, which itself delays of a surgery together with the technical steps of the gastric emptying. Again, in some circumstances, this hiallowed sufcient s shi and emergency surgery should ureshi e e r r operation. The patient should be may not be possible, u u nQ nQ an Q In some circumstances, such habe sh time to come to decision. not delayed if the patient is not fasted. Instead, ea es esha Z Z Z f f f o o o t ht ight as an unconscious patient requiring emergency the anaesthetic team can perform rig yrigh py opsurgery, opyr a rapid sequence Coit C C is not possible to obtain consent, in which case it is induction, which minimises the risk of aspiration. acceptable to perform the surgery if it is judged to be in the patients best interests. righ Copy t of Z e Qu shan resh i righ Copy t of Z e Qu shan resh i righ Copy t of Z e Qu shan resh i

Orthopaedic X-Rays - Hip Case 5 463

Orthopaedic X-Rays

E) Early mobilisation shi Incorrect. Young patients with ureshi Qure fracture are best treated by surgical Q ana intracapsular n h han sis e es e Z Z Z xation even if the fracture is undisplaced. It also f f f o o o ht right right B) Total hip replacement Incorrect. If the femoral very unlikely for a young person pyrig Co Copyhead Copy to present with a does not survive or surgical xation is not possible, healing intracapsular fracture, as this is a high-energy then this is the best surgical option. However, a THR injury which will be picked up on initial presentation. does not last forever and it is likely a 20 year old with a hi hi hi THR will need further surgery in the future. Therefore, ures ures ures Q Q Q n n n a it is usually best to surgical xation (see C) in hattempt ha ha es f Zwith f Zes f Zes o o o t t t young patient femoral neck fractures regardless h h h rig rig rig py Coof Copy Copy degree of inital displacement.

KEY POINT

Cop C) Blood cross-matched Incorrect. Patients Cop

t of yrigh

Zesh

ure an Q

shi t of yrigh

Zesh

ure an Q

shi op t of yrigh

Zesh

ure an Q

shi

undergoing surgery for femoral neck fractures are at risk of signicant intra-operative blood loss and should have a valid group hiand save (G&S). If there is hi ures is at high risk of bleeding, ures Q Q concern that the patient n n a ha Zesshould Zesh f they f then have blood cross-matched prior to o o t t h h rig pyrig cross-matched Copy surgery. In an emergency, waiting for Co blood is not necessary as O negative or type specic blood is readily available. D) Results of a full set of bloods shi including urea and Qure Elderly patients with neck electrolytes a - Incorrect. n n h es esha Z Z of femur fractures often have multiple medical f f o o right right Copy conditions and most take several medications. Copy These can adversely affect their renal function and electrolyte balance, which needs to be recognised and addressed prior to surgery. It is hi therefore important such patients esdiscuss urto Q n n a with the anaesthetist. If you are unsure about h s e esha Z Z f f o o t t the patients other co-morbid conditions or past righ yrigh Copy medical history, you should seek advice Copfrom the medical registrar. In emergency situations, the full blood result may not be available at the time of surgery. In such cases, the i anaesthetic team esh r may have to correct any electrolyte abnormality u Q an han esh during the operation. fZ f Zes
Orthopaedic X-Rays

C E) None of the above Correct. Although not absolutely necessary, these should all be done for the standard patient with a neck of femur fracture. ight opyr of Ze Qu shan resh i

Copy

right

rig Copy

ht of

nQ esha

ures

hi

ion helps classify intracap ureshi s sul ar Qurefractures based Q han on the degree of displa s e Z cem ent. Undisplaced fractures right of (Ga rden 1 and 2) are at lower y p o risk of AVN, as thC e capsular blood vessels are less likely to be damaged. These patie nts can be considered for surgical xation using, for example, cannulated screw shi s, shi e r Qu thus preserving the patient s native femo an Qure ralh he Displaced fractures (Ga f Zes ad. o t rden 3 an h d 4) are g at significant pyri risk of AVN and usu o C all y require some form of arthroplasty. Hemiarthrop lasty is a good option for minimally mobile elderl y patients who are frail. igood for more i A THs Rh is e ac r tiv e older patients but n Quresh Qu a carries a higher risk of dis h location . of Zes o t t h h In g g yo i i un ger patients, fix r yr ing the fracture improves op Copy functional outcomeC if the femoral head can be preserved. If the younge r patient then develops AVN, they ar shi e young and fit enough to survive a shi re u se co Q Qure nd op era tio n n n. a a esh esh t of Z t of Z h h g g i i r r Copy Copy

IMPORtAnt LEARninG POints: Thh e Ga i rden classificat

Cop

t of yrigh

Zesh

ure an Q

shi C Displaced op

t of yrigh

sh Intra-capsular fracture Qure Zesh an

i C Undisplaced op t of yrigh Zesh

ure an Q

shi

ight opyr

of Ze

Qu shan

i Young resh C i

Elderly

Surgical Fixation

Assess mobility and medical state

ight opyr

of Ze

Qu shan

i Young resh C

Elderly

ight opyr

of Ze

Qu shan

resh

Surgical xation

righ Copy

t of Z

Qu shan

resh

If mobile and independent THR

Ze ht of g i r y p C If o requires walking
aids, carers or care home hemiarthroplasty

Qu shan

resh

Assess mobility and patient medical state

righ Copy

t of Z

Qu shan

resh

righ Copy

t of Z

Qu shan

resh

righ Copy

t of Z

Qu shan

Independent, well, then consider surgical xation i

resh

Unwell, dementia, in pain, consider hemiarthroplasty

righ Copy

t of Z

Qu shan

resh

464 Orthopaedic X-Rays - Hip Case 5

of Ze CT Head......................... 525 right

CT Cervical Spine.......... 530

Copy

sh

ure an Q

shi

of Ze CT in Orthopaedics...... 530 right Copy

CT SCANS
sh ure an Q

shi Copy

CT Chest......................... 531

Ze CT Abdomen and t of Pelvis.......535 righ

sh

ure an Q

shi

hi hi hi ures ures ures Q Q Q n n n ha ha What are CT scans and how are of Ze The sha CT image f Zes f Zes o o t t t h h h rig rig yrig you to view Looking through a CT scan allows Copy performed? Copy Cop they
axial, coronal and sagittal planes of the body A CT scanner essentially consists of an X-ray tube (gure 2). The standard CT image is the axial which spins around the CT table (gure 1). The (transverse) image hi and on this the left side of thereshi shi ure ures u X-ray tube is housed in the gantry, which is the Q Q image corresponds to the right side of the n n an Q ha ha hbody s s s e e e Z Z Z donut shaped f part of the scanner. The patient lies of (for example in, gure 16, the iheart, ofwhich is a left ght o ritable right r ght y y y p p p on the (usually on his/her back). The couch o o o sided structure, is on the C C C right side of the image). travels through the centre of the gantry as the X-ray At rst glance, this may appear counterintuitive. tube spins around the patient, taking hundreds of However, considering how we look at a frontal chest X-rays from different angles. The data are processed X-ray helps us understand why the CT images are shi i i sh resh rethis by a computer to produce Quimages. Quin Qure displayed way. n n a a h h han

Co

ht o pyrig

f Zes

rig Copy

ht of

nQ esha

ures

hi

patient is standing directly opposite and looking towards us (i.e. we are face to face). Therefore, their left hand side is in the right side of our eld of shi cardiac shadow and other left sided shi view, andu rethe Q so Qure n n a a h structures appear on the right side of the X-ray, and sh s of Ze of Zethe same t t h h g g vice versa. A coronal CT image is exactly i i r r Copy Copy opposite and looking its as if the patient is standing directly at us.

Co

ht o pyrig

f Zes

f Zes o t h g When we look at a frontal chest ri X-ray, it is as if the Copy

Now imagine h the i patient lies back onto a bed (or eshi res u the a CT table) so their feet are nearest us and their Q Qur n an h h h s s s e e e Z atfthem, their left fZ f Z head is furthest away. As we look ght o ight o ight o side still remains in the right i r r r y y y p p p Co Co Co side of our eld of view and vice versa. So, if we take an image in the axial (transverse) plane with the patient lying on their back and viewh it from their feet end, then we will i shi s i Figure 1. Diagram of a CTu scanner e e r r resh u u see that their left sided structures, such as the heart, Q Q Q n n han sha esha will be on the right side of the image. When f Zgantry f Ze f Zes we look o o o t t t The hole in the is relatively large and the h h h ig rig pyrig at axial CT images on a C monitor, opyr it is as if we are Copy Co scan very quick (lasting seconds), making CT standing at the feet end of the patient and looking suitable for, and well tolerated by, most patients, up towards their head. including those who are unwell. Patients need to

ure an Q

shi

i i i be able to lie at on their back The same principles apply to MRI images. reshand remain still, resh resh u u u Q Q Q n n n albeit for a short s period of time. Depending on ha ha e ha fZ f Zes Sagittal f Zes o o o the scan, they may be required to hold their breath t t t h h h rig rig pyrig Copy Co Copy momentarily. Contrast agents (IV, oral or rectal) Axial may be required to improve diagnostic accuracy of Coronal the scan (this will be decided by the radiologist). Indications for contrast wille be considered within i h hi hi ur s a hot ushing ures ures Q Q Q n n n the text. IV contrast can cause ha ha sha f Ze f Zes f Zes o o o sensation when administered the radiographers t t t h h h rig rig yrig Cop Copy Copy will warn the patient about this.
CT produces images with much greater contrast between structures than is usually seen with X-rays, shi rethan u although MRI is even better CT. Q an

Co

ht o pyrig

f Zes

righ Copy

t of Z

Qu shan

resh

i Copy

righ

Figure 2. Axial,n coronal Qure a h s and of Ze planes t sagittal

shi

CT Scans 521

CT Scans

MRI Spine. ...................... 549


Qu shan

of Z MRI y Head. right ......................544 Cop

esh

ure an Q

shi

t of Z MRCP.............................. 553 yrigh Cop

MRI SCANS
esh ure an Q t of Z e Qu shan

shi Cop

MRI Small Bowel.......... 554


i righ Copy resh i

Z MRI Knee y & Other ht of Joints.....555 rig

esh

ure an Q

shi

Ze What it and how is it t of his g i r y p o C performed?

resh

ight opyr

of Ze

Qu shan

resh

The physics behind MRI is very complicated. It is hi how MR images hi not necessary to fully understand ures ures Q Q n n are produced (in the hasame way you can drive a car Zesha f Zesexactly how the engine works!); o of t h without knowing rig right y y p p o o C C however, a basic understanding is helpful.

righ Copy

t of Z

Qu shan

resh

In simple terms, MRI uses a magnetic eld and radiowaves to produce images. iThe patient lies shi shi resh Qucauses Qure Qure inside the magnet,hwhich hydrogen ions n n n a a a h h s f Zethe f Zes f Zes o o o (protons) within body to align (either with or t t t h h h g g g ri ri yri against Figure 1. Diagram of a MRI scanner. Copy the magnetic eld). A radiofrequency Copy pulse Cop of a specic magnitude and duration is then sent through the patient. This transfers energy to the protons (excitation) and changes their direction shi shi eshi r u Q Qure Qure of alignment. Overhtime, the energy from these n n n a a a s sh eshno fZ One of the benets of MRI overh CT is that of Ze of Ze o excited g protons is lost (decays) and the protons t t t h h g g i i i r r opyr as described ionising radiation used. Instead, realign This Copy with the magnetic eld (relaxation). Copy C above, MRI uses a magnetic eld and radiowaves process emits radiofrequencies which are detected to produce the images. However, MRI scanning has by the MRI scanner and result in the image. The other hazards associated with it. speed of the relaxation process depends on the i i hi resh resh u u ures Q Q Q size and duration of the initial radiofrequency n n n a a a sh esh esh of Ze of the substance in whichithe pulse, the structure t t of Z t of Z h h h g g g i i r r r hydrogen ions/protons are in and the inuence Copy Copy Copy of surrounding hydrogen ions/protons. Thus, the The strong magnetic eld of the MRI scanner different tissues and substances result in different can turn ferromagnetic objects, such as scissors signals which are detected by the MRI scanner and i i i h s andQ non-MRI re resh compatible oxygen cylinders, into resh u u u Q Q inuence the image produced. han han han to cause f Zes f Zes projectiles, with the potentialh f Zesserious o o o t t t h h igstaff. The MRI injury or death to patients or rig In contrast pyrig to CT, the hole in the MRI scanner opyr Co Copy where C magnet is essentially always on; therefore, only the patient lies is relatively small, the scans take a MRI compatible metallic objects should be taken long time to be acquired and the scanner is noisy. into the MRI scanner room. Patients who are claustrophobic i or unable to lie still h hi hi s e urnot be able to tolerate ures elds can interfere with some ures for prolonged periods may Q Q Q The magnetic n n n a ha ha a MRI. Zesh f Zes f Zes electronic devices, such as certain fpacemakers and o o o t t t h h h rig rig rig must be checked y y y p p p o o o cochlear implants. Such devices C C C Various different types of sequences can be acquired to see whether they are MR safe before a patient which provide different types of information. The or staff member with one of them can enter the science behind the different types of sequence is MRI room. hi i understood by hi sh complex and does not need to be e r u ures ures Q Q Q n n n a ha non-specialists. weighted images are useful for f Zesha esh f ZT1 f Zes o o o t t t h h h demonstrating anatomy and lymph nodes, whereas rig rig pyrig Co Copy uid Copy T2 weighted sequences are useful for showing These can result in heating of tissues and and therefore areas of oedema. structures. Usually, the body can dissipate the heat i with vasodilation. However, the potential for burns Images can be acquired in various planes. However, h hi hi ures ures ures Q Q Q n n n if non-MRI compatible monitoring leads and a usually cannot be reformatted eshexists a unlike CT, the images ha Zesh fplanes f Z electrodes are used. f Zes o o o t t t into different once they have been acquired. h h h g g g ri ri ri

Magnetic elds

Radiofrequency elds

Copy

Copy

Copy

MRI Scans 543

MRI Scans

Use of ionising radiation

p Cop Ultrasound.............................558 Pelvic Co Cop Neck Ultrasound............................562 Musculoskeletal Ultrasound.........564

t of yrigh

Zesh

ure an Q

ULTRASOuND SCANS
t of yrigh Zesh ure an Q hi t of yrigh Zesh hi

shi

shi

t of yrigh

Zesh

ure an Q

shi

Chest Ultrasound............................558 FAST Scanning..................................563 Ultrasound Guided Procedures.....564


s ures ..................560 Abdominal Ultrasound. Vascular Ultrasound.......................563 Qure an an Q Cop t of yrigh Zesh Cop ight opyr of Ze Qu shan resh

What is it and how is it performed? n Qureshi

allows the velocity to be calculated (the angle between the ultrasound beam and the direction hi hi of ow also ures needs to be measured). The velocity ures Q Q n n a ow can be used to determine the e a of of degree h sha Z sh f Ze f Zes a stenosis. Doppler also provides f tracing o o o t Ultrasound uses very high frequency sound waves t t h a of h h rig rig pyrig Copy are inaudible to humans) to produce Coimages. Copy and direction (which the ow in terms of amplitude The ultrasound waves are produced by the ultrasound (waveform) and the appearance of the waveform transducer and travel as a beam through the body. can be a useful indicator of pathology. i When they come across a boundary between two h hi hi es es Ultrasound usually a quick and well-tolerated ures Qurcan Quris Q n n n different tissues, the waves either be reected or a a a esh esh esh it is performedo in the radiology fZ f Zexamination. Ideally, fZ o o refracted. The ultrasound waves which are reected t t t h h h g g g i i i r r r y y y department using the departmental ultrasound Cop are detected by the transducer and are Cop Cop back used machines. The patient can be scanned on the to make the images. The amplitude of the reected ultrasound couch or in a trolley/bed depending on waves, combined with the time they take to return to their clinical condition. Portable ultrasounds can the transducer, are used to generate hi the image. s e eshi who are unable to travel to Qureshi r be usedQ on urpatients Qu n n a a h han eshultrasound department. However, the portable Zestravel Zthe Zes Ultrasound waves well through solid and f f f o o o t t t igh righ righ ultrasound machines produce liquid waves opyr Copymedia. In contrast, the ultrasound C Copyinferior images compared with the departmental machines, meaning are scattered by gases. Therefore, gel is used as an the examination is usually less comprehensive. interface between the transducer and the patient to improve the transmission of ultrasound waves The patient may i have to move into different positions i shibody. re resh resh u u between the transducer and the Q Q during n n the examination; therefore, patient mobility n Qu B-mode (brightness mode) produces real-time 2 that are returned and detected. Therefore, it is easier dimensional images (gure 6A). The amplitude to image supercial rather than deep structures. of the echo for each pixel is represented using a Additionally, the i i image quality in overweight and eshi resh ultrasound image. resh r grey-scale. This is the standard u u Q Q obese au n ademonstrated anpatients is often reduced and occasionally an Q h h h s s s Pathology fmay be by a change in e e e f Z is too o Z if the patient t of Zdiagnostic scan is not possible r ht o g ightor ighin i r r the size shape of a structure, or an alteration y y y p p p large. Co Co Co its brightness (echogenicity). Colour ow imaging is used to show the direction of ow and provides an estimation of velocity hi eshi no ionising radiation, it has no Qureshi ures is usually blood uruses Ultrasound Q Q (gure 6B). The h ow assessed n n n a sha es any esha e signicant adverse effects and is safe to use in all Z Z Z f f f owg but can be owing or moving substance, o o o ri ht right right y y y patients. p p p o o o C such as a jet of urine entering the bladder C C from the ureter. The colour ow information is superimposed on the B-mode image. In general, red indicates ow towardss the i As mentioned above, ultrasound travels well through hi ultrasound h hi ure yellow/green turbulent ures ures Q Q Q transducer, blueh away, and n n n solid useful for a a eshaand liquid. It is therefore most o Zesh and f Zes in velocity and/or the direction f Zimaging f kidneys ow. Alterations of o o t t t solid organs, such as the liver, h h h rig rig pyrig can indicate pathology. Coow Copy Copy such as the bladder. spleen, and uid lled structures, The main common indications are discussed below Doppler imaging can be used to accurately calculate the velocity of ow (gure 2). The change but further details can be found in the Royal College i (Making the best use of reshi hi ultrasound waves of Radiologists hguidelines in frequency of the transmitted ures ures u Q Q n n clinical radiology services/iRefer). an Q ha received by the transducer Zesha compared with Zesthose Zesh

a ha esha Zesh Zand Zestissue f f f o o o cooperation are important. The more t t t There are three main types of ultrasound imaging: igh righ righ opyr through, the fewer Copy Copy Ctravel ultrasound waves have to

Use of ionising radiation

Main indications

r Copy

ight o

r Copy

ight o

r Copy

ight o

Ultrasound Scans 557

USS Scans

Myocardial perfusion scan............ 567 Bone imaging.......................568


Qu shan i resh

Zesh Zesh Zesh f f f o o o t t t h h h Lung ventilation-perfusion scan... 566pyGenitourinary scans............567 PET/CT......................569 ig r rig rig Copy Co Copy

ure an Q

NUCLEAR MEDICINE
shi ure an Q shi

ure an Q

shi

Ze What it and how is it t of his g i r y p o C performed?

hi hi ures ures Q Q n n a body, and hence provides functional ha in the sh f Ze f Zes rather o o t t h h than anatomical imaging. This has many advantages; rig pyrig Copy Co it is particularly helpful as there may be signicant

functional changes before there is a resultant anatomical change (e.g. the function of a kidney Nuclear medicine (nuclear imaging/gamma i h hi hi s may signicantly e ur ures decrease before any obvious ures imaging) uses radiopharmaceuticals to produce Q Q Q n n n a ha anatomical change occurs). Conversely, anh anatomical sha images. Radiopharmaceuticals are composed of twoof Ze f Zes f Zes o o t t t h h h abnormality may persist long after ig appropriate rig rig components; a radionuclide and a pharmaceutical. opyr Copy Copy C treatment even though it is no longer functionally Radionuclides are unstable isotopes which undergo active (e.g. brotic scarring at the site of a previous radioactive decay. The radiation emitted from certain tumour deposit). Such changes may not be detectable radionuclides can be detected and measured, for i i h h hi s s techniques, such as CT or MRI, but by other u imaging example by a gamma n camera. The pharmaceutical ures Qure Q re Q n n a a a h radionuclide to tissues of h functional changes may be picked Z h nuclear the up with is used to localise the f Zes f Zes f es o o o t t t h h h g g g medicine techniques. ri diagnostic 99 pyri interest. For example, Technetium py pyri methylene diphosphonate (99mTc MDP) is a commonly The main limitation of nuclear imaging is the lack used radiopharmaceutical in bone scanning. The of spatial resolution and poor anatomical detail. methylene diphosphonate (pharmaceutical) is a Therefore, functional and anatomical imaging are shi i hi restherefore resh bisphosphonate analogue and localises to u u Q Q Qure complementary and the two are increasingly used n n n a a a h h h 99m s s s e activity. As Tc (a radionuclide) areas of osteoblastic f Ze f Ze t of Z h ght o ig ight o concomitantly. i r r r y y y decays, it emits gamma rays which can be detected. p p p Co Co Co PET is a specic type of nuclear medicine. Fluorine-18 The exact combination of pharmaceutical and (18F) combined with deoxyglucose (FDG) is the most radionuclide depends on the indications for the commonly used radiopharmaceutical. Deoxyglucose i scan. The main radiopharmaceuticals will be briey shi shi e r is a glucose analogue and therefore localises to Quresh u ure Q Q n n n a a a mentioned below. h sh of increased metabolic activity. The esh e es sites physics t of Z t of Z t of Z h h h g g g i i i r r r y behind PET is different from other forms of nuclear The methods of acquiring the images vary Cop Copy Copy 18 imaging. As F decays, it releases a positron (a depending on the type of scan. In general, the positively charged electron), which travels only a few radiopharmaceutical is delivered to the patient (IV, the body before being annihilated by oral, inhaled) and after an appropriate amount of time, millimetres in i shi shi reaction results in two high energy e e r r resh an electron. This u u u Q Q Q the patient lies in the gamma camera, which detects han shan shan being released simultaneously esopposite photons f Ze f Ze f Zin the location and amount of radiopharmaceutical in o o o t t t h h h rig rig yrig directions. A specic PET camera the body. Cop Copy Copy is used to detect the two annihilation photons and from this determine X-rays, CT, MRI, ultrasound and uoroscopy where the annihilation reaction occurred. PET is provide anatomical imaging (i.e. they acquire almost always combined with CT (PET-CT) to improve i i shi and pathology is sh e r images of anatomical structures the quality of the data (the CT is used to permitQuresh u ure Q Q n n n ha ha from sha identied as changes attenuation corrections and it allows data Zes f Zes in the normal anatomy). By f Ze fthe o o o t t t h h h rig nuclear medicine provides functional rig contrast, PET to be fused onto a CT,o pyrig provides detailed Copy Copy C which imaging: the pharmaceutical components take anatomical information). part in metabolic reactions and other processes in the body (e.g. bisphosponate analogues used hi hi eshi of osteoblastic in bone imaging localise to urareas ures ures Q Q Q n n n a ha sha imaging relies on radioactivefdecay activity, uorodeoxyglucose used in PET Nuclear Zeshof f Zes f Ze o o o t t t h h h ig rig is metabolised like glucose and some rig scanning yra radionuclides and therefore source of ionising Copy Copy Copis radiopharmceuticals used in renal imaging are radiation. When the radionuclide is administered ltered by kidneys). to the patient, it usually travels throughout the body and therefore results in a radiation dose to the i imaging process i The use of a pharmaceutical in the hi sh shcontrast, re re u u entire body (in X-rays and CT only give a ures Q Q Q n n n therefore meanse nuclear medicine can assess a ha sha signicant radiation dose to the areas Zesh imaged). f Z s and other functional processes f Ze f being o o o t t t metabolic reactions h h h g g g ri ri ri

Co

Co

Co

Use of ionising radiation

Copy

Copy

Copy

Nuclear Medicine Scans 565

Nuclear Medicine Scans

h h sh f Zes swallow............................ h f Ze f Zes o o o t t t . .................576 Contrast 572 Contrast enema. h h rig rig rig Copy Copy Copy

ure an Q

shi

FLUOROSCOPY
ure an Q shi

ure an Q

shi

Barium follow-through...................576
Qu shan i resh

Tubograms...........................576

Whathis it e and how is it t of Z g i r y Cop performed?

hi hi ures ures Q Q n n The the shatype of contrast used depends on esha f Ze fZ o o t t examination being performed. Most contrast h h rig rig py Copy Cois material, such as barium, positive contrast it

attenuates/blocks the X-rays signicantly more Fluoroscopy (screening) uses X-rays and contrast than tissues and organs, and therefore appears material to produce images. The patient lies or i dark on uoroscopic imaging and bright on formal h hi hi ures ures ures Q Q Q stands on the screening table. The patient and n n n X-rays. Positive contrast materials can be waterha ha sha Zes fare f Ze f Zessoluble (e.g. o o o X-ray camera positioned and series of low-dose t t t soluble (e.g. gastrografn) or non-water h h h rig yrig pyrigthe X-rays after Copy are taken, often before, during and Co Copthe barium). Barium produces best images but is administration of contrast material. only safe for use inside the gastrointestinal tract. Water-soluble contrast, on the other hand, is safe Two types of image can be taken when screening i for use almost h hi shieverywhere in the body, including ures allows multiple ures (gure 1). Fluoroscopic imaging Q Qure cavity. Water-soluble contrast Q n n n the peritoneal is a a a h sh Zeshin quick succession. They are of images to tbe ftaken f Ze f Zes o o o t t therefore used for examinations where there may h h h g g g ri ri yri use lower but Copy quality than formal X-ray exposures Cop Copy be leakage from the gastrointestinal tract (e.g. if less radiation. The greyscale used is the opposite the patient has a suspected perforated oesophagus) to standard X-rays, with high attenuating material and for tubograms (when contrast is injected into and structures, such as bone, appearing as dark i an iatrogenic tube, such as a surgical drain, urinary hi shi resh ures areas, and low attenuating substances, such as gas, Qure Qu Q n n n catheter or nasogastric tube). a a a h sh Zesh appearingtas areas. Formal X-ray exposures t of Zes oflight of Ze t h h h g g g i i i r yr be acquired. These are like a normal yr can also Air or gas can be used as ap negative contrast Cop Copy X-ray Co in appearance (using the standard grey scale) and material (it attenuates the X-rays much less than provide a more detailed image, but use a higher tissues and organs). A double contrast examination radiation dose and cannot be performed in quick uses both a positive contrast material and air. i hi hi resh ures u ures succession. Usually an combination of these types of Q Q Q n n a a a h esh esh imaging are during uoroscopic studies. f Zes t oused t of Z t of Z

Copy
A

righ

Contrast within cholecystostomy tube

Contrast within the common bile duct

righ Copy

B
Contrast within cholecystostomy tube

Copy

righ

Contrast within the common bile duct

Cop

Cop i resh

Cop resh i
Contrast within the gallbladder

ight opyr

of ZeContrast within the


gallbladder

Qu shan

o right y p o C i

f Ze

Qu shan

ight opyr

of Ze

Qu shan

resh

e Ze fZ of Ze Image This image demonstrates that high material, such as the contrast ht oA: htattenuation ht ofwithin the g g g i i i r r r y y y op op on uoroscopic images and the edges C Cop gallbladder and common bile duct, areC dark of structures, such as the
gallbladder are not particularly sharp.

h sh sh es Figure 1. Examples of uoroscopic and formal X-ray exposure ura Q Qurefrom a cholecystostomy tubogram. Qure shan shan shan

r Copy

Image B: This formal X-ray exposure performed in the same patient shows the usual greyscale we expect in hi how the edges of the gallbladder hi eshi X-rays, with high contrast structures and material appearing white. Notice ur ures ures Q Q Q n n n a ha sha and common bile duct are better dened compared with the uoroscopic image. esh fZ f Ze f Zes

ight o

o right y p o C

r Copy

ight o

Fluoroscopy 571

Fluoroscopy

t of yrigh

Zesh

ure an Q

shi t of yrigh

Zesh

ure an Q

shi t of yrigh

Zesh

ure an Q

shi

CASE INDEX
Orthopaedic X-Rays
Lumbar degenerative changes (case about cauda equina syndrome)............367 Anterior wedge compression fracture............375 Cervical spine fracture.........................................383 Anterior shoulder dislocation............................391 Supracondylar fracture........................................399 Salter Harris fracture............................................407 Scaphoid fracture..................................................415 Colles fracture........................................................423 Hip joint osteoarthritis........................................429 Perthes disease......................................................435 Pelvic fracture.........................................................443 Slipped upper femoral epiphysis......................451 Intracapsular fracture of the neck of femur....................................................................457 Extracapsular fracture of the neck of femur....................................................................465 Paediatric aggressive bone lesion....................473 Lipohaemarthrosis/Tibial plateau fracture/Fibular fracture.....................................481 Knee joint osteoarthritis.....................................489 Tibial and bular shaft fractures......................497 Osteomyelitis/Periosteal reaction...................505 Bimalleolar fracture/Dislocation of the ankle joint.........................................................513

Chest X-Rays
Primary spontaneous pneumothorax.............. 29 Tension pneumothorax......................................... 37 Normal CXR. ............................................................... 45 Collapsed right upper lobe................................... 53 Pleural effusion........................................................ 61 Hilar mass.................................................................. 69 Apical lung mass with rib destruction............. 77 Multiple pulmonary metastases........................ 85 Sarcoidosis................................................................. 91 Ingested foreign body............................................ 97 Left lower lobe consolidation............................105 Possible pulmonary embolism..........................113 Congestive cardiac failure. ..................................121 Free subdiaphragmatic gas................................129 Mediastinal mass..................................................137 Lung abscess and pneumonia...........................145 Left lower lobe collapse and humeral fracture......................................................................153 Incorrectly placed NG tube.................................161 Pericardial effusion...............................................169 Calcied pleural plaques.....................................175

Abdominal X-Rays
Normal AXR.............................................................189 Small bowel obstruction.....................................195 Large bowel obstruction.....................................203 Sigmoid volvulus....................................................211 Pneumoperitoneum.............................................217 Thumbprinting.......................................................225 Pneumatosis intestinalis and pneumoperitoneum.............................................233 Constipation............................................................241 Splenomegaly.........................................................247 Gallstones................................................................255 Renal calculi.............................................................263 Chronic pancreatitis. .............................................269 Uterine broid........................................................277 Splenic artery aneurysm.....................................285 Abdominal aortic aneurysm..............................293 Sclerotic bone metastases..................................299 Ankylosing spondylitis.........................................305 Ingested foreign bodies.......................................313 Duodenal atresia/Double-bubble sign. ..........321 Pneumatosis intestinalis.....................................327

Bonus Chest X-Rays


Apical pneumothorax..........................................579 Normal CXR. .............................................................581 Free subdiaphragmatic gas................................583 Pleural effusion......................................................585 Misplaced NG tube...............................................587 Left lower lobe collapse.......................................589 Pericardial effusion...............................................591 Misplaced NG tube and dilated small bowel loops under the diaphragm........................................................593 Left lower lobe consolidation............................595 Right middle lobe consolidation. ......................597 Right upper lobe collapse...................................599 Right upper lobe consolidation/Basal atelectasis................................................................601 Right lower lobe consolidation.........................603 Free subdiaphragmatic gas................................605 Lingula consolidation...........................................607 Left upper lobe collapse. ......................................609 Pneumomediastinum..........................................611

Bonus Abdominal X-Rays


Gallstones................................................................613 Migrated IUCD........................................................615 Large bowel obstruction.....................................617 Normal AXR.............................................................619 Small bowel obstruction/ Pneumoperitoneum.............................................621 Thumbprinting/Toxic dilatation of the transverse colon.............................................623 Bilateral nephrostomies......................................625 Thumbprinting.......................................................627 Necrotising enterocolitis.....................................629 Pagets disease........................................................631 Splenomegaly.........................................................633 Intra-abdominal surgical swab/ Small bowel obstruction.....................................635

Bonus Orthopaedic X-Rays


5th metacarpal fracture......................................637 Normal foot X-ray..................................................639 Healed fractures (rib/clavicle/coracoid)/ Pneumothorax........................................................641 Normal elbow X-ray..............................................643 Slipped upper femoral epiphysis......................645 Lisfranc injury..........................................................647 Distal radial (with dorsal angulation) fracture......................................................................649 Weber C ankle fracture........................................651 5th metatarsal fracture.......................................653 Lipohaemarthosis..................................................655 Anterior shoulder dislocation............................657 Comminuted mid shaft femoral fracture......659 Comminuted humeral surgical neck fracture......................................................................661 Monteggia fracture...............................................663 Posterior shoulder dislocation..........................665 Rheumatoid arthritis............................................667 Distal radial (minimally displaced) fracture......................................................................669 Pathological shaft of femur fracture...............671 Acromio-clavicular joint disruption................673 5th metatarsal fracture.......................................675 Enchondroma (lytic bone lesion). .....................677 Distal radial (buckle) fracture............................679 Intracapsular fracture of the femoral neck.... 681 Triquetral fracture.................................................683 Weber A ankle fracture........................................685 Radial head fracture.............................................687 Trimalleolar fracture.............................................689 Extracapsular (subtrochanteric) fracture of the femoral neck...............................................691 Supracondylar fracture........................................693

Case Index 695

INDEX
Page numbers followed by f indicate gures.
AAA, see Abdominal aortic aneurysm ABCDD mnemonic, chest X-ray assessment airway, 1920 breathing, 20 cardiac and mediastinum, 20, 21f delicates, 21 diaphragm, 2021, 22f ABCDE assessment, pelvic fracture, 446 ABCDEF mnemonic, pulmonary oedema chest X-ray assessment, 25 Abdominal aorta, abdominal X-ray assessment, 185f Abdominal aortic aneurysm (AAA) abdominal X-ray, 293f, 294f causes, 296 clinical ndings, 296 computed tomography angiography, . 542f diameter, 297 management, 297 rupture imaging, 298 Abdominal computed tomography colonoscopy, 539f haematuria, 541f haemorrhage, 537f indications, 535 renal tract calculi, 540f sepsis, 536f trauma, 535f tumour staging, 538f Abdominal ultrasound, 559f, 560f Abdominal X-ray adequacy, 181 case examples abdominal aortic aneurysm, 293f, . 294f, 295298 ankylosing spondylitis, 305f, 306f, 307312 bone metastasis, 299f, 300f, 301304 bowel ischaemia, 233f, 234f, 235240, 627f, 628f colitis, 225f, 226f, 227232 constipation, 241f, 242f, 243246 duodenal atresia, 321f, 322f, 323326 foreign body ingestion, 313f, 314f, 315320 gallstones, 255f, 256f, 257262, 613f, 614f inammatory bowel disease, 623f, 624f intrauterine contraceptive device migration, 615f, 616f large bowel obstruction, 203f, 204f, 205210, 617f, 618f necrotising enterocolitis, 327f, 328f, 331332, 629f, 630f nephrostomy complications, 625f, 626f Pagets disease, 631f, 632f pancreatitis, 269f, 270f, 271276 pneumoperitoneum, 217f, 218f, 219224, 234f, 621f, 622f renal calculi, 263f, 264f, 265268 sigmoid colon volvulus, 211f, 212f, 213216 small bowel obstruction, 195f, 196f, 197202, 635f, 636f splenic artery aneurysm, 285f, 286f, 287292 splenomegaly, 247f, 248f, 249254, 633f, 634f uterine broid, 277f, 278f, 279284 checklist, 186 dose, 192193, 523 indications, 192 normal ndings, 189f, 190f, 191194, 619f, 620f projection, 181 small/large bowel differences, 193 systematic review, 181185 Abscess, see Lung abscess; Pancreatic abscess Achilles tendon, rupture, 517 Acute pancreatitis causes, 273 complications, 276 management, 275 pleural effusion induction, 66 ultrasound for gallstones, 274 Adhesions, small bowel obstruction, 200 Advance Trauma Life Support (ATLS), 446447, 478 AIN, see Anterior interosseous nerve Alcoholism acute pancreatitis, 273 chronic pancreatitis, 272 lung abscess risk factor, 149 Allergy, contrast agents, 523 AMPLE assessment, 447 Anal ssure, constipation, 245 Anaphylaxis, breathlessness, 34 Anatomical snuff box, pain and scaphoid fracture, 418 Ankle medial clear space, 518 orthopaedic X-ray assessment, 362f, 363f fracture with talar shift, 651f, 652f malleolar fracture case examples, 513f, 514f, 515520, 685f, 686f, 689f, 690f Ottawa Ankle Rules, 516 Weber classication of lateral malleolus fractures, 363, 517 talar shift, 518519 triplane fracture, 414 Ankle jerk, spinal injury assessment, 380 Ankylosing spondylitis abdominal X-ray, 305f, 306f, 309 clinical presentation, 308 extra-spinal joint involvement, 310 hepatic brosis, 311 treatment, 311312 Anterior interosseous nerve (AIN) injury following supracondylar fracture, 404 testing, 427 Aortic dissection, chest computed tomography, 534f Appendicitis, foreign body ingestion, 430 Asbestos exposure, case example with chest X-ray of pleural plaques, 175f, 176f, 177180 Ascites, portal hypertension, 290 Aspirin, pericardial effusion management, 174 Atherosclerosis, abdominal aortic aneurysm, 296 ATLS, see Advance Trauma Life Support Avascular necrosis hip, see Perthes disease scaphoid, 419 Axillary nerve, injury in shoulder dislocation, 396 Babinski sign, 304 Back pain management, 374 red ags, 308 Bamboo spine, ankylosing spondylitis, 309 Barium follow-through, 576 Barium swallow constipation induction, 246 principles, 572 small bowel obstruction diagnosis, 202 views, 573f, 574f, 575f Bats wing consolidation, 122f Bell classication, necrotising enterocolitis, 332 Bennetts fracture, 364 Biliary colic, gallstone presentation, 259 Blood culture, necrotising enterocolitis, 331 Blood gas, venous central line, 167168 Bone metastasis abdominal X-ray, 299f, 300f causes, 302 clinical ndings, 302 spinal cord compression management, 304

696 Index

INDEX
Bone scan principles, 565, 568f, 569 prostate cancer staging, 302303 radiation dose, 566 Bony Bankart lesion, 397 Bowel ischaemia abdominal X-ray, 233f, 234f, 235240 acute versus chronic, 240 case examples with abdominal X-ray, 233f, 234f, 235240, 627f, 628f computed tomography, 237, 239 management of acute disease, 237 pneumatosis intestinalis, 236, 238 Bowel obstruction abdominal X-ray assessment, 182, 183f case examples with abdominal X-rays large bowel obstruction, 203f, 204f, 205210, 617f, 618f small bowel obstruction, 195f, 196f, 197202, 621f, 622f, 635f, 636f central loops in small bowel obstruction, 198 foreign body ingestion, 320 pseudo-obstruction versus mechanical large bowel obstruction, 206 small/large bowel differences on X-ray, 193 stenting of large bowel Breast cancer lobar lung collapse, 156 sclerotic bone metastasis, 302 Breathlessness, differential diagnosis, 34 Bronchiectasis, pneumonia complication, 111112 Bronchoscopy, foreign body retrieval, 104 Burst fracture, 364 placement verication, 167168 Chance fracture, 364, 379 Chest computed tomography, see also Computed tomography pulmonary angiography aortic dissection, 534f indications, 531 lobar lung collapse, 532 lung parenchyma assessment, 532f pulmonary nodule, 531f trauma, 533f Chest drain Seldinger technique, 68 tension pneumothorax, 44 Chest ultrasound, 558, 559f Chest X-ray abnormality assessment, 19 calcication causes, 180 case examples asbestos exposure and pleural plaques, 175f, 176f, 177180 foreign body ingestion, 97f, 98f, 99104 lobar collapse right upper lobe collapse, 53, 54f, 5560, 599f, 600f left lower lobe collapse, 153f, 154f, 155160, 589f, 590f lung abscess, 145f, 146f, 147152 lung cancer hilar mass, 69f, 70f, 7176 metastasis sites, 7475 Pancoast tumour, 77f, 78f, 7984 pulmonary metastases, 85f, 86f, 8790 mediastinal mass, 137f, 138f, 139141 normal ndings, 46f, 4752 Caecum, volvulus, 213214, 216 pericardial effusion, 169f, 170f, CAP, see Community acquired pneumonia 591f, 592f Cardiothoracic ratio pleural effusion, 61f, 62f, 6368, congestive cardiac failure, 125 585f, 586f measurement, 5051 pneumomediastinum, 611f, 612f Carotid artery, Doppler ultrasound, 558, pneumonia, 105f, 106f, 107112, 559f 595f, 596f, 597f, 598f, 601f, 602f, Carpal tunnel, anatomy, 422 603f, 604f, 607f, 608f Cauda equina syndrome pneumoperitoneum, 129f, 130f, clinical features, 370 131136, 583 f , 584 f decompression surgery, 373 pneumothorax lumbar spine X-ray, 367f, 368f primary spontaneous, 29f, 30f, magnetic resonance imaging, 372, 3136 550f small left apical pneumothorax, physical examination, 371 579f, 580f Cavitating tumour, lung, 148 tension pneumothorax, 37f, 38f, Central cord syndrome, 382 3944 Central line pulmonary embolism, 113f, 114f, chest X-ray, 22f 115120, 581f, 582f insertion, 166167 pulmonary oedema, 121f, 122f, 123128 sarcoidosis of lung, 91f, 92f, 9396 tube and line displacement, 161f, 162f, 163168, 587f, 588f, 593f, 594f checklist, 28 consolidation causes, 108, 117, 122f, 162f dose, 4849 indications, 48 inspiration during, 1819 lines and tubes, 22f lung collapse, see Lobar lung collapse lymphadenopathy, 94 patient details, 18 penetration assessment, 19 pleural effusion, 25f pneumonia, 23, 24f, 25 pneumothorax, 25f projection, 17f, 18 pulmonary oedema, 25f review ABCDD mnemonic airway, 1920 breathing, 20 cardiac and mediastinum, 20, 21f delicates, 21 diaphragm, 2021, 22f summarisation, 23 rotation, 18f Cholecystitis, gallstone presentation, 259 Chronic myeloid leukaemia, splenomegaly, 252 Chronic obstructive pulmonary disease (COPD), 20, 603f, 604f Chronic pancreatitis, abdominal X-ray, 269f, 270f Clavicle, see Shoulder Clips, see Sterilisation clips Cocksackie virus, pericardial effusion induction, 172 Coffee bean sign, sigmoid colon volvulus, 215 Colchicine, pericardial effusion management, 174 Colitis case example with abdominal X-ray, 225f, 226f, 227232 causes, 228 ulcerative colitis, see Inammatory bowel disease Colles fracture, 364, 423f, 424f, 425428 Colon cancer colonoscopy, 209 large bowel obstruction, 207 lobar lung collapse, 156 Colon, see Large bowel

Index 697

INDEX
Colonoscopy, 209, 539f Community acquired pneumonia (CAP) case examples, 595f, 596f, 601f, 602f pathogens, 110 scoring, 111 Compartment syndrome, 402, 486, 503 Computed tomography (CT) abdominal aortic aneurysm rupture imaging, 298 abdominal CT, 535f, 536f, 537f, 538f, 539f, 540f, 541f angiography, 542f bone tumour, 479 bowel ischaemia, 237, 239 cancer staging, 159 cervical spine, 344f, 388389, 530f chest CT, 531f, 532f, 533f, 534f contraindications, 524 contrast agents allergy, 523 metformin patient precautions, 524 nephropathy induction, 523524 types, 523 dose, 523 foreign body ingestion, 101 head CT, 525, 526f, 527f, 528f, 529f indications, 524 lung abscess, 150151 lung cancer, 75 orthopaedic CT, 530f, 531 pancreatic abscess, 276 pelvis fracture, 446 PET/CT, see Positron emission tomography pleural calcication, 179 pneumoperitoneum, 132133, 223224 principles, 521f pulmonary metastases, 90 scanner, 521f sigmoid colon volvulus, 216 tibial plateau fracture, 488 windowing, 522f, 523 Computed tomography pulmonary angiography (CTPA) pleural effusion, 67 pulmonary embolism, 531f Congestive cardiac failure assessment, 117 chest X-ray ndings, 114115 clinical features, 114 Constipation ankylosing spondylitis, 306f bone metastasis, 302 causes, 244245 complications, 246 imaging, 241f, 242f, 245246 management, 245 tenesmus, 244 Continuous positive airway pressure (CPAP), pulmonary oedema management, 128 Contrast agents computed tomography allergy, 523 metformin patient precautions, 524 nephropathy induction, 523524 types, 523 gadolinium for magnetic resonance imaging, 544 Contrast enema, 576f Contrast swallow, see Barium swallow COPD, see Chronic obstructive pulmonary disease Costophrenic angle, blunting, 30f, 62f CPAP, see Continuous positive airway pressure Cremasteric reex, spinal injury assessment, 380 CRITOE acronym, ossication centres, 351, 353f Crohns disease, see Inammatory bowel disease CTPA, see Computed tomography pulmonary angiography Cubitus varus, supracondylar fracture, 406 CURB 65, pneumonia scoring, 111 Danozol, uterine broid management, 283 Deep vein thrombosis (DVT) complications, 119120 pneumonia complication, 111112 prophylaxis in ulcerative colitis are management, 232 ultrasound, 118 Demyelination, magnetic resonance imaging, 547f Dermatome, spinal injury assessment, 380, 389390 Diarrhoea, bloody diarrhoea assessment, 229 Dislocation, 339, 340f Disseminated intravascular coagulation, necrotising enterocolitis, 334 Diverticulitis, stricture formation, 208 Docusate sodium, constipation management, 245 Double bubble sign, 322f Downs syndrome, duodenal atresia, 325 Duodenal atresia abdominal X-ray, 321f, 322f, 325 associated conditions, 325 case example with abdominal X-ray, 321f, 322f, 323326 double bubble sign, 322f management, 326 signs and symptoms, 324 stenosis similarity, 324 Duodenum, volvulus, 213 DVT, see Deep vein thrombosis Dysmenorrhoea, uterine broids, 281 ECG, see Electrocardiogram Elbow anatomy, 405 anterior interosseous nerve injury, 404 effusion X-ray, 337f, 687f, 688f, 693f, 694f orthopaedic X-ray assessment, 350, 351f, 352f, 353f, 403 normal ndings, 643f, 644f radial head fracture, 687f, 688f supracondylar fracture case examples, 399f, 400f, 401406, 693f, 694f deformities following, 405406 Gartland classication, 353, 403 ossication centres, 351, 353f Electrocardiogram (ECG), congestive cardiac failure, 127 Emergency surgery, prerequisites, 463464 Emphysema, lung abscess risk factor, 149 Empyema pleural calcication, 178 pneumonia complication, 111112 Enchondroma, location and age, 480 Endoscopic retrograde cholangiopancreatography (ERCP), acute pancreatitis, 273 Endoscopy battery stuck in mid-oesophagus, 319 complications, 320 Endotracheal tube (ET) chest X-ray, 22f positioning, 166 Enema, sigmoid colon volvulus management, 216 Enterocolic stulae, necrotising enterocolitis, 334 Epilepsy, magnetic resonance imaging, 548 ERCP, see Endoscopic retrograde cholangiopancreatography ET, see Endotracheal tube Ewings sarcoma differential diagnosis, 477 location and age, 480 Faecal loading, see Constipation FAST, see Focused assessment with sonography in trauma Femoral vein, ultrasound, 563f Femur neck fracture, see Hip shaft fracture X-rays, 339f, 659f, 660f, 671f, 672f

698 Index

INDEX
Fibula, see also Knee shaft fracture case example, 497f, 498f, 499504 Finger, X-ray bone lesion in proximal phalanx, 677f, 678f lucent lesion, 338f subluxation versus dislocation, 339, 340f views, 335f Fish bone, X-ray ndings of ingestion, 103 Fistula, Crohns disease, 230 FLAIR, see Fluid attenuation recovery Fluid attenuation recovery (FLAIR), 547f Fluid resuscitation, ulcerative colitis are management, 232 Fluid therapy duodenal atresia, 26 necrotising enterocolitis, 332333 Fluoroscopy barium follow-through, 576 barium swallow, see also Barium swallow principles, 572 views, 573f, 574f, 575f contraindications, 572 contrast enema, 576f indications, 572 principles, 571 radiation dose, 572 tubogram, 576, 577f Focused assessment with sonography in trauma (FAST), 448, 563 Foot, metatarsal fracture, 639f, 640f, 647f, 648f, 653f, 654f, 675f, 676f Foreign body ingestion abdominal X-ray, 313f, 314f age groups, 316 battery stuck in mid-oesophagus, 319 case examples abdominal X-ray, 313f, 314f, 315320 chest X-ray, 97f, 98f, 99104 oropharynx trapping, 317 physical examination, 318 pneumoperitoneum induction, 135 Foreign body inhalation, lobar lung collapse, 157 Functional ileus, small bowel obstruction differentiation, 199 Gadolinium, precautions, 544 Galeazzi fracture, 364 Gallbladder, abdominal X-ray assessment, 185 Gallstones acute pancreatitis, 273 case examples with abdominal X-ray, 255f, 256f, 257262, 613f, 614f cholecystitis management, 262 clinical presentation, 259 risk factors, 258 ultrasound, 274, 560f, 561f Garden classication, hip fractures, 359, 461 Garlands triad (1, 2, 3 sign), 96 Gartland classication, supracondylar fracture, 353, 403 Gas, see also Pneumoperitoneum; Pneumothorax pneumatosis intestinalis, 236, 238 subdiaphragmatic, 22f, 129f, 130f, 131136, 605f, 606f Gastric tube, duodenal atresia, 326 Genitourinary scan principles, 565 radiation dose, 566 renal cortex assessment, 567568 Glenohumeral joint, see Shoulder Globular heart, pericardial effusion chest X-ray, 174 Glycerol suppository, constipation management, 245 Glyceryl trinitrate (GTN), pulmonary oedema management, 128 Goitre mediastinal mass differential diagnosis, 142 ultrasound, 558f Golden S sign, 26f, 54f, 59 Growth plate fracture growth disturbances, 412 management, 413 Salter Harris classication, 364, 408f, 411412 physis components, 410 GTN, see Glyceryl trinitrate Gunstock deformity, supracondylar fracture, 406 Gustilo classication, open fractures, 504 haemorrhage, 525, 526f, 527 hydrocephalus, 526f, 528 indications, 525, 527 mass effect, 527 space-occupying lesions, 527f Head magnetic resonance imaging congenital abnormalities, 549 demyelination, 547f epilepsy, 548 indications, 544 pituitary, 548 space occupying lesions, 548f stroke, 546f vascular abnormalities, 549 weighted images, 545f Heparin-induced thrombocytopaenia (HIP), 119 Hepatic brosis, ankylosing spondylitis, 311 Hereditary spherocytosis, splenectomy, 253 Hilar mass, case example, 69f, 70f, 7176 Hill Sachs lesion, 365, 397 Hilum overlay sign, mediastinal mass, 142 HIP, see Heparin-induced thrombocytopaenia Hip ankylosing spondylitis, 310311 avascular necrosis case example, 435f, 436f, 437442 femur neck fracture case examples extracapsular fracture, 465f, 466f, 691f, 692f intracapsular fracture, 457f, 458f, 459464, 681f, 682f Garden classication, 359, 461 management, 462463, 470471 mechanism of injury, 468 mortality, 471472 overview, 462 2 to 4 parts classication, 360, 470 indications, 433434 orthopaedic X-ray assessment, 357f, 358f, 359f, 360 Haematuria, abdominal computed osteoarthritis tomography, 541f case example, 336f, 429f, 430f, Haemolytic anaemia, gallstone risks, 258 431434 Haemoptysis, differential diagnosis, 72 X-ray, 336f Haemorrhoids, constipation association, parts classication of neck of femur 246 fractures, 360 Haemothorax, pleural calcication, 18 Perthes disease, 442 Hamptons hump, 117 septic arthritis distinguishing from Hand, see also Finger irritable hip, 512 erosive arthropathy, 667f, 668f slipped upper femoral epiphysis case metacarpal fracture, 637f, 638f examples, 451f, 452f, 453456, 645f, Hangmans fracture, 364 646f Hartmanns procedure, 210 surgical approaches, 434 Head computed tomography Trendelenburg sign, 432433 contrast enhanced imaging, 528, 529f Hippocratic method, shoulder dislocation grey-white matter differentiation, 528f reduction, 395

Index 699

INDEX
Holstein Lewis fracture, 365 Hormone replacement therapy (HRT), pulmonary embolism risks, 116 Horners syndrome, lung cancer, 8081 Hounsfeld unit (HU), 522 HPOA, see Hypertrophic pulmonary osteoarthropathy HRT, see Hormone replacement therapy HU, see Hounsfeld unit Hydronephrosis, abdominal aortic aneurysm, 296 Hypercalcaemia constipation, 244 renal calculi, 266 Hypertension, splenic artery aneurysm risks, 290 Hypertrophic pulmonary osteoarthropathy (HPOA), lung cancer, 74 Hypotension pericardial effusion, 173 tension pneumothorax, 41 Hypoxia, tension pneumothorax, 41 Hysterectomy, uterine broid management, 284 right upper lobe, 26, 53, 54f, 5560, 599f, 600f clinical presentation, 56 computed tomography, 532 foreign body induction, 102 Low-molecular-weight heparin, pulmonary embolism management, 119 Luftsichel sign, 26f Lumbar spine, see Cauda equina syndrome; Spine Lung parenchyma assessment with chest computed tomography, 532f X-ray, see Chest X-ray Lung abscess aetiology, 150151 chest X-ray ndings, 145f, 146f differential diagnosis, 148 risk factors, 149 treatment, 151152 Lactulose, constipation management, 245 Lung cancer Large bowel case examples with chest X-rays abdominal X-ray assessment, 182f hilar mass, 69f, 70f, 7176 abscess, 334 metastasis sites, 7475 constipation and perforation, 246 Pancoast tumour, 77f, 78f, 7984 Idiopathic thrombocytopaenic purpura, obstruction, 182, 183f, 617f, 618f pulmonary metastases, 85f, 86f, Splenectomy, 253 obstruction, see Bowel obstruction 8790 Inammatory bowel disease sigmoid colon volvulus, 211f, 212f, consolidation on chest X-ray, 108 abdominal X-ray ndings, 623f, 624f 213216 lobar lung collapse, 156 Crohns disease versus ulcerative small bowel differences on X-ray, 193 Lung collapse, see Lobar lung collapse colitis, 230 thumbprinting, 185f, 185 Lung ventilation/perfusion scan ares Laryngeal cancer, asbestos exposure, 179 principles, 565 toxic megacolon assessment, 231 Left lower lobe collapse, 26f, 153f, 154f, pulmonary embolism, 566, 567f ulcerative colitis management, 232 155160 radiation dose, 566 gallstone risks with Crohns disease, 258 Left upper lobe collapse, 26, 58, 609f, 610f Lymphadenopathy pneumoperitoneum induction, 135 Levonorgestrel intrauterine device, chest X-ray ndings, 94 renal calculi, 266 uterine broid management, 283 sarcoidosis of lung, 91f, 92f, 94 stricture formation, 208 Lipohaemarthrosis Lymph node, calcication in upper left Iniximab, ankylosing spondylitis fracture association, 484 quadrant of abdominal X-ray, 288 management, 312 tibial plateau fracture case studies, Lymphoma Interosseus nerve, testing, 427 481f, 482f, 483488, 655f, 656f hilar mass, 73 Interventional radiology, bowel Lisfranc injury, 365 mediastinal mass differential ischaemia, 237 Liver diagnosis, 142143 Intra-aortic balloon pump, pulmonary abdominal X-ray assessment, 185 splenomegaly, 252253 oedema management, 128 ultrasound, 560f, 561f Intrauterine contraceptive device (IUCD), Liver transplantation, splenic artery Magnetic resonance X-ray of migration, 615f, 616f aneurysm risks, 290 cholangiopancreatography (MRCP), 261, IUCD, see Intrauterine contraceptive Lobar lung collapse 553f device cancer association, 156 Magnetic resonance imaging (MRI) causes, 57 bone tumour of knee, 479 Jaundice, gallstone presentation, 259 chest physiotherapy, 60 cauda equina syndrome, 372 Jefferson fracture, 365, 388 chest X-ray cervical spine, 388389 Jones fracture, 365 left lower lobe, 26f, 153f, 154f, contraindications, 544 155160, 589f, 590f gadolinium contrast agent, 544 Kerley B lines, 25f left upper lobe, 26, 58, 609f, 610f head MRI Kidney right lower lobe, 27f congenital abnormalities, 549 abdominal X-ray assessment, 185 right middle lobe, 27f demyelination, 547f failure, see Renal failure stones, see Renal calculi ultrasound, 560, 561f Kleins line, 366 Knee aspiration, 487 bone tumour case study, 473f, 474f, 475480 magnetic resonance imaging, 555f orthopaedic X-ray assessment, 360, 361f osteoarthritis case example, 489f, 490f, 491496 tibial plateau fracture case studies, 481f, 482f, 483488, 655f, 656f X-ray features, 494495 Knee replacement, 496 Kochers method, shoulder dislocation reduction, 395

700 Index

INDEX
epilepsy, 548 indications, 544 pituitary, 548 space occupying lesions, 548f stroke, 546f vascular abnormalities, 549 weighted images, 545f indications, 544 knee, 555f osteomyelitis, 510 Pancoast tumour, 83 principles, 543 safety, 543 scanner, 543f small bowel, 554f spinal cord compression, 303 spine MRI cauda equina syndrome, 550f indications, 549 normal ndings, 549f spinal canal stenosis, 552f spinal cord compression, 550, 551f spinal cord trauma, 553f uterine broids, 282 weighted sequences, 544 Maissonneuve injury, 365 Malaria, splenomegaly, 252 Malleolus, see Ankle Malrotation, duodenal atresia, 325 Medial clear space, 518 Median nerve, injury in Colles fracture, 426427 Mediastinum chest X-ray assessment, 20, 21f, 140 gas, see Pneumomediastinum mass, case example with chest X-ray, 137f, 138f, 139141 mediastinitis and oesophagus foreign body induction, 100 Menorrhagia, uterine broids, 281 Mesothelioma, asbestos exposure, 179 Metacarpal, fracture, 637f, 638f Metal detector, ingested coin detection, 100 Metastasis, lung cancer, 7475 Metatarsal, fracture, 639f, 640f, 647f, 648f, 653f, 654f, 675f, 676f Mitral regurgitation calcication on X-ray, 180 pulmonary oedema, 126 Metformin, contrast agent precautions, 524 Monteggia fracture, 365 MRCP, see Magnetic resonance cholangiopancreatography MRI, see Magnetic resonance imaging Mucus plug, lobar lung collapse, 157 Multiple myeloma, 158, 480 Myasthenia, lung cancer, 74 Myelobrosis, splenomegaly, 252 Myocardial infarction, pericardial effusion induction, 172 Myocardial perfusion scan principles, 565, 567 radiation dose, 566 Myomectomy, uterine broid management, 284 Myotome, spinal injury assessment, 380, 390 fracture appearance, 336f descriptors, 338339 names, 364366 hip assessment, 357f, 358f, 359f, 360 knee assessment, 360, 361f pelvis assessment, 357f, 358f, 359f, 360 projection, 335f shoulder assessment, 348, 349f, 350f spine cervical spine assessment, 340341, Nasendoscopy, foreign body retrieval, 104 342f, 343f, 344f, 345f Nasogastric (NG) tube thoracolumbar spine assessment, acute pancreatitis, 275 345, 346f, 347f chest X-rays, 22f, 161f, 162f, 165, 587f, subluxation versus dislocation, 339, 340f 588f, 593f, 594f systematic review, 336f, 337f, 338f placement, 164 technical adequacy, 336 small bowel obstruction wrist assessment, 354f, 355f, 356f management, 201 Ossication centres, elbow, 351, 353f Necrotising enterocolitis Osteoarthritis abdominal X-ray, 327f, 328f, 331332, hip case example, 336f, 429f, 430f, 629f, 630f 431434 Bell classication system, 332 knee case example, 489f, 490f, 491496 complications, 333334 severity assessment, 433 management, 332333 X-ray features, 432 risk factors, 330 Osteomyelitis stricture formation, 208 clinical ndings, 508 Needle decompression, tension differential diagnosis, 477 pneumothorax, 38, 42, 44 magnetic resonance imaging, 510 Nephrostomy, complications, 625f, 626f management, 511 NG tube, see Nasogastric tube pathogens, 509 Nuclear medicine, see Bone scan; tibia case example, 505f, 506f, Genitourinary scan; Lung ventilation/ 507512 perfusion scan; Myocardial perfusion Osteophyte, knee arthritis, 494496 scan; Positron emission tomography Osteosarcoma location and age, 480 Obesity, gallstone risks, 258 pathological fracture, 158 Oesophageal varices, portal hypertension Ottawa Ankle Rules, 516 as cause, 291 Ovarian cyst, ultrasound, 562f Oesophagus battery stuck in mid-oesophagus, 319 stula from foreign body ingestion, 320 Pagets disease, X-ray ndings, 631f, 632f Pancoast tumour, see Lung cancer perforation from foreign body Pancreas ingestion, 320 abdominal X-ray assessment, 185 Open fracture annular pancreas, 324325 Gustilo classication, 504 ultrasound, 560 management, 501502 Pancreatic abscess, acute pancreatitis Opioids, constipation, 244 association, 276 Orthopaedic computed tomography, Pancreatitis, see Acute pancreatitis; 530531 Chronic pancreatitis Orthopaedic X-rays, see also specic Patella reex, spinal injury assessment, 380 bones and joints Pelvic ultrasound, 562f ankle assessment, 362f, 363f Pelvis checklist, 366 fracture elbow bladder and urethral injuries, 449 assessment, 350, 351f, 352f, 353f case example, 443f, 444f, 445450 Gartland classication of computed tomography, 446 supracondylar fracture, 353

Index 701

INDEX
management, 450 mortality, 449 types, 448449 orthopaedic X-ray assessment, 357f, 358f, 359f, 360 tumour staging with computed tomography, 538f Pericardial effusion, case examples with chest X-ray, 169f, 170f, 591f, 592f Periosteal reaction aggressive bone lesion, 476 osteomyelitis, 505f, 506f Peritonitis, pneumoperitoneum induction, 136 Perthes disease differential diagnosis, 440 severity assessment, 441 treatment, 441442 X-ray case example, 435f, 436f, 437442 PET, see Positron emission tomography Phlebolith, 278f Physis, see Growth plate Pituitary, magnetic resonance imaging, 548 Pleural calcication causes, 178 imaging, 177f, 179 Pleural effusion case examples with chest X-ray, 61f, 62f, 6368, 585f, 586f chest X-ray ndings, 25f clinical signs and symptoms, 64 congestive cardiac failure, 125 draining, 68 exudate analysis, 6465 exudative effusion aetiology, 6566 pneumonia complication, 111112 transudate versus exudate effusions, 66 ultrasound, 559f Pneumatosis intestinalis, differential diagnosis, 236, 238 Pneumomediastinum, case example, 611f, 612f Pneumonia case examples with chest X-rays, 105f, 106f, 107112, 595f, 596f, 597, 598f, 601f, 602f, 603f, 604f, 607f, 608f chest X-ray ndings, 23, 24f, 25 community acquired pneumonia case examples, 595f, 596f, 601f, 602f pathogens, 110 scoring, 111 complications, 111112 consolidation on chest X-ray, 108109 pleural effusion induction, 65 Pneumoperitoneum abdominal X-ray assessment, 184f, 185f case examples with abdominal X-ray, 217f, 218f, 219224, 234f, 583f, 584f, 621f, 622f causes, 134135, 220 chest X-ray ndings, 129f, 130f, 131136 post-operative, 136 Pneumothorax breathlessness, 34 case examples with chest X-rays primary spontaneous, 29f, 30f, 3136 small left apical pneumothorax, 579f, 580f tension pneumothorax, 37f, 38f, 3944 chest X-ray ndings, 25f evolution from simple pneumothorax, 40 pneumonia complication, 111112 shoulder X-ray, 641f, 642f tension pneumothorax, 20 Portal hypertension, 290291, 560 Positron emission tomography (PET) indications, 420 PET/CT principles, 565 radiation dose, 566 tumours, 569f Posterior interosseus nerve, testing, 427 Pregnancy pulmonary embolism evaluation, 118 splenic artery aneurysm risks, 290 X-ray safety, 14 Premature infants, necrotising enterocolitis, 330 Prostate cancer bone metastasis, 299f, 300f, 301304 staging, 302303 Pulmonary artery hypertension hilar mass, 73 pulmonary embolism complication, 119120 Pulmonary embolism breathlessness, 34 case examples with chest X-ray, 113f, . 114f, 115120, 581f, 582f complications, 119120 lung ventilation/perfusion scan, 566, 567f pregnant patient evaluation, 118 risk factors, 116 treatment, 119 Pulmonary brosis, 96, 179 Pulmonary oedema case example with chest X-ray, 121f, 122f, 123128 causes, 126 chest X-ray ndings, 25f, 108 treatment, 128 Pulmory infarction haemoptysis, 72 lung abscess differential diagnosis, 148 pleural effusion induction, 66 Radial inclination, 355 Radial nerve, testing, 427 Radius, see Elbow; Wrist Renal artery stenosis, pulmonary oedema, 126 Renal calculi abdominal X-ray, 263f, 264f calcication in upper left quadrant of abdominal X-ray, 288 computed tomography, 267268, 540f management, 268 risk factors, 266 symptoms, 266 uric acid stones, 267 Renal failure contrast agent nephropathy induction, 523524 pericardial effusion induction, 172 pulmonary oedema, 126 Renal tubular acidosis, renal calculi, 266 Retroperitoneal brosis, abdominal aortic aneurysm, 296 Rheumatoid arthritis, hand X-ray, 667f, 668f Right lower lobe collapse, 27f Right middle lobe collapse, 27f Right upper lobe collapse, 26, 53, 54f, 5560, 599f, 600f Riglers sign, 234f Rolandos fracture, 365 Round atelectasis, asbestos exposure, 179 Sacroiliac joint, abdominal X-ray, 194 Saddle anaesthesia, cauda equina syndrome, 371 Sail sign, 26, 27f, 154f Salter Harris classication, growth plate fractures, 364, 408f, 411412 Sarcoidosis clinical presentation, 9596 differential diagnosis in lung, 78 hilar mass, 73 lung, 91f, 92f, 9396 risk factors, 9495 Scaphoid, see Wrist SCIWORA, see Spinal cord injury without radiological abnormality Segond fracture, 365 Seldinger technique, 68 Senna, constipation management, 245 Septic arthritis, distinguishing from irritable hip, 512

702 Index

INDEX
Shentons line, 366 Short bowel syndrome, necrotising enterocolitis, 334 Shoulder ankylosing spondylitis, 310 dislocation axillary nerve injury, 396 bony abnormalities with anterior dislocations, 397 instability following injury, 397398 reduction, 395 types, 394 orthopaedic X-ray acromio-clavicular joint disruption, 673f, 674f assessment, 348, 349f, 350f glenohumeral joint dislocation, 391f, 392f, 393398, 657f, 658f humeral neck fracture, 661f, 662f posterior dislocation, 665f, 666f SiADH, see Syndrome of inappropriate antidiuretic hormone secretion Sickle cell anaemia, calcication in upper left quadrant of abdominal X-ray, 288 Sigmoid colon volvulus abdominal X-ray, 211f, 212f, 213216 contrast enema, 576f Sigmoidoscopy, sigmoid colon volvulus management, 216 Silhouette sign mediastinal mass, 142 pneumonia chest X-ray, 24f, 109 Silicosis, calcication on X-ray, 180 Slipped upper femoral epiphysis (SUFE) case examples, 451f, 452f, 453456, 645f, 646f clinical presentation, 454 contralateral slippage incidence, 455 management, 456 pathology, 454 Trethowans sign, 455 Small bowel abdominal X-ray assessment, 182f barium follow-through, 576 dilation in necrotising enterocolitis, 331332 large bowel differences on X-ray, 193 magnetic resonance imaging, 554f obstruction, see Bowel obstruction ultrasound, 561f Smiths fracture, 365, 427428 Spinal canal stenosis, magnetic resonance imaging, 552f Spinal cord compression bone metastasis, 302304 cauda equina syndrome decompression surgery, 373 magnetic resonance imaging, 550, 551f Spinal cord injury without radiological abnormality (SCIWORA), 384 Spinal cord trauma, magnetic resonance imaging, 553f Spinal shock, 381 Spine, see also Cauda equina syndrome back pain management, 374 cauda equina syndrome decompression surgery, 373 cervical spine computed tomography, 530f computed tomography of cervical spine, 388389 magnetic resonance imaging of cervical spine, 388389 orthopaedic X-ray central cord syndrome, 382 cervical spine assessment, 340341, 342f, 343f, 344f, 345f case example of fracture without trauma, 382f, 383f, 384390 lines for lateral X-ray assessment, 387 Chance fracture-associated injuries, 379 Jefferson fracture, 388 L1 fracture, 375f, 376f level of sensation and fracture location, 380, 389390 stable fracture, 376 thoracolumbar spine assessment, 345, 346f, 347f Spine magnetic resonance imaging cauda equina syndrome, 550f indications, 549 normal ndings, 549f spinal canal stenosis, 552f spinal cord compression, 550, 551f spinal cord trauma, 553f Spleen abdominal X-ray assessment, 185f congenital asplenia, 251 erythrocytes, 251 lymphocyte production, 251 size, 251 Splenectomy indications, 253 management in post-operative period, 254 splenic artery aneurysm, 292 Splenic artery aneurysm abdominal X-ray of calcication, 285f, 286f, 288 pseudoaneurysm differentiation, 289 risk factors, 290 treatment, 292 Splenomegaly case examples with abdominal X-ray, 247f, 248f, 249254, 633f, 634f causes, 252 physical examination, 250 portal hypertension, 290 splenic artery aneurysm, 286f ultrasound, 560 Staphylococcus aureus, osteomyelitis association, 509 Sterilisation clips abdominal X-ray assessment, 186 displacement, 248f Stomach, volvulus, 213 Streptococcus pneumoniae, community acquired pneumonia, 110 Stroke computed tomography of haemorrhage, 525, 526f, 527 magnetic resonance imaging, 546f Subluxation, 339, 340f SUFE, see Slipped upper femoral epiphysis Superior vena cava, stenting in Pancoast tumour, 84 Supracondylar fracture, see Elbow Syndrome of inappropriate antidiuretic hormone secretion (SiADH), lung cancer, 74 Tachycardia, tension pneumothorax, 41 Talus, shift, 518519 TB, see Tuberculosis Tenesmus, constipation, 244 Teratoma, mediastinal mass differential diagnosis, 142143 Thrombolysis, pulmonary embolism management, 119 Thumbprinting, colitis, 218 Thurstand Holland fragment, 365 Thymoma, mediastinal mass differential diagnosis, 142143 Thyroid cancer, lobar lung collapse, 156 Tibia, see also Knee osteomyelitis case example, 505f, 506f, 507512 shaft fracture case example, 497f, 498f, 499504 Toxic megacolon, 231 Tranexamic acid, uterine broid management, 283 Trauma, initial series X-rays, 447 Trendelenburg sign, 432433 Trethowans sign, 366, 455 Triplane fracture, 414 Triquetral bone, see Wrist Tuberculosis (TB) differential diagnosis, 89

Index 703

INDEX
hilar mass, 73 pleural effusion induction, 66 Tubogram, 576, 577f Ulcerative colitis, see Inammatory bowel disease Ulna distal fracture, see Wrist shaft fracture, 663f, 664f Ulna variance, 355 Ultrasound scans abdominal aortic aneurysm, 297 abdominal ultrasound, 559f, 560f B-mode, 557 chest ultrasound, 558, 559f colour ow imaging, 557, 563f congestive cardiac failure echocardiogram, 127 contraindications, 558 deep vein thrombosis, 118 Doppler imaging, 557558, 559f focused assessment with sonography in trauma, 448, 563 gallstones, 260, 274 guided procedures, 564 indications, 557 musculoskeletal ultrasound, 564 neck, 558f, 559f pelvic ultrasound, 562f pleural effusion, 6768 principles, 557 reection of waves, 558 safety, 557 vascular ultrasound, 563f Ultrasound therapy, uterine broid management, 284 Urinary incontinence, cauda equina syndrome, 370 Urinary tract infection, renal calculi, 266 Uterine broid abdominal X-ray of calcication, 277f, 278f case example with abdominal X-ray, 277f, 278f, 279284 magnetic resonance imaging, 282 management, 283284 sites, 281 symptoms, 280 VACTERL, duodenal atresia, 325 Varicella pneumonia, calcication on X-ray, 180 Veil sign, 26f Ventilation/perfusion scan, see Lung ventilation/perfusion scan Vertebral body osteitis, ankylosing spondylitis, 309 Volar angulation, 355 Volar Bartons fracture, 365 Volvulus sigmoid colon volvulus, 211f, 212f, 213216 sites, 213 Wackenheims line, 366 Wall test, 308 Warfarin initial prothrombotic effects, 119 pulmonary embolism management, 119 Water skiing, pneumoperitoneum induction, 135 Weber classication, malleolus fractures, 363, 517, 686f Wegeners granulomatosis, 148 Westermarks sign, 114f Wilms tumour, 89 Wrist carpal tunnel anatomy, 422 orthopaedic X-ray assessment, 354f, 355f, 356f case examples Colles fracture, 423f, 424f, 425428 distal radius fracture, 407f, 408f, 409414, 649f, 650f, 669f, 670f, 679f, 680f scaphoid fracture, 415f, 416f triquetral fracture, 356f, 683f, 684f distal radius radiological measurements, 428 radial inclination, 355 radius fracture, 356f Smiths fracture, 365, 427428 ulna variance, 355 volar angulation, 355 X-ray, see also specic areas and structures densities, 12f dose, 192193, 523 generation, 11f hazards, 13 image production, 12 magnication of anteroposterior view, 12, 13f pregnancy, 14 regulatory legislation, 1314 requesting form, 1415 radiology team interactions, 1516

704 Index

Introduction Chest X-Rays Abdominal X-Rays Orthopaedic X-Rays CT Scans MRI Scans USS Scans Nuclear Medicine Scans Fluoroscopy Bonus Chest X-Rays Bonus Abdominal X-Rays Bonus Orthopaedic X-Rays

The Unofficial Guide to Radiology


The Unofficial Guide to Radiology follows on from the The Unofficial Guide to Passing OSCEs. This book teaches systematic analysis of the three main types of X-rays: chest, abdominal and orthopaedic, with additional chapters looking at all the other main radiology tests such as CT and MRI. The layout is designed to make the book as relevant to clinical practice as possible; the X-rays are presented in the context of a real life scenario. The reader is asked to interpret the X-ray before turning over the page to reveal a model report accompanied by a fully annotated version of the X-ray. To further enhance the clinical relevance, each case has 5 clinical and radiology-related multiple-choice questions with detailed answers. These test core knowledge for exams and working life, and illustrate how the X-ray findings will influence patient management.

This book is suitable for:


Medical Students Radiographers Nursing Students Junior Doctors Physicians Associates Nurses Advanced Nurse Practitioners

Radiology is a constant challenge for students and doctors in busy clinical units: having a good command of the essentials is a real advantage. This book is well-presented and very accessible. The annotated examples provide realistic challenges with immediate feedback. It didnt take long before I felt better prepared for my next ward round!

Simon Maxwell, Professor of Student Learning, University of Edinburgh


Perhaps one of the biggest strengths of this book is the cases section, allowing you to practice not only interpreting high quality images but also to link them to a case history. The questions that follow not only test your radiology, but also your understanding of signs, symptoms, underlying pathophysiology and management of the condition. As well as detailed answers in each section, the book also shows you the best way to present each case, whether in an OSCE situation or on a ward round. The ease of use, detailed pictures and emphasis on key points of this one should cement it as the number one undergraduate book for radiology.

James Brookes, Medical Student


Which radiographs from each system are most likely to be presented in exams? This excellent book presents the classics, and at one level this makes it a high-yield textbook that will be extremely valuable to medical students and junior doctors. What is especially striking is the definition and clarity of the illustrations, with on-image labelling enabling one to be absolutely certain of which is the endotracheal tube, the nasogastric tube and the central line, for example.

Bob Clarke, Associate Dean, Professional Development, London. Director, Ask Doctor Clarke Ltd.
ISBN: 978 0 9571499 4 6

RRP 39.99

Join Our Medical Book Writing Project (details inside)

S-ar putea să vă placă și